Aiims PG 2005 (Based On Memory)

Download as pdf or txt
Download as pdf or txt
You are on page 1of 197

www.EduMedWeb.com www.EduMedWeb.

com Page 1

AIIMS PG 2005
(Based on Memory)
1. Hypertrophic pulmonary osteoarthropathy is seen more commonly with which of the following
lung carcinoma’s
a. Squamous cell carcinoma
b. Adenocarcinoma
c. Epidermoid carcinoma
d. Oat cell carcinoma

2. By knowing the half life of a drug it would help in knowing the


a. Median lethal dose
b. Median effective dose
c. Potency of drug
d. Frequency of dosing

3. Which of the following types of human papiloma viruses is implicated in causation of


carcinoma cervix
a. 6
b. 11
c. 16
d. 42

4. The best way to visualize the corneoscleral angle is


a. Slit lamp microscopy
b. Gonioscopy
c. Opthalmoscope
d. Keratometer

5. Mebendzole is used for treatment of all of the following except


a. Intestinal ameobiasis
b. Trichuriasis
c. Ankylostoma
d. Round worm

6. Which of the following causes visceral larva migrane


a. Toxocara catis
b. Ankylostoma doudenale
c. Loa Loa
d. Trichuris trichura

7. Which of the following is content of littre’s hernia


a. Sigmoid colon
b. Urinary bladder
c. Meckel’s diverticulum
d. Ileus

8. Amino Acid Score (AAS) refers to


a. The ratio of number of essential amino acids present in the given diet to reference protein
b. The ratio of number of limiting amino acids present in the given diet to reference protein

www.EduMedWeb.com www.EduMedWeb.com www.EduMedWeb.com


www.EduMedWeb.com www.EduMedWeb.com Page 2

c. The number of limiting amino acids present in the given diet


d. The number of essential amino acids present in the given diet
9. Most common tumor of the appendix is
a. Adenocarcinoma
b. Carcinoid
c. Squamous cell carcinoma
d. Metastasis

10. Formation of ammonium ions to remove the excess acid in urine is produced by which of the
following enzymes
a. Glutaminase
b. Glutamate pyruvate transaminase
c. Glutamate dehydrogenase
d. Carboxylase

11. The normal PR interval is in seconds


a. 0.05-0.08
b. 0.12-0.16
c. 0.8-1.6
d. 1.2-2.3

12. The normal sperm count is


a. 4-5 million/mm3
b. 60-120 million/mm
c. 60-120 lakhs/ml
d. 40-50 lakhs/mm3

13. To balance pressure natriuresis, the renel sodium excretion is


a. Increased with Increased arterial pressure
b. Decreased with Increased pressure
c. Decreased with decreased pressure
d. Increased with decreased pressure

14. The limiting amino acid in wheat is


a. Lysine
b. Threonine
c. Tryptophan
d. Cysteine

15. Chronic retropharyngeal abcess is formed due to


a. Acute cervical lymphadenitis
b. Caries spine
c. Tonsillitis
d. Trauma

16. Post cricoids malignancy is commonly in


a. Alcoholics
b. Smokers
c. Plummer vinsons syndrome
d. Reflux esophagitis

www.EduMedWeb.com www.EduMedWeb.com www.EduMedWeb.com


www.EduMedWeb.com www.EduMedWeb.com Page 3

17. The Henderson Hassalbach equation is used for measuring


a. The sodium potassium balance
b. The acid base balance
c. The kinetics of enzymatic reaction
d. Anion gap

18. All of the following are anabolic except


a. Growth hormone
b. Cortisol
c. Insulin
d. IGF – 1

19. The right homonymous hemianopia the lesion is in


a. The right optic nerve
b. Optic chiasma
c. The left Optic tract
d. The Optic radiations in the left cortex

20. Osteochondritis of the lunate bone is known as


a. Friebergs
b. Keinbocks
c. Perthes
d. Kohlers

21. The incubation period of mumps is


a. 7 days
b. 10 days
c. 18 days
d. 25 days

22. To differentiate between osmotic and secretary diarrhea it is best to measure


a. The stool osmolol gap
b. The stool anion gap
d. The water content of the stools

23. Cobble stone appearance is seen in


a. Epidemic keratoconjunctivitis
b. Vernal conjunctivitis
c. Trachoma
d. Herpes infection

24. The difference between use a selective alpha 1 blocker to using a non selective alpha
blocker is
a. Less reflux tachycardia
b. Decreased postural hypotension
c. More vasodilation
d. Vasoconstriction

25. All of the following are epinephrine on beta receptors except


a. Tachycardia
b. Ventricular arrythmia’s

www.EduMedWeb.com www.EduMedWeb.com www.EduMedWeb.com


www.EduMedWeb.com www.EduMedWeb.com Page 4

c. A-V block
d. Increased cardiac output
26. Symes amputation is of which of the following types
a. Provisional non end bearing
b. Provisional end bearing
c. Definitive non end bearing
d. Definitive end bearing

27. All of the following are true about amiodarone


a. It acts on fast sodium channels
b. Increase action potential
c. It decreases conduction in HIS purkinje fibres
d. It increase refractory period on A-V node

28. The cell of origin in giant cell tumor is


a. Osteoblast
b. Osteoclast
c. Undifferintiated cells
d. Giant cells

29. In vaginal prolapsed the cause for decubitus ulcer is


a. Friction
b. Congestion
c. Infection
d. Mechanical

30. DIC is seen in all of the following except


a. Intra uterine death
b. Extraamniotic instillation of ethacrydine lactate
c. Blood loss
d. Amniotic fluid embolism

31. All of the following are conservative management used in ectopic pregnancy except
a. Salpingostomy
b. Salpingotomy
c. Salpingectomy
d. Methotrexate

32. Hyperemisis Gravidarium in 1st trimester is seen with increased frequency in all of the
following except
a. H.Mole
b. Twins
c. Preeclampsia
d. Primigravida

33. Volume of synovial fluid in knee joint is


a. 0.5-4 ml
b. 4-8 ml
c. 10-20 ml
d. 8-12 ml

www.EduMedWeb.com www.EduMedWeb.com www.EduMedWeb.com


www.EduMedWeb.com www.EduMedWeb.com Page 5

34. The increased production of vitamin D induced transport protein increases absorption of
a. Iron
b. Magnesium
c. Calcium
d. Zinc

35. The blocking antibodies in pernicious anemia are


a. Directed against parietal cells
b. Prevents binding of IF and B12
c. Prevents absorption of IF and B12 complex
d. Are directed against enterocytes in ileum

36. Most common site of urethral rupture is


a. Prostatic
b. Penile
c. Bulbar
d. Membranous

37. WHO day is celebrated all over the day on


a. April 10
b. April 7th
c. May 31st
d. Dec 1st

38. Small Pox was declared globally aradicated on


a. 7th May 1977
b. 1975
c. 8th May 1980
d. 14th April 1977

39. The membranous separating the region between the feotal skull bones is called as
a. Fontanels
b. Wormian Bones
c. Sutures
d. Ostia

40. Bisphosphonates act by


a. Preventing calcium absorption in GIT
b. Inhibiting osteoclast mediated bone resorption
c. Preventing Ca excretion

41. Altering blood pH and preventing Ca resorption


a. Odd’s ratio measures the strength of association and
b. The ratio of incidence of disease in exposed to nonexposed
c. The risk factors and outcome
d. The ratio between exposed to the incidence of disease in the total population

42. The structures marking the separation of the duodenum and the jejunum is
a. The superior mesenteric vein
b. The bile duct
c. The ligament of trietz

www.EduMedWeb.com www.EduMedWeb.com www.EduMedWeb.com


www.EduMedWeb.com www.EduMedWeb.com Page 6

d. The caudate lobe of liver


43. In an immunosupressed individual suffering from viral, opportunistic fungal and parasitic
infections is suffering mainly from a defect in
a. B cells
b. T cells
c. Complement
d. Neutrophils

44. For contraception (post contraception) all of the following are used except
a. Mifepristone
b. Oral contraceptives
c. Levonorgestrol
d. Prostaglandins

45. All of the following are used in treatment of endometriosis except


a. Danazol
b. Gonadotropin releasing hormone
c. OCP’s
d. Gonasotropin releasing hormone agonist

46. During a surgery ligation of which of the following small veins in the abdomen can lead to
death of the patient
a. IVC
b. Superior mesenteric vein
c. Coronary vein
d. Splenic vein

47. All of the following are associated with maculopapular rash except
a. Typhoid
b. Dengue
c. Chicken pox
d. Measles

48. Punctate basophilia is seen in


a. Erythrocytes
b. Nuetrophils
c. Lymphocytes
d. Eosinophils

49. The fibrous layer is missing in


a. Pars tensa
b. Pars flaccid
c. Umbo
d. Anullus of tympanic membrane

50. Myotonic dystrophy inheritance is by


a. Autosomal Dominant
b. X-linked recessive
c. Autosomal recessive
d. Multifactorial

www.EduMedWeb.com www.EduMedWeb.com www.EduMedWeb.com


www.EduMedWeb.com www.EduMedWeb.com Page 7

51. X-linked dominant inheritance pattern is seen in


a. Polycystic Kidney disease
b. Vitamin D resistant rickets
c. Cystic fibrosis
d. Von willibrands disease

52. Von willibrands disease is transmitted as


a. Autosomal recessive
b. Autosomal Dominant
c. X-linked recessive
d. Y-linked recessive

53. Internal hordeolum is


a. Chronic granulomatous infection of Meibomian glands
b. Acute supperative infection of Meibomian glands
c. Acute infection of glands Zei’s
d. Lacrimal gland infection

54. The protein content in breast milk is


a. 0.8 gm/100 ml
b. 0.9 gm/10 ml
c. 1 gm/10 ml
d. 2 gms/100 ml

55. Dysphagia lusoria is due to


a. Abnormal origin of left subclavian artery
b. Abnormal origin of right subclavian artery
c. Compression by aortic arch
d. Obstruction due to foreign body

56. Sensitivity is written in the formula as


a. TP/FP+FN
b. TP/FN+TN
c. TP/TN+TP
d. TP/TN+FP

57. Annual parasite incidence measures


a. Number of confirmed cases due to malaria in 1 yr *1000 / population under surveillance
b. Number of cases of malaria due to falciparam in 1 yr *1000 / Total population
c. Number of slides examined *1000 / Total population
d. Number of faciparum malaria cases in 2-10 yr old children *1000 / Total population

58. As per WHO TB is said to be under control when


a. <1% population is tuberculin positive
b. <1% population is sputum positive
c. <1% infection children less then 14 years
d. <5% TB cases

59. Tullio Phenomenon refers to


a. Increased sensitivity to hearing
b. Better hearing in noisy surrounding

www.EduMedWeb.com www.EduMedWeb.com www.EduMedWeb.com


www.EduMedWeb.com www.EduMedWeb.com Page 8

c. Impedence matching
d. Vertigo on hearing high freq sounds

60. Thornwalds disease is infection of


a. Pharyngeal bursa
b. Space of gillete
c. Crypta magna
d. Subhyoid infection

61. All of the following are true regarding ulcerative colitis except
a. Decreased goblet cells
b. Crypt abcess
c. Destruction of crypts
d. Transmural infraction

62. Avoiding reflex is due to


a. Right frontal lobe disease
b. Dominant parietal lobe disease
c. Non dominant parietal lobe disease
d. Left Frontal lobe disease

63. All of the following are true regarding cerebellar lesion except
a. Hypotonia
b. Adiodokinesia
c. Resting tremors
d. Ataxia

64. Normal oxygen tension with decreased oxygen carrying capacity is seen in
a. Histotoxic hypoxia
b. Anemic hypoxia
c. Stagnant hypoxia
d. Hypoxic hypoxia

65. Loss of speech with inability to read, understand in a patient is referred to as


a. Dyslexia
b. Aphasia
c. Disconnection syndrome
d. Apraxia

66. Abelson oncogene is associated with which of the following leukemias


a. Acute myeloid leukemia
b. Acute leukemic leukemia
c. Chronic myeloid leukemia
d. Chronic leukemic leukemia

67. In CML the translocation occurring is


a. Abl of chromosome 9 to bcr of chromosome 21
b. Abl of chromosome 9 to bcr of chromosome 22
c. Bcr of chromosome 9 to abl of chromosome 21
d. Bcr of chromosome 9 to abl of chromosome 22

www.EduMedWeb.com www.EduMedWeb.com www.EduMedWeb.com


www.EduMedWeb.com www.EduMedWeb.com Page 9

68. M4 in the classification of AML denotes which of the following stages


a. Promyelocytic
b. Monocytic
c. Myelomonocytic
d. Megakaryocytic

69. The m4 stage in AML is associated with the presence of


a. Megaloblasts
b. Myeloblasts
c. Both myeloblasts and megaloblasts
d. Promylocytes

70. KANAVEL’s sign is seen in


a. Acute tenosynovitis
b. Pancreatitis
c. Appendicitis
d. Ectopic pregnancy

71. The most common cause of multiple strictures in the small intestine is
a. TB
b. Ischeamic enteritis
c. Crohns disease
d. Ulcerative colitis

72. In a post appendectomy patient the cause of an external fistula formation is


a. TB
b. Intussuception
c. Crohns disease
d. Ulcerative colitis

73. Feacal fistula at the umbilicus is due to


a. Patent urachus
b. Vescico intestinal diverticulum
c. Omphalocele
d. Infection

74. Most common cause of vescico vaginal fistula in developing countries is due to
a. Obstructed labour
b. Werthiems hysterectomy
c. Ceasarian section
d. Forceps delivery

75. All of the following operations are done in incontinence except


a. Kelleys stitch
b. Aldridge’s operation
c. Marshal marchetti krantz operation
d. Shirodhkars sling operation

76. Most common type of incontinence seen in UTI is


a. False
b. Overflow

www.EduMedWeb.com www.EduMedWeb.com www.EduMedWeb.com


www.EduMedWeb.com www.EduMedWeb.com Page 10

c. Urge
d. True

77. Deep tranverse arrest is commonly associated with which of the following types of pelvis
a. Platypelloid
b. Android
c. Anthrapoid
d. Gyenecoid

78. Which of the following is responsible for inability to rotate anteriorly in the occipitoposterior
position
a. Moderate size fetus
b. Gynecoid pelvis
c. Weak uterine contractions

79. Which of the following sinuses is commonly associated with development of osteomyelitis in
the face
a. Maxillary
b. Ethmoid
c. Frontal
d. Sphenoid

80. Atropine causes decrease salivation and bronchodilatation. These properties enable it to
used in as
a. Preaneasthetic medication
b. Hyertension
c. Supraventricular tachycardia
d. MI

81. All of the following are true about succinylcholine except


a. Causes persistent depolarizing block
b. Metabolized by psuedocholinesterase
c. Block can be reversed by neostigmine
d. Has a short half life

82. Maculopaular rash is seen is all of the following except


a. Typhoid
b. Varicella
c. Dengue
d. Measles

83. Diastomatomyelia is
a. Discontinuity in spinal cord
b. Spur in spinal cord
c. Absence of dorsal column
d. Dislocated disc

84. The weight of the body is transmitted through the axis of the vertebrae passing through
a. Lamina
b. vertebral Body
c. Pedicle

www.EduMedWeb.com www.EduMedWeb.com www.EduMedWeb.com


www.EduMedWeb.com www.EduMedWeb.com Page 11

d. Dorsal arch

85. All of the following are selective beta blockers except


a. Atenolol
b. Metoprolol
c. Bisoprlol
d. Propranolol

86. All of the following are seen in Tetralogy of Fallot except


a. VSD
b. Overiding aorta
c. Pulmonary stenosis
d. Left ventricular hypertrophy

87. In most patients with multiple sclerosis, CSF proteins are


a. Increased
b. Decreased
c. Normal
d. Absent

88. In Freidrichs type V hyperlipedemias the protein reised is


a. IDL
b. VLDL
c. LDL
d. Chylomicronsii

89. Atoprine is used in treatment of all the following except


a. Iridocyclitis
b. Angle closure glaucoma
c. Corneal ulcer
d. Before refraction assessment

90. Wickhams striae is characteristically is seen in


a. Psoriasis
b. Lichen planus
c. Pemphigus
d. Pemphigoid

91. Rhinitis sicca involves


a. Anterior nares
b. Septum
c. Posterior wall
d. Lateral wall

92. Acute pancreatitis is caused most commonly by


a. Gallstones
b. Alcohol
c. Trauma
d. Drugs

93. All of the following about meperidine are true except

www.EduMedWeb.com www.EduMedWeb.com www.EduMedWeb.com


www.EduMedWeb.com www.EduMedWeb.com Page 12

a. Has shorter half life then morphine


b. Greater spasmogenic action then morphine
c. Has dependency
d. Better oral availability

94. The lateral boundary of femoral canal is formed by


a. Lacunar ligament
b. Femoral ligament
c. Femoral vein
d. Femoral nerve

95. Posterior relation of Winslows foramen is


a. IVC
b. Liver
c. Doudenum
d. Pancreas

96. The death sentence given by a judicial officer has to be confirmed by


a. Sessions court
b. Additional sessions court
c. High court
d. Supreme court

97. The asbestos which of least likely pathogenic is


a. Crocodolite
b. Crysolite
c. Amsolyte
d. Trenolite

98. Most common surgical complication of typhoid is


a. Myocarditis
b. Cholecystitis
c. Hemorrhage
d. Ileal perforation

99. In a patient with highly selective protienuria the protein lost is


a. Transferrin
b. Albumin and fibrinogen
c. Transferrin and fibrinogen
d. Fibronogen

100. The immunologic response giving protection to Hepatitis A for long periods
a. IgM
b. IgG
c. IgA
d. IgD

101. In window period in Hepatitis B infection, the severly shows


a. HbsAg
b. Anti HbsAg
c. HbeAg

www.EduMedWeb.com www.EduMedWeb.com www.EduMedWeb.com


www.EduMedWeb.com www.EduMedWeb.com Page 13

d. Anti HbeAg

102. Characteristic skin lesion in Peutz Jeghers syndrome is


a. Freckles
b. Lentigenes
c. Café u-lait spots
d. Adenoma Sebacium

103. Characteristic lesion in tuberous sclerosis


a. Café u-lait
b. Adenoma sebaceum
c. Lentigenes
d. Melanoma

104. All of the following are common uses of blockers and calcium channel blockers except
a. Atrial arrhythmias
b. Hypertension
c. Variant angina
d. Migraine

105. Cardiac output is decreased by


a. Increased sympathetic activity
b. Tachycardia
c. Increased preload
d. Decreased venous return

106. The parasympathetic effect on bronchial smooth muscle causes


a. Increased tidal volume
b. Bronchoconstriction
c. Bronchodilatation
d. No effect

107. All of the following are true about cochlea except


a. Lesion in cochlea causes nerve deafness
b. Contains the tectorial membrane which is fine tuned
c. It contains the hearing sense organs
d. Lesion causes vertigo

108. Myopia is best corrected by


a. Convex lenses
b. Concave lenses
c. Astigmatic lenses
d. Prisms

109. All of the following are causes of complicated cataract except


a. Malignant Myopia
b. Diabetes mellitus
c. Retinitis Pigmentosa
d. Iridocyclitis
110. The most common causes of psuedohermaphroditism is
a. Ovarian dysgenesis

www.EduMedWeb.com www.EduMedWeb.com www.EduMedWeb.com


www.EduMedWeb.com www.EduMedWeb.com Page 14

b. Ovarian tumor
c. Congenital adrenal hyperplasia
d. PCOD

111. The D/D of rectal prolapsed in a child is


a. Intusseception
b. Rectal polyp
c. Heamorrhoids
d. Tumor

112. Most common causes of stridor in newborn is


a. Subglottic stenosis
b. Laryngomalacia
c. Foreign Body
d. Infection

113. All of the following are functions of saliva except


a. Acts as lubricant for mastication
b. It helps in perception of taste by dissolving
c. It helps in digestion of carbohydrates
d. It helps in digestion of proteins

114. The blood supply of SA node is


a. Left coronary artery
b. Right coronary artery
c. Left circumflex artery
d. Anterior descending artery

115. The graph used to measure to unrelated variables like height and weight are
a. Scatter diagram
b. Bar chart
c. Pie chart
d. Histogram

116. Reverse intussuception occurs in


a. Ileocolic
b. Ileoceacal
c. Colocolic
d. Jejunogastric

117. Aspirin can be used in all of the following except


a. Dysmennorhes
b. Acute inflammation of joints
c. Post MI
d. Viral infection

118. All of the following are true about metropathia heamoragica except
a. Swiss cheese endometrium
b. Common nearing menopause
c. Anovulatory cycles
d. Metrorhagia

www.EduMedWeb.com www.EduMedWeb.com www.EduMedWeb.com


www.EduMedWeb.com www.EduMedWeb.com Page 15

119. Savlon contains


a. Cetavlon + hibitane
b. Cetavlon + Chlorheximide
c. Hibitane + Chlorhexenol
d. Cetavlon + Chlorohexenol

120. Most common adverse effect of Ziduvudine is


a. Myopathy
b. Bone marrow toxicity
c. Hemolytic anemia
d. Hepatitis

121. All of the following are advantages of using INH in anti TB treatment except
a. It reduces load of TB bacilli
b. It kills intracellular TB bacilli
c. It acts on dormant bacilli
d. It has preventable side effects like peripheral neuropathy

122. The only drug available for treatment of nephrogenic diabetes insipidus is
a. Chlorpropamide
b. Thiazide
c. Desmopressin
d. Frusemide

123. All of the following are physiological cysts in the ovary except one which is a tumor
a. Theca cysts
b. Dermoid
c. Luteal cysts
d. Corpus luteum cysts

124. Development of testes is dependant upon which of the following


a. X chromosome
b. Y chromosome
c. XY chromosomes
d. XX chromosomes

125. Turners syndrome karyotype is


a. 45, XXY
b. 45 x 0
c. 45, XX
d. 45, XY

126. All of the following are indications of enucleation except


a. Malignant myopia
b. Pthysis bulbi
c. Panopthalmitis
d. Total anterior staphyloma

127. Purely ketogenic amino acid


a. Proline

www.EduMedWeb.com www.EduMedWeb.com www.EduMedWeb.com


www.EduMedWeb.com www.EduMedWeb.com Page 16

b. Tyrosine
c. Leucine
d. Phenylanine

128. Ponderal index is


a. Wt/Ht2
b. 100-ht in cms
c. Wt/cube root of Ht
d. Observed weight/Expected weight

129. WHO’s definition of blindness is


a. <6/60
b. <3/60
c. <20/60
d. <1/60

130. All of the following are seen in abruption placenta except


a. Tenderness over the abdomen
b. Renal failure
c. Uterine inversion
d. DIC

131. PQLI includes


a.IMR, life expectancy at 1 year, economic growth product
b.IMR, life expectancy at 1 year, literacy
c. MMR, literacy economic growth product
d. IMR, life expectancy at birth and knowledge

132. Freshly prepared bleaching powder yields what % of chlorine


a.66%
b.33%
c.50%
d.20%

133. Iodized salt at production level and at consumer level should be


a. 45 ppm and 15 ppm
b. 30 ppm and 15 ppm
c. 40 ppm and 20 ppm
d. 20 ppm and 10 ppm

134. The toxin implicated in lathyrism is


a. Pyrralizidone
b. BOAA
c. Sanguinarine
d. Aflatoxin

135. Chronic noise exposure above what level causes deafness


a. 100 Db
b. 85 Db
c. 70 Db
d. 140 Db

www.EduMedWeb.com www.EduMedWeb.com www.EduMedWeb.com


www.EduMedWeb.com www.EduMedWeb.com Page 17

136. All of the following are seen in horners syndrome except


a. Ptosis
b. Miosis
c. Proptosis
d. Anhydrosis

137. Antigen drift involving influenza indicates


a. Major shift in the structure of the antigens
b. Minor variations in the viral nucleocapsid
c. Minor antigenic variations in the heamagluttinin and neuroaminidase antigens
d. Recombination with host DNA

138. Skin contains which type of glands largely


a. Apocrine and sweat glands
b. Ecocrine and sweat glands
c. Holocrine and sweat glands
d. Sebacious and sweat glands

139. Acute flaccid paralysis is suddenly caused by


a. Hypercalcemia
b. Hypocalcemia
c. Hyperkalemia
d. Hypokalemia

140. In a patient with polyuria and polydipsia with passage of dilutes urine is suffering from
abnormality involving
a. Aldosterone
b. ADH
c. Insulin
d. Growth hormone

141. Chlamydia is best treated by


a. Tetracycline
b. Ampicillin
c. Gentamycin
d. Cephalexin

142. True about competitive inhibitor is


a. Decrease in Km and Vmax
b. Decrease in Vmax and Km is same
c. Increase in Km and Vmax is same
d. Decrease in Km and Vmax is same

143. Best drug to potentiate DOPA is


a. Pyridoxine
b. Carbidopa
c. Pergolide
d. Amantadine
144. Current modes of investigation for infertility to check functioning of tubes are all of the
following except

www.EduMedWeb.com www.EduMedWeb.com www.EduMedWeb.com


www.EduMedWeb.com www.EduMedWeb.com Page 18

a. Air insufflations
b. Sonosalpingography
c. Hydterosalpingography
d. Laproscopic chromotubation

145. Kernicterus is caused by


a. Bilirubin monoglucoronide
b. Albumin bound bilirubin
d. Bilirubin diglucoronide

146. Mechanism of action of cluvalinic acid is


a. Inhibits cell wall synthesis
b. Inhibits beta lactamase
c. Acts on protein synthesis
d. Causes miss reading of DNA

147. Prolactin secretion is inhibited by


a. Growth hormone
b. Stomatostatin
c. Dopamine
d. TSH

148. In increased stage of endometrial carcinoma treatment given is


a. Tamoxifen
b. Mrdroxyprogesesterone
c. Danazol
d. Methotrexate

149. Normal fluoride level for drinking purposes is in mgs/It


a. 0.08-0.5
b. 0.8-1.2
c. 1-1.75
d. 1.5-3.5

150. Epinephrine can be used in all of the following except


a. Anaphylactic shock
b. Chronic bronchial asthma
c. Cardiac resuscitation
d. To prevent local bleed

151. The most common nerve involved in leprosy is


a. Ulnar nerve
b. Lateral popliteal nerve
c. Sciatic nerve
d. Radial nerve

152. Dangerous area of face is


a. Olfactory bulb
b. Woodruffs area
c. Littles area
d. Inferior turbinate

www.EduMedWeb.com www.EduMedWeb.com www.EduMedWeb.com


www.EduMedWeb.com www.EduMedWeb.com Page 19

153. During surgery involving abdomen ligation of a small vein cause death of the patient
a. Coronary vein
b. Superior mesenteric vein
c. Splenic vein
d. IVC

154. The main blood supply of the small gut is


a. Celiac axis
b. Superior mesenteric vein
c. Inferior mesenteric vein
d. Middle colic vein

155. The external ear develops from which of the following


a. 1st branchial arch
b. 2nd branchial arch
c. 1st branchial cleft
d. 2nd branchial cleft

156. Division of nuclear material occurs in which of the following stages


a. Metaphase I
b. Prophase I
c. Anaphase
d. Prophase II

157. Rotation of radius occurring without extension and flexion at the elbow joint is caused by
a. Anconeus and supinator
b. Brachioradialis and Brachialis
c. Pronator quadrates and pronator teres
d. Triceps and Biceps brachii

158. Depression of mandible is caused by


a. Temporalis
b. Lateral pterygoid
c. Medial pterygoid
d. Masseter

159. Which of the following amino acids undergoes hydroxylation and is involved in the
formation of collagen
a. Proline
b. Tryptophan
c. Phenylalanine
d. Cystiene

160. Which of the following are true about action conduction


a. On activation there is influx of potassium and efflux of sodium
b. It is decremental in nature
c. It does not follow all or none law
d. It requires a threshold stimulus to be activated

161. The most physiological operation for chronic duodenal ulcer is

www.EduMedWeb.com www.EduMedWeb.com www.EduMedWeb.com


www.EduMedWeb.com www.EduMedWeb.com Page 20

a. Highly selective Vagotomy


b. Truncal vagotomy and gastrojejunostomy
c. Antrectomy and truncal vagotomy
d. Selective vagotomy

162. The least recurrence rate is seen in which of the following procedures for peptic ulcer
a. Trancal vagotomy and gastrojejunostomy
b. Highly selective Vagotomy
c. Trancal vagotomy and antectomy
d. Selective vagotomy

163. Most of the long bones in the body have which of the following type of joints
a. Synarthrosis
b. Dyarthrosis
c. Amphiarthrosis
d. Synostosis

164. The fungi present in the reticuloendothelial system is


a. Candida
b. Histoplasma capsulatum
c. Sporothrix
d. Aspergillus

165. Alpha waves are which of the following frequencies


a. 7-13 Hz
b. 10-20 Hz
c. 15-25 Hz
d. 4-5 Hz

166. In a patient who is resting but is awake and has his closed will show which of the following
readings on EEG
a. Alpha waves
b. Beta waves
c. Theta waves
d. Delta waves

167. Which of the following are true about neutrophils


a. Constitute 5-10% of leucocytes
b. Are phagocytic cells
d. Are increased in chronic inflammation
d. Are present mainly in extravascular regions

168. In a patient suffering through trauma involving regions that are extra abdominal can still
cause death due to which of the following reasons
a. Splenic rupture
b. Acute dilatation of stomach
c. Fat embolism
d. Intedtinal infarction

169. All of the following are true regarding Angle closure glaucoma except
a. Vertically dilated pupil

www.EduMedWeb.com www.EduMedWeb.com www.EduMedWeb.com


www.EduMedWeb.com www.EduMedWeb.com Page 21

b. Deep AC
c. Ciliary congestion
d. Haloes may appear

170. All of the following are true about angle closure glaucoma except
a. Prodormal phase initially
b. Atrpoine is used in treatment
c. Gonioscopy is used to visualize angle
d. Iridotomy is required

171. Which of the following drugs cause photoxicity


a. Phenytoin
b. Amiodarone
c. Digoxin
d. Verapamil

172. Erythema Nodosum is caused by


a. Sulphonamides
b. Phenytoin
c. Phenylbutazone
d. Penecillin

173. Which of the following cytokines synthesized by the LOX enzyme is involved in chemotaxis
a. B4
b. C4
c. D4
d. E4

174. True about TxA2 synthesized by endothelium is


a. Inhibits platelet aggregation
b. Causes vasoconstriction
c. All of the above
d. None of the above

175. Benzathine penicillin is used in treatment of


a. Endocarditis
b. Septicemia
c. Primary syphilis
d. UTI

176. Most common cause of infection in surgical wound is


a. Staph epidermidis
b. Staph aureus
c. Pseudomonas
d. Streptococci

177. Chronic exposure with which of the following particle size causes chronic lung disease
a. 0.5-3 microns
b. 5-10 microns
c. 3-5 microns
d. 0.01-0-5 microns

www.EduMedWeb.com www.EduMedWeb.com www.EduMedWeb.com


www.EduMedWeb.com www.EduMedWeb.com Page 22

178. Which of the following is a common child affective viruses


a. Adenovirus
b. CMV
c. Para influenza
d. Herpes simplex

179. Narcolepsy is
a. Narcotic induced epilepsy
b. Narcotic induced depression
c. Attacks of sleep and paralysis
d. Narcotic withdrawal reaction

180. The best way to gauge the IQ of a child is by looking at


a. Gross motor development
b. Fine motor development
c. Psychosocial
d. Language

181. Cherry red spot is seen in


a. CRVO
b. CRAO
c. Branch retinal vein occlusion
d. Papillodema

182. Cherry red spot is commonly seen in


a. Gauchers disease
b. Taysachs disease
c. Hurlers syndrome
d. Von gierkes disease

183. Cubitus varus deformity occurs most commonly due to


a. Malunited lateral condyle #
b. Malunited supracondylar #
c. Medial epicondyle #
d. Elbow dislocation

184. Vibices is another term for


a. Subcutaneous hypostasis
b. Arboresent markings
c. Lichtenburgs markings
d. Joule burns

185. When permanent canine appears it correlates with the appearance of which of the
following ossification centers
a. Olecranon
b. Pissiform
c. Patella
d. Head of femur

186. In comparison of famotidine to cimetidine. They differ in all of the following except

www.EduMedWeb.com www.EduMedWeb.com www.EduMedWeb.com


www.EduMedWeb.com www.EduMedWeb.com Page 23

a. Antiandrogenic effect
b. Oral bioavailability when given along with food
c. Duration of action
d. Side effects

187. Posterior pituitary stores and releases which of the following hormones
a. TSH and GH
b. TSH and LG
c. ADH and Prolactin
d. Oxytocin and vasopressin

188. Invitro fertilization is mainly used when the cause of infertility is due to
a. Oligospermia
b. Tubal abnormaities
c. Anovulation
d. Cord factor

189. Regarding water absorption in PCT all of the following are seen in except
a.60% reabsorbed
b. Depends on ADH
c. Passive
d. Active absorption of solutes

190. IV bicarbonate is given in


a. Metabolic alkalosis
b. Metabolic acidosis
c. Head injury
d. Uncompensated respiratory acidosis

191. Normal anion gap acidosis is seen in


a. Chronic renal failure
b. Diarrhea
c. Diabetic ketoacidosis
d. Methanol poisoning

192. The endothelial cells have specialized receptors for which of following
a. HDL
b. LDL
c. Oxidized LDL
d. Oxidized VLDL

193. Most common parotid tumor is


a. Pleomorphic adenoma
b. Adenocarcinoma
c. Warthins tumor
d. SCC

194. The toxin implicated in lathyrism is


a. Sanguinarine
b. Beta oxalyl amino acid
c. Pyrrazolidine

www.EduMedWeb.com www.EduMedWeb.com www.EduMedWeb.com


www.EduMedWeb.com www.EduMedWeb.com Page 24

d. Aflatoxin

195. The enzyme required for the generation of the ammonium ion in the kidney is
a. Glutamate dehydrogenase
b. Glutamate aspartate transaminase
c. Glutaminase
d. Glutamate carboxylase
-----------------------------------------------------------------------------------------------------------------------------

www.EduMedWeb.com www.EduMedWeb.com www.EduMedWeb.com


www.EduMedWeb.com www.EduMedWeb.com Page 1

AIIMS PG MAY 2006


(Based on Memory)
ANATOMY
1. The cell Junctions allowing exchange of cytoplasmic between two cells are called:
A. Gap Junctions
B. Tight Junctions
C. Anchoring Junctions
D. Focal Junctions
Ans: A

2. The cells belonging to the following type of Epithelium are provided with extra reserve cell-
membrane:
A. Transitional
B. Stratified squamous
C. Stratified Cuboidal
D. Stratified Columnar
Ans: A

3. All the following muscles are grouped together as muscle of mastication except:
A. Buccinator
B. Masseter
C. Temporalis
D. Pterygoids
Ans: A

4. The development of first connection tissue forms “non synovial joint” is called as:
A. Syndesmosis
B. Synchondrosis
C. Symphysis
D. Suture
ANS: B

5. The Buffering capacity of a buffer is maximum at pH equal to


A. 0.5pKa
B. pKa
C. pKa+1
D. 2pKa
Ans: B

BIOCHEMISTRY

6. Which of the following is not a post-transcriptional modification of RNA?


A. Splicing

www.EduMedWeb.com www.EduMedWeb.com www.EduMedWeb.com


www.EduMedWeb.com www.EduMedWeb.com Page 2

B. 5’ – capping
C. 3’ – poly adenylation
D. Glycosylation
Ans: D

7. The following separation techniques on molecular size of proteins?


A. Chromatography on a carboxymethyl cellulose column.
B. Iso-electric focusing
C. Gel Filtration chromatography
D. Chromatography on a diethylaminoethyl (DAAE) cellulose column.
Ans: C

8. R – Oxidation of odd – chain fatty acids produces:


A. Succinyl CoA
B. Propionyl CoA
C. Accetyl CoA
D. Malonyl CoA
Ans: B

9. The protein rich in basic amino acids which functions in the packaging of DNA in chro
mosomes is:
A. Histone
B. Collagen
C. Hyaluronic acid binding protein
D. Fibrinogen
Ans: A

10. The primary role of chaperones is to help in:


A. Protein synthesis
B. Protein degradation
C. Protein denaturation
D. Protein folding
Ans: D

DENTAL MATERIALS
11. The force applied during condensation of amalgam having a condenser measuring 2 mm in
diameter:
A. 3-4 lb
B. 8-17 lb
C. 10-15 lb
D. 7-12 lb
Ans: A

12. The angles between adhesive and adherent are zer degree. It indicates:

www.EduMedWeb.com www.EduMedWeb.com www.EduMedWeb.com


www.EduMedWeb.com www.EduMedWeb.com Page 3

A. Complete wetting of surfaces


B. Rough surfaces between adhesive and adherent
C. Irregularities present between the adherent surfaces.
D. The adherent and adhesive molecule are tangent to each other.
Ans: A

13. Which of the following is not a true wax?


A. Paraffin
B. Ozokerite
C. Cocoa butter
D. Candelilla
Ans: C

14. The half line of mercury in human body is:


A. 55 days
B. 55 hours
C. 55 week
D. 55 months
Ans: A

15. Thediameter of sqrue and sub-surface porosity will have:


A. Direct relationship
B. Inverse relationship
C. Inverse square relationship
D. No relationship
Ans: A

16. The length of sqrue and subsurface porosity will have:


A. Direct relationship
B. Inverse relationship
C. Inverse square relationship
D. No relationship at all.
Ans: B

17. Which of the following is not true about casting gypsum bonded investment material:
A. It is used for cast metal alloys
B. 50 – 65% of gypsum changes to form (-hemihydrate
C. The investment material is not heated above 700O temperature
D. Heating above 700O of investment causes formation of sulphur dioxide from copper sulphate.
Ans: B

18. Which of the following is not correct about dental stone?


A. High strength low expansion dental stone contains some additive to reduce expansion.
B. Dental stone is manufactured by heating at temperature 110O – 12O C in an open auto clave.

www.EduMedWeb.com www.EduMedWeb.com www.EduMedWeb.com


www.EduMedWeb.com www.EduMedWeb.com Page 4

C. Dental stone is much harder and stronger than B-hemihydrate.


D. Microscopically it is seen as cleavage fragments and crystals in the form of rods and prisms.
Ans: B

19. Like the eutectic transformation, the peritectic reaction ia an invariant reaction (i.e. it occurs
at a particular composition and temperature). The reaction can be written as:
A. Liquid + β → α
B. Liquid + β → Liquid + α
C. Liquid → α –solid solution + β-solid solution
D. α + β → Liquid
Ans: A

PATHOLOGY
20. Which of the following procedures is routine for karyotyping using light microscopy:
A. C-banding
B. G- banding
C. Q- banding
D. Brd V-Staining
Ans: B

21. An Albino girl gets married to a normal boy. What are the chances of their having an
affected child and what are the chances of their children being carriers:
A. None affected, all carriers
B. All normal
C. 50% carriers
D. 50% affected, 50% carriers
Ans: A

22. Osteomalacia is associated with:


A. Decrease in osteoid volume
B. Decrease in osteoid surface
C. Increase in osteoid maturation time
D. Increase in mineral opposition rate.
Ans: C

23. In leukemic patient, gingival bleeding occurs during oral prophylaxis because of
A. Increased leukocytes count
B. Increased calcium level in blood
C. Platelet disorder
D. Deficiency of clotting factor
Ans: C

24. The average number of CD4 ceels in body fluid is:


A. 800-1000/mm3

www.EduMedWeb.com www.EduMedWeb.com www.EduMedWeb.com


www.EduMedWeb.com www.EduMedWeb.com Page 5

B. 800-1200/mm3
C. 500-800/mm3
D. 400-600/mm3
Ans: B

MICROBIOLOGY
25. Heat labile instruments for use in surgical procedures can be sterilized by:
A. Absolute alcohol
B. Ultra violet rays
C. Chlorine releasing compounds
D. Ethylene oxide gas
Ans: D

26. A 60 year old man is diagnosed to be suffering legionnaire’s disease after returns from
attending a convention. He could have acquired it:
A. From a person suffering from the infection while traveling in the aeroplane
B. From a chronic carrier in the convention center
C. From inhalation of the aerosols in the air conditioned room at convention center
D. By sharing an infection towel with a fellow delegate at the convention
Ans: C

27. The most common micro organism causing sub-acute bacterial endocarditis after oral
surgical procedure is:
A. Streptococcus viridans
B. Streptococcus muta
C. Streptococcus aureus
D. Streptococcus albus
Ans: A

PHARMACOLOGY
28. An anesthetist orders a new attendant to bring the oxygen cylinder. He wiil attendant to
identify the correct cylinder by following color code:
A. Black cylinder with white shoulder
B. Black cylinder with grey shoulder
C. White cylinder with black shoulder
D. Grey cylinder with white shoulder
Ans: A

29. 50% eyelid Ptosis, blurring of vision, slurring of speech, indicating the correct level after
Diazepam sedation is best explained by:
A. Tinel’s sign
B. Verill’s sign
C. Corman’s sign
D. Bell’s sign

www.EduMedWeb.com www.EduMedWeb.com www.EduMedWeb.com


www.EduMedWeb.com www.EduMedWeb.com Page 6

Ans: B

30. All of the following regarding biovailability of a drug are true, except:
A. It is the proportion (fraction) of unchanged drug that reaches the systemic circulation
B. Bioavailability of an orally administered ddug can be calculated by comparing the under curve
(0- ) after oral and intravenous (iv) administration.
C. Low oral bioavailability always and necessarily means poor absorption.
D. Bioavailability can be determined from plasma concentration of urinary exctetion data.
Ans: C

31. Hemorrhage secondary to heparin administration can be corrected by administration of:


A. Vitamin K
B. Whole Blood
C. Protamine
D. Ascorbic acid
Ans: C

DENTAL HISTOLOGY
33. Which of the following does not contain Taste Buds?
A. Circumvallate papillae
B. Filliform papillae
C. Fungiform papillae
D. Foliate papillae
Ans: B

34. The disturbances occurred during “calcification” stage of tooth development are seen in:
A. Peg lateral
B. Microdontia
C. Supenumery tooth
D. Interglobular dentin
Ans: D

GENERAL MEDICINE
35. Thirty eight children consumed eatables procured form a single source at a picnic party.
Twenty children developed abdominal cramps followed by vomiting and watery diarrhea 6-10
hours after the party. The most likely etiology for the outbreak is?
A. Rotavirus infection
B. Entro-toxigenic E-coli infection
C. Staphylococcal toxin
D. Clostridium perfringens infection
Ans: D

www.EduMedWeb.com www.EduMedWeb.com www.EduMedWeb.com


www.EduMedWeb.com www.EduMedWeb.com Page 7

36. In a post-operative intensive care-unit, five patients develop post- operative wound imfection
in the same ward. The best method to prevent cross infection in other patients in the same ward
is to?
A. Give antibiotics to all other patients in the ward.
B. Fumigate the ward.
C. Disinfect the ward with sodium hypochlorite
D. Practice proper hand washing.
Ans: D

37. A 5 years old boy passed 18 loose stools in last 24 hours and twice vomited in last 4 hours.
He is irritable but drinking fluids. The optional therapy for this chid is:
A. Intravenous fluids
B. Oral rehydration fluids
C. Intravenous fluid initially for 4 hours follwed by oral fluids
D. Plain water ad libitun
Ans: B

38. Bedsore is an example of:


A. Typical ulcer
B. Trophic ulcer
C. Venous ulcer
D. Post-thrombotic ulcer
Ans: B

39. The most common histological type of thyroid cancer is:


A. Medullary type
B. Papillary type
C. Follicular type
D. Anaplastic type
Ans: B

COMMUNITY DENTISTRY & P.S.M


40. For the calculation of positive predictive of a screening test, the denominator is comprised:
A. True Positives + False Negatives
B. False Positives + True Negatives
C. True Positives + False Positives
D. True Positives + False Positives
Ans: C

41. The parameters of sensitivity and specificity are used for assessing:
A. Criterion Validity
B. Construct Validity
C. Discriminate Validity
D. Content Validity

www.EduMedWeb.com www.EduMedWeb.com www.EduMedWeb.com


www.EduMedWeb.com www.EduMedWeb.com Page 8

Ans: A

42. A measure of location which divides the distribution in the ratio of 3 : 1 is:
A. Mdian
B. First quartile
C. Third Quartile
D. Mode
Ans: C

43. Chi-Square test is used to measure the degree of:


A. Causal relationships between exposure and effect.
B. Association between two variables.
C. Correlation between two variables.
D. Agreement between two observations.
Ans: B

44. Elements of primary healthcare include all of the following except:


A. Adequate supply of safe water and basic sanitation.
B. Providing essential drugs
C. Sound referral system
D. Health education
An: C

45. In water Fluoridation of a Community which of the following sequential is correct:


A. Decayed tooth, central water suply agency, fluoride concentration in water and community
applicant.
B. Decayed tooth, fluoride concentration in water, central water suply agency and community
applicant.
C. Community applicant, decayed tooth, central water suply agency and fluoride concentration
in water.
D. Central water suply agency, decayed tooth, community applicant and fluoride concentration
in water.
Ans: B

46. The age of the child used in the determination of child mortality rate is:
A. 0-5 years
B. 1-4 years
C. 5-8 years
D. 0-1 year
Ans: B

47. Primordial prevention is used to:


A. Prevent development of risk factor in the community.
B. Prevent the transmission of diseases.

www.EduMedWeb.com www.EduMedWeb.com www.EduMedWeb.com


www.EduMedWeb.com www.EduMedWeb.com Page 9

C. Prevent the development of diseases.


D. To establish the diagnosis and treatment of diseases.
Ans: A

ORAL PATHOLOGY & MEDICINE


48. A tumor occurred in oral cavity which is represented as T3 N2 MO. This tumor is classified
as:
A. Stage I
B. Stage II
C. Stage III
D. Stage IV
Ans: D

49. The characteristic “Multiple punched out lesion” is seen in:


A. Multiple myeloma
B. Hyperparathyroidism
C. Fibrous Dysplasia
D. Osteosarcoma

Ans: A
50. The other name for factor XI is:
A. Stuart power factor
B. Calcium ion.
C. Plasma thromboplastin antecedet
D. Plasma thromboplastin Component
Ans: C

51. Which of the following sugars present in the human diet is considered the most promoting?
A. Sucrose
B. Fructose
C. Lactose
D. Glucose
Ans: A

52. The non-cariogenic substitute used to replace the cariogenic diet is


A. Lactose
B. Glucose
C. Cooked Starch
D. Sorbitol
Ans: D

53. Which of the following is most appropriate therapy for a patient suffering from lesions of
infectious mononucleosis?
A. Analgesics

www.EduMedWeb.com www.EduMedWeb.com www.EduMedWeb.com


www.EduMedWeb.com www.EduMedWeb.com Page 10

B. Antibiotic
C. Anticonvulsant
D. No therapy as it is self limiting disease
Ans: D

54. The most common site of oral cancer among Indian population is:
A. Tongue
B. Floor of mouth
C. Alveolobuccal complex
D. Lip
Ans: C

55. The minimum distance used between film, object and target during radiography is:
A. Four feet
B. Five feet
C. Six feet
D. Seven feet
Ans: B

56. The normal percentage of enlargement or magnification considered normal in cephalogram


is:
A. 1-2%
B. 6-7%
C. 10-13%
D. 14-16%
Ans: B

57. “Syncope” occurs while operative processes due to:


A. Cerebral edema
B. Cerebral hyperemia
C. Cerebral hypoxia
D. Cerebral degeneration
Ans: C

58. The straw colored proteinaceous fluid present in odontogenic keratocyst contains protein:
A. Less than 3.5 gm/dl
B. More than 3.5 gm/dl
C. Less than 3.5 mg/dl
D. More than 3.5 mg/dl
Ans: A

59. Which of the following is used in the management of hemophilic patient?


A. Tranexamic acid
B. Acetic acid

www.EduMedWeb.com www.EduMedWeb.com www.EduMedWeb.com


www.EduMedWeb.com www.EduMedWeb.com Page 11

C. Ascorbic acid
D. Palmitoic acid
Ans: A

60. Which of the following is correct reason of facial nerve injury during forceps delivery in
labour:
A. The mastoid process is absent at birth
B. The parotid glands is in developing stage
C. The beak of the forceps engages the main trunk of facial nerve.
D. The sublingual hematoma during delivery causes neuropraxia.
Ans: A

61. The first instrument used on the facial surfaces of gingiva while gingivectomy is:
A. Orban’s Knife
B. Kirkland Knife
C. BP blade with no. 11 blade
D. BP blade with no. 12 blade
Ans: B

62. Oral prophylaxis is contraindicated in:


A. Pre-pubertal gingivitis
B. Pregnancy gingivitis
C. Ulcerative refractory gingivitis
D. Leukemic gingivitis
Ans: D > C

63. Which of the following is used as a biodegradable membrane in “guided tissue regeneration”
process?
A. Polylactic acid
B. Polytetrafluroethylene
C. Millipore
D. Core membrane
Ans: A

64. Which of the following immunoglobulin is present in higher concentration in gingival


crevicular fluid?
A. Ig A
B. Ig G
C. Ig E
D. Ig M
Ans: B

65. Trauma from occlusion causes:


A. Periodontitis

www.EduMedWeb.com www.EduMedWeb.com www.EduMedWeb.com


www.EduMedWeb.com www.EduMedWeb.com Page 12

B. Gingivitis
C. Periodontal pocket
D. Widening of periodontal ligament.
Ans: D

66. The local drug delivery system “ELYZOL” contains:


A. Metronidazole
B. Penicillin
C. Sanguinarine
D. Tetracycline
Ans: A

67. The relationship of the denture base that resists dislodgement of denture in horizontal
direction is:
A. Stability
B. Pressure
C. Support
D. Retention
Ans: A

68. Relining of complete denture is not indicated when:


A. Denture contains broken teeth
B. There is excessive resorption of the ridge
C. Vertical dimension is excessively reduced and has to be changed
D. Centric relation does not coincide with centric occlusion
Ans: C

69. The most suitable margin design for all ceramic restoration is:
A. Shoulder
B. Chamfer
C. Shoulder with bevel
D. Depends upon operators choice
Ans: A

70. In fixed partial denture it is better for posterior pontic to avoid contacts in:
A. Working side contacts
B. Balancing side contacts
C. Side to side contact
D. Centric occlusion
Ans: B

71. The pontic design of choice in the appearance zone of maxillary and mandibular bridge:
A. Hygienic
B. Modified ridge lap

www.EduMedWeb.com www.EduMedWeb.com www.EduMedWeb.com


www.EduMedWeb.com www.EduMedWeb.com Page 13

C. Saddle
D. Conical
Ans: B

72. The pin used in pin restoration of a root canal treated tooth is:
A. Self threaded pin
B. Cemented pin
C. Frictional pin
D. Any of the above
Ans: B

73. Which of the following is not responsible for endogenous staining of teeth during
development?
A. Tetracycline
B. Rh incompatibility
C. Neonatal liver disease
D. Vitamin C deficiency
Ans: D

74. Difference between physical characteristics of reamers and files is:


A. The cross-section of reamers is square and files are triangular in cross section.
B. The numbers of flutes on the blade are more in files than in reamers.
C. The reamers have more flutes in the blade.
D. Files have two superficial grooves to produce flutes in a double helix design.
Ans: B

75. Buckley’s solution is composed of:


A. Cresol, formaldehyde, water and glycerin
B. Formaldehyde, resorcinol, water
C. Iodoform, glutaraldehyde, ZOE
D. Chloraldehyde, parachloral, methanol water
Ans: A

76. A dentist did restorative procedure in a 11 year old child and found the primary molar are in
various stages of exfoliation and there is slight anterior crowding presents between the anterior
teeth. The dentist should give his next appointment:
A. After three months for bservation
B. After six months for recall checkup
C. After one year
D. When all the permanent teeth will erupt
Ans: A

77. The rectangular wire used in edgewise appliance in primarily mean for:
A. Correction of crown root position

www.EduMedWeb.com www.EduMedWeb.com www.EduMedWeb.com


www.EduMedWeb.com www.EduMedWeb.com Page 14

B. Increasing the strength of wire


C. Correction of Arch-length Deficiency
D. Used for correction of anterior crowding
Ans: A

78. The buccal coil spring is used to regain the space present between premolar and first molar.
What is the most common complication post treatment?
A. Pain
B. Gingival irritation
C. Tendency for the 1st molar to intrude
D. Tendency for the for the 1st premolar to rotate
Ans: D

79. The arch space for eruption of permanent second and third molar is created by:
A. Apposition of alveolar precess
B. Resorption of posterior border of Ramus
C. Resorption of anterior border of Ramus.
D. Apposition of lower body of mandible.
Ans: C

80. The ANB angle of 2 degree usually indicats:


A. A favorable relationship of maxillary alveolar base to Mandibular alveolar bas
B. A favorable relationship of mandible to cranium.
C. Poor cranial growth
D. Upright incisors
Ans: A

81. Superposition in cephalometric studies is done from “registration point” This best
demonstrates:
A. Growth of structure farthest from point
B. Growth of structure nearest to point.
C. Growth at that point
D. All of the above
Ans: A

82. When a maxillary removable orthodontic appliance is first placed, the effect on the patient’s
speech will probably include:
A. Difficulty with lingual vowels for a few days.
B. Difficulty with lingual vowels for several weeks.
C. Difficulty with linguoalveolar consonants for a few days.
D. Difficulty with linguoalveolar consonants for several weeks.
Ans: C

www.EduMedWeb.com www.EduMedWeb.com www.EduMedWeb.com


www.EduMedWeb.com www.EduMedWeb.com Page 15

83. During smile a plesure carve is formed following the curve of interior teeth and some portion
of mesial surfaces of each tooth is visible. How much percentage of mesial surface teeth is
visible during smilling or “Golden Esthetic Rule” is:
A. 50%
B. 60%
C. 70%
D. 80%
Ans: B

84. Cephalometric analysis are used to evaluate growth changes by superimposing on:
A. Sella-nasion plane
B. Mandibular plane
C. F-H plane
D. Occlusal plane
Ans: A

85. The pulse-oxymetry is used in the determination of:


A. Rtae of flow
B. Oxygen Saturation
C. Blood Volume
D. Blood coefficint
Ans: B

86. The “Stainless Steel Crown” is invented by:


A. Humphrey
B. Willet
C. Cvek
D. Addleston
Ans: A

87. The non-invasive method to measure the blood flow is:


A. Electric pulp test
B. Percussion
C. Radiograph
D. Laser Doppler flometry
Ans: D

88. The KRI paste is composed of:


A. Iodoform camphor, parachlorophenol and menthol
B. Iodoform and ZOE
C. Parachlorophenol, camphor and menthol
D. Calcium hydroxide and iodoform
Ans: A

www.EduMedWeb.com www.EduMedWeb.com www.EduMedWeb.com


www.EduMedWeb.com www.EduMedWeb.com Page 16

89. While fabricating bilateral distal shoe appliance for maintaining space lost by primary molar,
the first factor considered is:
A. Tooth anatomy
B. Load of distribution
C. Flexibility of wire
D. Space present posterior to permanent
Ans: B

90. The distribution of dental caries in th ebest described by:


A. Descriptive and analytical study
B. Descriptive and experimental study
C. Descriptive, analytical and experimental study
D. Analytical and experimental study
Ans: A

-----------------------------------------------------------------------------------------------------------------------------

www.EduMedWeb.com www.EduMedWeb.com www.EduMedWeb.com


www.EduMedWeb.com www.EduMedWeb.com Page 1

AIIMS PG 2009
(Memory Based)
1. The ophthalmic division of fifth cranial nerve exits the skull through
A. Superior orbital fissure
B. Foraman ovale
C. Inferior orbital fissure
D. Forman spinosum
Ans: A

2. Which of the following is not true of occulomotor nerve


A. Enter orbit through the inferior orbital fissure
B. Parasympathetic ganglion placed in course of the occulomotor nerve
C. All the muscles of eye except superior oblique and lateral rectus are supplied by it
D. Causes constriction of pupil
Ans: A

3. A typical cervical vertebra is differentiated from thoracic vertebra by


A. shape of the body
B. by the presence of foramen transversarium
C. by the direction of facing of Superior articular facet
D. All of the above
Ans: B

4. Which of the following are least involved in the Proprioception of the muscles of head and
neck region
A. Facial nerve and trigeminal
B. Occulomotor and Trochlear
C. Spinal accessory nerve and vagus nerve
D. Facial nerve and glossphrayngeal
Ans: C

5. Which of the following adaptation is well suited to increase the work capacity at high altitude
A. Decreasing workload and increasing duration of exercise
B. Decreasing workload as well as duration of exercise
C. Increase workload as well as duration of exercise
D. Increase workload and decreasing duration of exercise
Ans: A

6. The muscle contraction is initiated by which of the following at synaptic junction


A. Release of acetyl choline at presynaptic channels
B. Opening of calcium channels at presynaptic channels
C. Opening of calcium channels at post synaptic channels
D. Release of acetyl choline at post synaptic junction
Ans: A

www.EduMedWeb.com www.EduMedWeb.com www.EduMedWeb.com


www.EduMedWeb.com www.EduMedWeb.com Page 2

7. Skeletal muscles
A. Contracts when calcium is taken up by sarcoplasmic reticulum
B. Contracts when actin and myosin filaments shorten
C. Contraction is initiated by calcium binding to troponin
D. Contraction is initiated by calcium binding to tropmyosin
Ans: A

8. Broca’s area is concerned with


A. Word formation
B. Comprehension
C. Repetition
D. Reading
Ans: A

9. All of the following statements are true regarding cardic muscle, except
A. Have multiple nuclei
B. Have gap junction
C. Have branching cell pattern
D. Functional syncitium
Ans: A

10. Blood brain barrier is absent in all of the following areas except
A. Subfornical region
B. Habenuclear trigone
C. Area posterma
D. Neurohypophysis
Ans: B

11. Which of the following is water soluble vitamin


A. Vitamin A and B
B. Vitamin B and C
C. Vitamin C and D
D. Vitamin D and E
Ans: B

12. Which of the following enzymes is acid labile (stable at acidic pH)-
A. Pepsin
B. Trypsin
C. Chymotrypsin
D. Carboxypeptidase
Ans: A

12. All of the following are true of HMP SHUNT except

www.EduMedWeb.com www.EduMedWeb.com www.EduMedWeb.com


www.EduMedWeb.com www.EduMedWeb.com Page 3

A. HMP shunt is an alternative pathway for oxidation of glucose that occurs in the cytosol
B. It is chracterised by the absence of production of ATP
C. It is active in adipose tisue, liver and gonads
D. The oxidative phase generates NADPH and the non oxidative phase generates pyruvate
Ans: D

13. Protein that precipitates on heating to 400C and redissolves on boiling is


A. Bence jones protein
B. Gamma globulin
C. Albumim
D. Myosin
Ans: A

14. Benzodiazepine antagonist?


A. Flumazenil
B. Furazolidone
C. Naloxone
D. Naltrexone
Ans: A

15. Drug not used in H. pylori:


A. Metronidazole
B. Omeprezole
C. Mosapride
D. Amoxicillin
Ans: C

16. Which is a prodrug?


A. Enalapril
B. Clonidine
C. Salmeterol
D. Acetazolamide
Ans: A

17. DOPA and 5-Hydroxytryptamine clinically significant because:


A. Both are acid-precursors of amine in the brain
B. both are neuromodulators
C. Are metabolites of amines
D. Both can cross the blood brain barrier easily
Ans: B

18. Which drug is not acetylated?


A. INH
B. Dapsone

www.EduMedWeb.com www.EduMedWeb.com www.EduMedWeb.com


www.EduMedWeb.com www.EduMedWeb.com Page 4

C. Hydralazine
D. Metoclopropamide
Ans: D

19. Which is not a second generation antihistaminic?


A. Loratidine
B. Acrivastatine
C. Cyclizine
D. Terfenidine
Ans: C

20. A child having hemophilia can take all of the following drugs except
A. Gamma amino butyric acid
B. Penicillin
C. Phenobarbitone
D. Aspirin
Ans: D

21. All of the following drug is effective in methicillin resistant staphylococcus aurus (MRSA)
infection except
A. Cotrimoxazole
B. Vancomycin
C. Ciprofloxacin
D. Cefaclor
Ans: D

22. Which of the following Antiepileptics can be given safely in pregnant woman
A. Phenytoin
B. Valproate
C. Carbamazepine
D. Phenobarbitone
Ans: D

23. A patient presents with pustules on hand and back. The bacteriological smear show
positive cocci on smear. Culture shows catalase negative and Beta hemolysis is produced.
Which of the following test best identifies the organism as group A streptococci.
A. Bacitracin sensitivity
B. Optochin sensitivity
C. Coagulation test.
D. Bile solubility
Ans: A

24. Aperson working in a slaugher house developed a small pustule on hand, which turned into
an ulcer with central black color. Which of the following can aid in diagnosis

www.EduMedWeb.com www.EduMedWeb.com www.EduMedWeb.com


www.EduMedWeb.com www.EduMedWeb.com Page 5

A. Carbol Fuschin
B. Methylene blue
C. Niel-janssen stain
D. Acid fast stain
Ans: B

25. An old diabetic patient present with multiple skin ulcers on the of the leg. Culture shows
gram posistive cocci in chains and produces clear hemolytic colonies. Which of the following
tests will best identify the organism
A. Catalase positive
B. Bacitracin
C. Optochin
D. Bile solubility
Ans: B

26. A n 60 years old debilitated male patient presented with fever, malaise, myalgia, anorexia,
and/or headache. He has chest pain and dry cough. Sputum examination showed growth on
charcoal yeast extract medium. The organism is likely
A. Streptococcus pneumoniae
B. Haemophilus influenzae
C. Staphylococci aureus
D. Legionella pneumophila
Ans: D

27. All of the are true about vibrio cholera except


A. It is non halophilic motile vibrio
B. Grows on simple standard media media
C. Man is the only natural host
D. Cannot survive in extracellur environment
Ans: D

28. Which of the following contains mostly obligate anaerobe species


A. Pseudomonas’
B. Staphylococcus
C. Streptococcus
D. Bacteriodes
Ans: D

29. Biological monitors for checking the efficacy of sterilization is checked by


A. Using spore formers
B. Using non pathogenic virus
C. Using yeasts
D. All of the above
Ans: A

www.EduMedWeb.com www.EduMedWeb.com www.EduMedWeb.com


www.EduMedWeb.com www.EduMedWeb.com Page 6

30. Earliest transient change following tissue injury is


A. Neutropenia.
B. Neutrophilia.
C. Monocytosis.
D. Lymphocytosis.
D. Lymphocytosis.
Ans: B

31. The general anesthetic with antiemetic property


A. diazepam
B. propofol
C. Promethazine
D. Fentanyl
Ans: B

32. Which of the following is not a contraindication of Centrineural anesthesia is


A. Patient on aspirin therapy
B. Reduced cadiac output
C. Patient on oral anticoagulant therapy
D. Raised intracranial pressure
Ans: A

33. Air abrasive technique the abrasive particles used are


A. 50 micrometer aluminium oxide.
B. 27 micrometer aluminium oxide.
C. 30 microns calcium carbonate
D. 50 micrometer iron oxide.

34. Which of the following cysts is involved with non-vital teeth?


A. Naso-alveolar cyst
B. Lateral periodontal cyst
C. Radicular cyst
D. Stafne’s bone cyst
Ans: C

35. Fordyce spots are


A. Fat tissues embedded in buccal mucosa
B. Red spots
C. Present on cheek mucosa lateral to the angle of mouth
D. All of the above
Ans: C

36. Fordyce’s granules are seen on lip & buccal mucosa. It consists of

www.EduMedWeb.com www.EduMedWeb.com www.EduMedWeb.com


www.EduMedWeb.com www.EduMedWeb.com Page 7

A. Ectopic Sebaceous glands


B. Ectopic Sweat glands
C. Mucous glands
D. Epithelium remnants
Ans: A

37. Pindborg tumor arise from


A. Cells rests of molassez
B. Cell rests of serrea
C. Dental Lamina
D. Stratum Intermedium
Ans: D

38. Which of the following is not a skin disease


A. Psoriasis
B. Erythema Multiforme
B. Erythema migrans
D. Pytriasis Rosea
Ans: C

39. A negative histopathology report of a highly suspicious oral lesion suggest?


A. No malignant potential of the lesion
B. Lesion should be stained with toludine blue like stains
C. Periodical recalls are necessary to assess the nature of the lesion
D. That another biopsy is necessary in view of the clinical impression
Ans: D
40. Which of the following is unlikely to cause enamel hypoplas
A. Rickets
B. Fluoride
C. Congenital syphilis
D. Cleidocranial dysostosis
Ans: D
41. Tuberculous lymphadenitis of submaxillary and cervical lymph nodes is seen in
A. Scrofula
B. Miliary TB
C. Lupus vulgaris
D. Lues
Ans: A
42. The peception of a taste with Complete loss of all taste stimuli
A. Ageusia
B. Hypogeusia
C. Dysgeusia
D. Torqugesia
Ans: D

www.EduMedWeb.com www.EduMedWeb.com www.EduMedWeb.com


www.EduMedWeb.com www.EduMedWeb.com Page 8

43. Advanced basal cell carcinoma frequently


A. metastasis by way of Arteries
B. spreads by Lymphatics
C. spreads by Nerve sheaths
D. Direct extension and invasion
Ans: D
44. Anitschkow cells are present in all of the following except
A. Apthous ulcer.
B. Sickle cell disease.
C. Iron deficiency anemia
D. Herpes simplex
Ans: D
45. While using the Radio visiography, the best method of infection control for receptors is
A. Autoclave the receptors after each use
B. Immerse the receptors in disinfectant
C. Wipe the sensor with 5.25% hypochlorite solution
D. Cover with impervious plastic sheath
46. Difference between Digital and conventional radiography is that in Digital radiography
A. X-rays are not used
B. Hard copy can not be obtained
C. No radiation hazard to the patient
D. There is different radiation receptor
Ans: D
47. Investigation of choice for a bony lesion in temporal bone is
A. Pluridirectional tomography
B. CT scan
C. MRI scan
D. Ultra Sono Graphy
Ans: B
48. The extra oral radiograph that best demonstrates the subcondylar portion
A. AP mandible
B. Water’s view
C. Reverse Towne’s view
D. Submental vertex
Ans: C
49. Best radiographic film to detect incipient caries is
A. speed E film
B. speed D film
C. speed F film
D. speed C film
Ans: C
50. Rdiographs of traumatised tooth is mainly necessary
A. To assess stage of root development
B. To rule out root fractures

www.EduMedWeb.com www.EduMedWeb.com www.EduMedWeb.com


www.EduMedWeb.com www.EduMedWeb.com Page 9

C. To have a base line comparison with future radiograph


D. To rule out hylanisation of pulp
Ans: C
51. Window like bone defect which do not involve marginal bone which are usually covered by
periosteum and gingival are called
A. Dehiscence
B. Crater
C. Fenestration
D. Trough
Ans: C
52. The best indication for bone fill procedures (osseous grafts for bone regeneration)
A. One walled infra bony pockets
B. One walled supra bony pockets
C. Two walled infra bony pockets
D. Three walled infra bony pockets
Ans: D
53. Which of the following is untrue of LJP
A. more common in females
B. mirror image type of bone loss is seen bilaterally
C. amount of bone destruction is proportional to the amount of plaque
D. aggressive periodontal bone destruction compared to normal periodontitis
Ans: C
54. Teeth least effected by periodontitis
A. Upper and lower first molar
B. Upper and lower incisors
C. Lower first molar and upper premolars
D. Upper canines and lower premolars
Ans: D
55. Moility of teeth is measured by using
A. Perodontometer
B. Periotron
C. Spechtometer
D. densitometer
Ans: A
56. The microorganism not associated with Perodontitis lesion is
A. Campylobacter rectus
B. Prevotella intermedia
C. Actinomyces viscosus
D. Actinobacillus actinomycetemcomitans
Ans: C
57. Which of the following cells are consistent with immunoglobulin levels in established
gingivitis
A. Plasma cells
B. Neutrophils

www.EduMedWeb.com www.EduMedWeb.com www.EduMedWeb.com


www.EduMedWeb.com www.EduMedWeb.com Page 10

C. Mast cells
D. Lymphocytes
Ans: A
58. Most periodontal damage occurs in the following conditions
A. intrusion
B. extrusion
C. infarction
D. Luxation
Ans: D
59. Specificity of a test is its ability to identify
A. True negative
B. False positive
C. False negative
D. True positive
Ans: A
60. Which of the folowing vaccine is not included in EPI (expanded programme of immunization)
schedule
A. OPV
B. BCG
C. MMR
D. DPT
Ans: C
61. All of the following are transmitted by lice except
A. Trench fever
B. Epidemic typhus
C. Relapsing fever
D. Q fever
Ans: D
62. Chronic carrier state is not a feature of which of the following diseases (another version of
the question is which of the following does’nt has heriditical (chronic carrier state??)occurance
A. Measles
B. Dipthria
C. Tuberculosis
D. Hepatitis B
Ans: A
63. Which of the following is a contains higheset amount of poly unsaturated fatty acids (PUFA)
A. Palm oil.
B. Soya bean oil.
C. Ground nut oil.
D. Olive oil.
Ans: B
64. In the lactobacillus count is 9500, the reading will be inferred as
A. Little or none
B. Slight

www.EduMedWeb.com www.EduMedWeb.com www.EduMedWeb.com


www.EduMedWeb.com www.EduMedWeb.com Page 11

C. Moderate
D. Marked
Ans: C
65. Which of the following is not present in carislov
A. leucine
B. glutamic acid
C. glycine
D. lysine
Ans: C
66. Ledermix paste contains
A. Minocycline
B. Oxytetracycline
C. Demeclocycline
D. Doxycycline
Ans: C
67. Chlorohexidine is least effective against
A. Candida
B. Streptococcus mutans
C. Enterobacter
D. E. fecalis
Ans: C
68. Which of the following lasers is a WATERLASE system of laser
A. YAG
B. Neon
C. YSGG
D. Argon
Ans: C
69. Which of the following teeth are most effected in a child of five years of age if put on
vigorous tetracycline therapy for one year duration
A. All primary teeth and first molars
B. Only permanent first molars
C. Permanent premolars and 2nd molars
D. Permanent incisors and canines
Ans: C
70. Which of the following is a space maintainer?
A. Pendulum appliance
B. Willets appliance
C. Twin block
D. Tip edge
Ans: B
71. Catalan’s appliance is used
A. space maintenance
B. correction of anterior crossbite
C. growth modulation of class II skeletal conditions

www.EduMedWeb.com www.EduMedWeb.com www.EduMedWeb.com


www.EduMedWeb.com www.EduMedWeb.com Page 12

D. all of the above


Ans: B
72. Serial extraction is contra indicated in all of the following except
A. impactions and anodontia
B. Openbite and spaced dentition
C. Class II and class III skeletal pattern
D. flaring and crowding of the lower teeth
Ans: D
73. The most common site of leak in CSF rhinorrhoea is into
A. Sphenoid sinus
B. Frontal sinus
C. Etmoidal sinus
D. maxillary sinus
Ans: C
74. Tertiary monoblock is used in
A. Orthograde MT A system
B. Acti V GP
C. Resilion monoblock obturation
D. Hydron
Ans: A
75. The recent modification of MTA used as a root canal irrigant is
A. MTA-A
B. MTA-B
C. MTA-C
D. MTA-D
Ans: D
76. Which of the following is a Tertiary monoblock system
A. Orthograde MT A system
B. Hydron
C. Resilion monoblock obturation
D. Acti V GP
Ans: D
77. Which of the following UDMA based sealer
A. Real seal
B. Endorez
C. AH 26
D. SEALAPEX
Ans: B
78. In fracture through mental foramen in mandible with less than 10mm of bone los treatment
would be,
A. Champy’s plate.
B. Lag screw
C. Non rigid fixation
D. Reconstruction plates

www.EduMedWeb.com www.EduMedWeb.com www.EduMedWeb.com


www.EduMedWeb.com www.EduMedWeb.com Page 13

Ans: D
79. Which of the following are most complicated fractures.
A. Symphysis
B. Body
C. Condyle
D. Angle
Ans: A
80. A transverse fracture of symphysis is treated by all of the following except
A. Two Compression plates. (2mm)
B. Two lag screws
C. Single Miniplate fixation (1.5mm)
4. 2.4 mm reconstruction plate.
Ans: C
81. Number of bones osteotised in Le – fort I osteotomy essentially are
A. 2
B. 3
C. 4
D. 5
Ans:
82. Which of the following chracterises Implant placement for overdentures in anterior maxilla
A. Two implants placed anterior to sinus in canine and premolar area on each side
B. Four implants placed in incisor region
C. Two implants placed in central incisor region
D. It is better to avoid implants anterior to sinus in overdenture construction
Ans: A
83. Which part of the central nervous system is mainly is infested by neuro cysticercosis.
A. cranial nerves
B. neurons
C. Brain parenchyma
D. Spinal cord
Ans: C
-----------------------------------------------------------------------------------------------------------------

www.EduMedWeb.com www.EduMedWeb.com www.EduMedWeb.com


www.EduMedWeb.com www.EduMedWeb.com Page 1

AIIMS PG MAY 2011


(Based on Memory)

1. A dead born foetus does not have:


1. Rigor mortis at birth.
2. Adipocere formation.
3. Maceration.
4. Mummification.
Ans: 2

2. False sense of perception without any external object or stimulus is known as:
1. Illusion.
2. Impulse.
3. Hallucination.
4. Phobia.
Ans: 3

3. Species identification is done by:


1. Neutron activation analysis (N.A.A.).
2. Precipitin test.
3. Benziine test.
4. Spectroscopy.
Ans: 2

4. In a suspected case of death due to poisoning where cadaveric rigidity is lasting longer than
usual, it may be a case of poisoning due to:
1. Lead
2. Arsenic.
3. Mercury.
4. Copper.
Ans: 2

5. ‘Whip-lash’ injuries is caused due to:


1. A fall from a height.
2. A cute hyperextension of the spine.
3. A blow on top to head.
4. A cute hyper flexion of the spine.
Ans:

6. All of the following form radiolucent stones except:


1. Xanthine.
2. Cysteine.
3. Allopurinol.
4. Orotic acid.
Ans: 2

7. A young female presents with history of dyspnoea on exertion. On examination, she has
wide, fixed split of S2 with ejection systolic murmur (III/VI) in left second intercostals space. Her
EKG shows left axis deviation. The most probable diagnosis is:

www.EduMedWeb.com www.EduMedWeb.com www.EduMedWeb.com


www.EduMedWeb.com www.EduMedWeb.com Page 2

1. Total anomalous pulmonary venous drainge.


2. Tricuspid atresia.
3. Ostium primum atrial septal defect.
4. Ventricular septal defect with pulmonary arterial hypertension.
Ans: 3

8. Which test is performed to detect reversible myocardial ischemia?


1. Coronary angiography.
2. MUGA scan.
3. Thallium scan.
4. Resting echocardiography.
Ans: 3

9. A 62 years old man with caracioma of lung presented to emergency department with
respiratory distress. His EKG showed electrical alternans. The most likely diagnosis is:
1. Pneumothorax.
2. Pleural effusion.
3. Cardiac tamponade.
4. Constrictive pericarditis.
Ans: 3

10. Atrial fibrillation may occur in all of the following conditions, except:
1. Mitral stenosis.
2. Hypothyroidism.
3. Dilated cardiomyopathy.
4. Mitral regurgitation.
Ans: 2

11. A patient with recent-onset primary generalized epilepsy develops drug reaction and skin
rash due to phenytoin sodium, The most appropriate to clonazepam.
1. Shift to clonazepam.
2. Restsrt phenytoin sodium after 2 weeks.
3. Shift to sodium valproate.
4. Shift to ethosuximide.
Ans: 3

12. Which of the following is the commonest location of hypertensive hemorrhage?


1. Pons.
2. Thalamus.
3. Putamen/external capsule.
4. Cerebellum.
Ans: 3

13. Which of the following is the most common central nervous sustem parastic infection?
1. Echinococcosis.
2. Sparganosis.
3. Paragonimiasis.
4. Neurocysticercosis.
Ans: 4

14. Which of the following is the most common tumor associated with type I neurofibromatosis?

www.EduMedWeb.com www.EduMedWeb.com www.EduMedWeb.com


www.EduMedWeb.com www.EduMedWeb.com Page 3

1. Optic nerve glioma.


2. Meningioma.
3. Acoustic Schwannoma.
4. Low grade astrocytoma.
Ans: 1

15. A patient undergoing surgery suddenly develops hypotension. The monitor shows that the
end tidal carbon dioxide has decreased abruptly by 15mmHg. What is the probable diagnosis?
1. Hypothermia.
2. Pulmonary embolism
3. Massive fluid deficit
4. Myocardial depression due to anesthetic agents.
Ans: 2

16. The commonest cause of death in a patient with primary amyloidosis is


1. Renal failure
2. Cardiac involvement
3. Bleeding diathesis
4. Respiratory failure
Ans: 1

17. A middle aged old man, with chronic renal failure is diagnosed to have sputum positive
pulmonary tuberculosis. His creatinine clearance is 25ml/min. All of the following drugs need
modification in doses except:
1. Isoniazid
2. Streptomycin
3. Rifampicin
4. Ethambutol.
Ans: 3

18. An HIV-positive patient is on anti retroviral therapy with zidovudine, lamivudine and
indinavir. He is proven to be suffering from genitor-urinary tuberculosis. Which one of the
following drugs not is given to this patient?
1. Isoniazid
2. Rifampicin
3. Pyrazinamide
4. Ethambutol
Ans: 2

19. A high amylase level in pleural fluid suggests a diagnosis of:


1. Tuberculosis
2. Malignancy
3. Rheumatoid arthritis
4. Pulmonary infarction
Ans: 2

20. Which of the following conditions is associated with Coomb’s hemolytic anaemia:
1. Thrombotic thrombocytopenic purpura.
2. Progressive systemic sclerosis.
3. Systemic lupus erythematosus.
4. Polvarteritis nodosa.

www.EduMedWeb.com www.EduMedWeb.com www.EduMedWeb.com


www.EduMedWeb.com www.EduMedWeb.com Page 4

Ans: 3

21. Which of the following marker in the blood is the most reliable indicator of recent hepatitis B-
infection?
1. HBsAg
2. lgG anti – HBs
3. lgM anti – HBc
4. lgM anti – Hbe
Ans: 3

22. The severity of mitral stenosis is clinically best decided by:


1. Length of diastolic murmur.
2. Intensity of diastolic murmur.
3. Loudness of first heart sound.
4. Split of second heart sound.
Ans: 1

23. The risk of developing infective endocarditis is the least in a patient with:
1. Small ventricular septal defect
2. Severe aortic regurgitation.
3. Severe mitral regurgitation
4. Large atrial septal defect
Ans: 4

24. The severity of mitral regurgitation is decided by all of the following clinical findings except:
1. Presence of mid-diastolic murmur across mitral valve.
2. Wide split of second heart sound.
3. Presence of left ventricular S3 gallop.
4. Intensity of systolic murmur across mitral valve.
Ans: 1

25. Congenital long QT syndrome can lead to:


1. Complete heart block
2. Polymorphic ventricular tachycardia.
3. Acute myocardial infarction.
4. Recurrent supra ventricular tachycardia.
Ans: 2

26. With reference to bacteroides fragilis all of the following statements are true except:
1. It is the most frequent anaerobe isolated from clinical samples.
2. It is not uniformly sensitive to metronidazole.
3. The lipopolysaccharide formed by B. fragills is structurally and functionally different from the
conventional endotoxin.
4. Shock and disseminated intravascular coagulation are common in bacteremia due to B.
Ans: 4

27. All of following statements are true regarding Q fever except:


1. It is a zoonotic infection.
2. Human disease is characterized by an interstitial pneumonia
3. No rash is seen
4. Weil Felix reaction is very useful for diagnosis.

www.EduMedWeb.com www.EduMedWeb.com www.EduMedWeb.com


www.EduMedWeb.com www.EduMedWeb.com Page 5

Ans: A

28. The following ststements are true regarding botulism except:


1. Infant botulism is caused by ingestion of preformed toxin.
2. Clostridium botulinum A,B,C and F cause human disease.
3. The gene for botulinum toxin is encoded by a bacteriophage.
4. Clostridium boriatti may cause botulism.
Ans: 1

29. Streptococcal Toxix shock syndrome is due to the following virulence factor.
1. M protein
2. Pyrogenic exotoxin
3. Streptolysin O.
4. Carbohydrate cell wall.
Ans: 2

30. A 24 year old male presents to a STD clinic with a single painless ulcer on external genitalia.
The choice of laboratory test to look for the etilogical agent would be:
1. Scrappings from ulcer for culture on chocolate agar with antibiotic supplement.
2. Serology for detection of specific lgM antibodies.
3. Scrappings from ulcer for dark field microscopy.
4. Scrappings from ulcer for tissue culture.
Ans: 3

31. There has been an outbreak of food borne salmonella gastroenteritis in the community and
the stool samples have been received in the laboratory. Which is the enrichment medium of
choice:
1. Cary Blair medium
2. V R medium
3. Selenite “F” medium
4. Thioglycollate medium
Ans: 3

32. A 20 year old male had pain abdomen and mild fever followed by gastroenteritis. The stool
examination showed presence of pus cells and RBCs on microscopy. The etiological agent
responsible is most likely to be:
1. Enteroinvasive E. coli.
2. Enterotoxigenic E. coli.
3. Enteropathiogenic E. coli.
4. Enetroaggregative E. coli.
Ans: 1

33. A man presents with fever and chills 2 weeks after a louse bite. There was a maculo-popular
rash on the trunk which spread peripherally. The cause of this infection can be:
1. Scurb typhus.
2. Endemic typhus.
3. Rickettsial pox.
4. Epidemic typhus.
Ans: 2

34. The virulence factor of Nesseria gonorrhoeae includes all of the following except:

www.EduMedWeb.com www.EduMedWeb.com www.EduMedWeb.com


www.EduMedWeb.com www.EduMedWeb.com Page 6

1. Outer membrane proteins


2. IgA Protease
3. M-proteins
4. Pilli
Ans: 3

35. A patient in an ICU is on a CVP line. His blood culture shows growth of grain positive cocci
which are catalase positive and coagulase negative. Themost likely etiological agent is:
1. Staplylococcus aureus
2. Staplylococcus epidermidis
3. Staplylococcus pyogenes
4. Enterococcus faecalis.
Ans: 2

36. According to WHO criteria, the minimum normal sperm cauntis:


1. 10 million/ml
2. 20 million/ml
3. 40 million/ml
4. 60 million/ml
Ans: 2

37. In triple screening test for Down’s syndrome during pregnancy all of the following are
included except:
1. Serum beta Hcg
2. Serum oestriol
3. Maternal serum Alfa fetoprotein
4. Acetyl cholinesterase
Ans: 4

38. An optic nerve injury may result in all of the following except:
1. Loss of vision in that eye.
2. Dilatation of pupil.
3. Ptosis
4. Loss of light reflex.
Ans: 3

39. The commonest cause of occipito-posterior position of fetal head during labour is:
1. Maternal obesity
2. Deflexion of fetal head
3. Multiparity
4. Android pelvis.
Ans: 2

40. The commonest cause of breech presentation is


1. Prematurity
2. Hydrocephalus
3. Placenta previa
4. Polyhydramnios
Ans: 1

41. The commonest congenital anomaly seen in pregnancy with diabetes mellitus is

www.EduMedWeb.com www.EduMedWeb.com www.EduMedWeb.com


www.EduMedWeb.com www.EduMedWeb.com Page 7

1. Multicystic kidneys
2. Oesophageal atresia
3. Neural tube defect
4. Enalapril
Ans: 3

42. Use of which of the following drug is contra-indicated in pregnancy.


1. Digoxin
2. Nigedipine
3. Amoxycillin
4. Ealapril
Ans: 4

43. Following renal disorder is associated with worst pregnancy outcome.


1. Systemic lupus erythematosus
2. IgA nephropathy
3. Autosomal dominant polycystic kidney disease
4. Scleroderma
Ans: 1

44. A perimanopausal lady with well differentiated adenocarcinoma of uterus has more than half
myometrial invasion, vaginal metastasis and inguinal lymph node metastasis. She is staged as:
1. Stage III B
2. Stage III C
3. Stage IV a
4. Stage IV b
Ans: 4

45. The following combination of agents are the most preferred for short day care surgerias
1. Propofol, fetanyl, isoflurane
2. Thiopentone sodium, morphine, halothane
3. Ketamine, pethidine, halothane
4. Propofol, morphine, halothane
Ans: 1

46. All of the following drugs have been used for medical abortion, except:
1. Mifepristone.
2. Misoprostol
3. Methotrexate
4. Atosiban
Ans: 4

47. A 21 year old primigravida is admitted at 39 weeks gestation with painless antepartum
haemorrhage. On examination uterus is soft, non tender and head engaged. The management
for her would be:
1. Blood transfusion and sedatives.
2. A speculum examination
3. Pelvic examination in OT
4. Tocolysis and sedatives.
Ans: 3

www.EduMedWeb.com www.EduMedWeb.com www.EduMedWeb.com


www.EduMedWeb.com www.EduMedWeb.com Page 8

48. Which statement is true regarding VENTOUSE (Vacuum Extractor)


1. Minor scalp abrasions and subgaleal heamatomas to new born are moe frequent than
forceps.
2. Can be applied when foetal head is above the level of ischial spine.
2. Maternal trauma is more frequent than forceps.
3. Can not be used when fetal head is not fully rotated.
Ans: 2

49. A drop in fetal heart rate that typically lasts less than 2 minutes and is usually associated
with umbilical cord compresion is called.
1. Early declaration
2. Late declaration
3. Variable declaration
4. Prolonged declaration
Ans: 3

50. All the following are known side effects with the use of tocolytic therapy except.
1. Tachycardia
2. Hypotension
3. Hyperglycemia
4. Fever
Ans: 2

51. All of the following factors decrease the minimum Alveolar Concentration (MAC) of an
inhalation anaesthetic agent except.
1. Hypothermia
2. Hyponatremia
3. Hypocalcaemia
4. Anemia
Ans: 3

52. The narrowest part of larynx in infants is at the cricoid level. In administering anesthesia this
may lead to all except:
1. Choosing a smaller size endotracheal tube.
2. Trauma t the subglottic region.
3. Post operative stridor
4. Laryngeal oedema
Ans: ?

53. The administration of succinylocholine to a paraplegic pateint led to the appearance of


dysarrhythmias, conduction abnormalities and finally cardiac arrest. The most likely cause is:
1. Hypercalcemia
2. Hyperkalemia
3. Anaphylaxis
4. Hyermagnesemia
Ans: 1

54. Following spinal subarachnoid block patient a develop hypotension. This can be managed
by the following means exept:
1. Lowering the head end
2. Administration of 100 ml of Ringers lacate before the block

www.EduMedWeb.com www.EduMedWeb.com www.EduMedWeb.com


www.EduMedWeb.com www.EduMedWeb.com Page 9

3. Vascopressure drug like methoxamine


4. Us of ionotrope like dopamine.
Ans: 4

55. In the immediate post operative period the common cause of respiratory insufficiency could
be because of the following, except:
1. Residual effect of muscle relaxant
2. Overdose of narcotic analgesic
3. Mild Hypovalemia
4. Mycocardial infarction
Ans: 4

56. On mutation, which of the following may give rise to hereditary glaucoma.
1. Optineurin
2. Ephrins
3. RBA8
4. Huntingtin
Ans: 1

57. Brain lipid binding protein is expressed by which of the following:


1. Mature astrocytes.
2. Oligodendrocytes.
3. Purkinje cells
4. Pyramidal neurons
Ans: 2?

58. All of the following ligaments contribute to the stability of ankle (talocrural) joint except.
1. Calcaneonavicular (spring)
2. Deltoid
3. Lateral
4. Posterior tibiofibular
Ans: 1 (associated with arch integrity)

59. In angina pectoris, the pain radiating down the left arm )sensory) fibres contained in the :-
1. Carotid branch of the glossopharyngeal nerve
2. Phrenic nerve
3. Vagus nerve and recurrent laryngeal nerve.
4. Thoracic splanchnic nerve
Ans: 4

60. All the following sings could result from infection within the right cavernous sinus except.
1. Constricted pupil in response to light
2. Engorgement of the retinal veins upon ophthalmoscopic examination.
3. Ptosis of the right eyelid.
4. Right opthalmoplegia.
Ans: 1

61. In dislocation of the jaw, displacement of the articular disc beyond the articular tubercle of
the temporomandibular joint results from spasm or excessive contraction of the following
muscle?
1. Buccinator

www.EduMedWeb.com www.EduMedWeb.com www.EduMedWeb.com


www.EduMedWeb.com www.EduMedWeb.com Page 10

2. Lateral pterygoid
3. Masseter
4. Temporalis
Ans: 2

62. Following surgical removal of a firm nodular cancer swelling in the right breast and
exploration of the right axilla, on examination the patient was found to have a winged right
scapula. Most likely this could have occurred due to injury to the:
1. Subscapular muscle
2. Coracoid process of scapula
3. Long thoracic nerve
4. Circumflex scapular artery
Ans: 3

63. A 50 year old man suffering from carcinoma of prostate showed areas of sclerosis and
collapse of
T10 and T11 vertebrae in X-ray. The spread of this cancer to the above vertebrae was most
probably through:
1. Sacral canel
2. Lymphatic vessels
3. Internal vertebral plexus of venis
4. Superiorectal veins
Ans: 3

64. Father to son inheritance is never seen in case of:


1. Autosomal dominant inheritance
2. Autosomal recessive inheritance
3. X- linked recessive inheritance
4. Multifactorial inheritance
Ans: 3

65. A 3-year old boy is detected to have bilateral renal calculi. Metabolic evaluation confirms the
presence of marked hypercalciuria with normal blood levels of calcium,magnesium,
phosphate,uric acid and creatinine. A diagnosis of idiopathic hypercalciuria is made. The dietary
management includes all, except
1. Increased water intake
2. Low sodium diet
3. Reduced calcium intake
4. Avoid meat proteins.
Ans: 3

66. The hormone associated with cold adaption:


1. Growth hormone.
2. Thyroxine.
3. Insulin.
4. Melanocyle Stimulating Hormone.
Ans: 2

67. All of the following are characteristic features of Kwashiorkor, except:


1. High blood osmolarity.
2. Hypoalbuminemia.

www.EduMedWeb.com www.EduMedWeb.com www.EduMedWeb.com


www.EduMedWeb.com www.EduMedWeb.com Page 11

3. Edema.
4. Fatty liver.
Ans: 1

68. Actyl Co-A acts as a substrate for all the enzymes except:
1. HMG- Co-A synthetase.
2. Malic enzyme.
3. Malonyl Co-A synthetase.
4. Fatty acid synthetase.
Ans: 2

69. The activity of the following enzyme is affected by biotin deficiency:


1. Transketolase,
2. Dehydrogenase.
3. Oxidase.
4. Carboxylase.
Ans: 4

70. A 55-year-old male accident victim in casualty urgently needs blood. The blood bank is
unable to determine his ABO group, as his red cell group and plasma group do not match.
Emergency transfusion of patient should be with:
1. RBC corresponding to his red cell group and colloids/crystalloid.
2. Whole blood corresponding to his plasma group.
3. O positive RBC and colloids/crystalloid.
4. AB negative whole blood.
Ans: 3

71. Although more than 400 blood groups have been identified, the ABO blood group system
remains the most important in clinical medicine because:
1. It was the blood group system to be discovered.
2. It has four different blood groups A, B, AB, O(H).
3. ABO (H) antigens are present in most body tissues and fluids.
4. ABO (H) antibodies are invariably present in plasma when persons RBC lacks the
corresponding antigen.
Ans: 4

72. Km of an enzyme is:


1. Dissociation constant.
2. The normal physiological substrate concentration.
3. The substrate concentration at half maximum velocity.
4. Numerically indentical for all isozymes that catalyze a given reaction.
Ans: 1

73. At the physiological PH the DNA molecules are:


1. Positively charged.
2. Negatively charged.
3. Neutral.
4. Amphipathic.
An: 2

74. Cholesterol present in LDL:

www.EduMedWeb.com www.EduMedWeb.com www.EduMedWeb.com


www.EduMedWeb.com www.EduMedWeb.com Page 12

1. Represents primarily cholesterol that is being removed from peripheral cells.


2. Binds to a receptor and diffuses across the cell membrance.
3. On acuumulation in the cell inhibits replenishment of LDL receptors.
4. When enters a cell, suppresses activity of acyl-CoA: cholesterol acytranferase ACAT.
Ans: 3

75. A newborn infant refuses breast milk since the 2nd day of birth, vomits on force-feeding but
accepts glucose-water, develops diarrhea on the third day, by 5th day she is jaundiced with liver
enlargement and eyes show signs of cataract. Urinary reducing sugar was positive but blood
glucose estimated by glucose oxidation method was found low. The most likely cause is
deficiency of:
1. Galactose-1-phosphate uridyl transferase.
2. Beta galactosidase.
3. Glucose-6- phosphatase.
4. Galactokinase.
Ans: 4

76. An obese lady aged 45 years, was brought to emergency in a semi comatose condition. The
laboratory investigations showed K+ (5.8mmol/L); Na+ (136mmol/L); blood PH (7.1), HCO3
(12mmol/L); ketone bodies (350 mg/dl). The expected level of blood glucose for this lady is:
1. < 45 mg/dl.
2. < 120 mg/dl.
3. > 180 mg/dl.
4. < 75 mg/dl.
Ans: 2?/3?

77. Replication and transcription are similar processes in mechanistic terms because both:
1. Use RNA primers for initiation.
2. Use decoxybonucleotides as precursors.
3. Are semi conserved events
4. phosphodiester bond formation with elongation occurring in the 5’ – 3’ direction.
Ans: 2

78. Commonest cause of neonatal mortality in India is:


1. Diarrheal diseases.
2. Birth injuries.
3. Lowbirth weight
4. Congenital anomalies.
Ans: 3

79. All of the following features are suggestive of asbestosis except:


1. Occurs within five years of exposure.
2. The disease progresses even after removal of contact.
3. Can lead to pleural mesothelioma.
4. Sputum contains asbestos bodies.
Ans: 1

80. In calculating Dependency Ratio, the numerator is expressed as:


1. Population under 10 years and 60 and above.
2. Population under 15 years and 60 and above.
3. Population under 10 years and 65 and above.

www.EduMedWeb.com www.EduMedWeb.com www.EduMedWeb.com


www.EduMedWeb.com www.EduMedWeb.com Page 13

4. Population under 15 years and 65 and above.


Ans: 4

81. A adult male patient presented in the OPD with complaints of cough and fever for 3 months
and heamoptysis off and on. His sputum was positive for AFB. On probing it was found that he
had already received treatment with RHZE for 3 weeks from a nearly hospital and discontinued.
How will you categorize and manage the patient?
1. Category III, start 2 (RHZ)3.
2. Category II, start 2 (RHZE)3.
3. Category I, start 2 (RHZE)3.
4. Category II, start 2 (RHZES)3.
Ans: 4

82. A screening test is used in the same way in two similar populations; but the proportion of
false positive results among those who test positive in population A is lower than those who test
positive in population B. What is the likely explanation?
1. The specificity of the test is lower in population A.
2. The prevalence of the disease is lower in population A.
3. The prevalence of the disease is higher in population A.
4. The specificity of the test is higher in population A.
Ans: 3

83. Residence of three village with three different types of water supply were asked to
participate in a study to identify cholera carries. Because several cholera deaths had occurred in
the recent past, virtually everyone present at the time submitted to examination. The proportion
of residents in each village who were carries was computed and compared. This study is a:
1. Cross – sectional study.
2. Case – control study.
3. Concurrent cohort study.
4. Non – concurrent.
Ans: 1

84. A drug company is developing a new pregnancy-test kit for use n an outpatient basis. The
company pregnancy-test on 100 women who are known to be pregnant. Out of 100 women, 99
showed positive test. Upon using the same test on 100 non-pregnant women, 90 showed
negative result. What is the sensitivity of the test?
1. 90%
2. 99%
3. Average of 90 & 99.
4. Cannot be calculated from the given data.
Ans: 2

85. An investigator wants to study the association between maternal intake of iron supplements
(Yes/No) and birth weights (in gms) of newborn babies. He collects relevant data from 100
pregnant woman and their newborns. What statistical test of hypothesis would you adise for the
investigator in this situation?
1. Chi-Square test.
2. Unpaired or independent t-test.
3. Analysis of Variance.
4. Paired t-test.
Ans: 4

www.EduMedWeb.com www.EduMedWeb.com www.EduMedWeb.com


www.EduMedWeb.com www.EduMedWeb.com Page 14

86. Pin index system is a safety feature adopted in anaesthesia. Machines to prevent:
1. Incorrect attachment of anaesthesia machines.
2. Incorrect attachment of anaesthesia face masks.
3. Incorrect inhalation agent delivery.
4. Incorrect gas cylinder attachment.
Ans: 4

87. A 9-years old boy has steroid dependent nephrotic syndrome for the last 5 years. He has
received corticosteroids almost continuosly during this period and has cushingoid features. The
blood preassure is 120/86 mmHg and there are bilaterel subcapsular cataracts. The treatment
of choice is:
1. Levamisole.
2. Cyclophosphamide.
3. Cyclosporin A.
4. Intravenous pulse corticosteroids.
Ans: 2

88. After a minor head injury a young patient was unable to close his left eye and had drooling
of saliva from left angle of mouth. He is suffering from:
1. VIIth nerve injury.
2. Vth nerve injury.
3. IIIrd nerve injury.
4. Combined VIIth and IIIrd nerve injury.
Ans: 1

89. Which one of the following does not produce cyanosis in the first year of life:
1. Atrial septal defect.
2. Hypoplastic left heart syndrome.
3. Truncus arteriosus.
4. Double outlet right ventricle.
Ans: 1

90. All of the following are given global prominence in the VISION 2020 goals, except:
1. Refractive errors.
2. Cataract.
3. Trachoma.
4. Glaucoma.
Ans: 1

91. For the field diagnosis of trachoma, the WHO recommends that follocular and intense
trachomainflammation should be assessed in:
1. Women aged 15 – 45 years.
2. Population of 10 to 28 year range.
3. Children aged 0 – 10 years.
4. Population above 25 years of age irrespective of sex.
Ans: ?

92. The eye condition for which the world Bank assistsnce was provided to the National
Programme for Control of Blindness (1994-2001) is:
1. Cataract.

www.EduMedWeb.com www.EduMedWeb.com www.EduMedWeb.com


www.EduMedWeb.com www.EduMedWeb.com Page 15

2. Refractive errors.
3. Trachoma.
4. Vitamin A deficiency.
Ans: 4

93. Under the school eye screening programme in India, the initial vision screening of school
children is done by:
1. School teachers.
2. Primary level health workers.
3. Eye specialists.
4. Medical officers.
Ans: 1

94. The usefulness of a screening test depends upon its:


1. Seensitivity.
2. Specificity.
3. Reliability.
4. Predictive value.
Ans: 1

95. For testing the statistical significance of the difference in heights of school children among
three socio-economic groups, the most appropriate statistical test is:
1. Student’s ‘t’ test.
2. Chi-aquared test.
3. Paired ‘t’ test.
4. One way analysis of variance (one way ANOVA).
Ans: 4

96. Reservoir of Indian Kala azar is:


1. Man.
2. Rodent.
3. Canine.
4. Equine.
Ans: 1

97. The following is true about the term ‘New Families’:


1. It is a variant of the 3-generation family.
2. It is applied to all nuclear families of less than 10 years duration.
3. It is a variant of the joint family.
4. It is applied to all nuclear families of less than 2 years duration.
Ans: 2

98. A 24 year old female has flaccid bullae in the skin and oral erosions. Histopathology shows
intraepidermal acantholytic blister. The most likely diagnosis is:
1. Pemphigoid.
2. Erythema multiforme.
3. Pemphigus vulgaris.
4. Dermatitis herpetiformis.
Ans: 3

99. Podophyllum resin is indicated in the treatment of:

www.EduMedWeb.com www.EduMedWeb.com www.EduMedWeb.com


www.EduMedWeb.com www.EduMedWeb.com Page 16

1. Psoriasis.
2. Pemphigus.
3. Condyloma acuminata.
4. Condyloma lata.
Ans: 3

100. The following drug is indicated in the treatment of pityriasis versicolar:


1. Ketoconazole.
2. Metronidazole.
3. Griseofulvin.
4. Chloroquine.
Ans: 3

101. Direct impact on the bone will produce a:


1. Transverse fracture.
2. Oblique fracture.
3. Spiral fracture.
4. Comminuted fracture.
Ans: 4

102. All of the following are seen in rickets, except.


1. Bow legs.
2. Gunstock deformity.
3. Pot belly.
4. Cranio tabes.
Ans: 2

103. Post-dural puncture headache is typically:


1. A result of leakage of blood into the epidural space.
2. Worse when lying down than in sitting position.
3. Bifrontal or occipital.
4. Seen within 4 hours of dural puncture.
Ans: 3

104. Kenny Packs were used in the treatment of:


1. Poliomyelitis.
2. Muscular dystrophy.
3. Polyneuropathies.
4. Nerve Injury.
Ans: 1

105. A pateint was administered epidural anaesthesia with 15ml 1.5% lignocaine with
adrenaline for hernia surgery., He developed hypotension and respiratory depression within 3
minutes after administration of block. The commonest cause would be.
1. Allergy to drug administered.
2. Systemic toxicity to drug administered.
3. Patient got vasovagal shock.
4. Drug has entered the sub arachnoid space.
Ans: 3

106. Which one of the following is the shortest acting intravenous analgesic:

www.EduMedWeb.com www.EduMedWeb.com www.EduMedWeb.com


www.EduMedWeb.com www.EduMedWeb.com Page 17

1. Remifantanil.
2. Fentanyl.
3. Alfentanil.
4. Sufentanil.
Ans: 1

107. The etiology of anterior ethmoidal neuralgia is:


1. Inferior turbinate pressing on the nasal septum.
2. Middle turbinate pressing on the nasal septum.
3. Superior turbinate pressing on the nasal septum.
4. Causing obstruction of sphenoid opening.
Ans: 2

108. The treatment of choice for stage 1 cancer larynx is:


1. Radical Surgery.
2. Chemotherapy.
3. Radiotherapy.
4. Surgery followed by radiotherapy.
Ans: 3

109. Stapes footplate covers:


1. Round window.
2. Oval window.
3. Interior sinus tympani.
4. Pyramid.
Ans: 2

110. All the following muscles are innervated by the facial nerve except.
1. Occipito-frontalis.
2. Anterior belly of diagastric.
3. Risorius.
4. Procerus.
Ans: 2

111. The following statements regarding Turner syndrome are except.


1. Occurrence of Turner syndrome is influenced by material age.
2. Most patients have primary amenorrhoea.
3. Most patients have short stature.
4. Edema of hands and feet is an important feature durring infancy.
Ans: 1

112. All of the following methods are used for the diagnosis of HIV infection in a 2 month old
child except:
1. DNA-PCR.
2. Viral culture.
3. HIV ELISA
4. p24 antigen assay.
Ans: 3

113. In neonatal screening programme for detection of congenital hypothyroidism, the ideal
place and time to collect the blood sample for TSH estimation is:

www.EduMedWeb.com www.EduMedWeb.com www.EduMedWeb.com


www.EduMedWeb.com www.EduMedWeb.com Page 18

1. Cord blood at time of birth.


2. Heal pad blood at time of birth.
3. . Heal pad blood on 4 day of birth.
4. Peripheral venous blood on 28 day.
Ans: 1

114. The Pin index code of Nitrous oxide is:


1. 2,5.
2. 1,5.
3. 3,5.
4. 2,6.
Ans: 1

115. A three years old presents with poor urinary stream. Most likely cause is:
1. Stricture urethra.
2. Neurogenic bladder.
3. Urethral calculus.
4. Posterior uretral valves.
Ans: 4

116. Peroconceptional use of the following agent leads to reduced incidence of neural tube
defects:
1. Folic acid.
2. Iron.
3. Calcium.
4. Vitamin A.
Ans: 1

117. A 11-month old boy, weighing 3 kg, has polyuria, polydipsia and delayed motor milestones.
Blood investigations show creatinine of 0.5 mg/dl, potassium 3 mEq/1, sodium 125 mEq/1,
chloride 88 mEq/1, calcium 8.8 mg/dl, PH 7.46 and brcarbonate 26 mEq/1. Ultrasonography
shows medullary nephrocalcinosis. The most likely diagnosis is:
1. Renal tubular acidosis.
2. Diabetes insipidus.
3. Bartters syndrome.
4. Pseudohypoaldosteronism.
Ans: 3

118. The treatment of choice for primary grade V vesico-ureteric reflux involving both kidneys in
a 6 month old boy is:
1. Antibiotic prophylaxis.
2. Ureteric reimplantation.
3. Cystoscopy followed by subureteric injection of teflon.
4. Bilateral ureterostomies.
Ans: 1?

119. The most common leukocytoclastic vasculitis affecting children is:


1.Takayasu disease.
2. Mucocutaneous lymphnode syndrome.
(Kawasaki disease)
3.Henoch Schonlein purpura.

www.EduMedWeb.com www.EduMedWeb.com www.EduMedWeb.com


www.EduMedWeb.com www.EduMedWeb.com Page 19

4. Polyarteritis nodosa.
Ans: 3

120. A-13-year old boy is referred for evaluation of nocturnal enuresis and short stature. His
blood pressure is normal. The hemoglobin level is 8g/dl. Urea 112 mg/dl, creatinine 6 mg/dl,
sodium 119 mEq/dl, potassium 4 mEq/l, calcium 7 mg/dl, phophate 6 mg/dl and alkaline
phophatase 300 U/l. Urinalysis shows trace proteinuria with hyaline casts; nored and white cells
are seen. Ultrasound shows bilateral small kidneys and the micturating cystourethrogram is
normal. The most likely diagnosis is:
1. Alport’s syndrome.
2. Medullary sponge kidney.
3. Chronic glomerulonephritis.
4. Nephronophthisis.
Ans: 4

121. The most common infectious agent associated with chronic pyelonephritis is:
1. Proteus vulgaris.
2. Klebsiella pneumonie.
3. Staphylococcus aureus.
4. Escherichia coli.
Ans: 1

122. Which of the following hepatitis viruses is a DNA virus:


1. Hepatitis C virus.
2. Hepatitis B virus.
3. Delta agent.
4. Hepatitis E virus.
Ans: 2

123. The mechanism that protects normal pancreas from autodigestion is:
1. Secretion of biocarbonate.
2. Protease inhibitors present in plasma.
3. Proteolytic enzymes secreted in inactive form.
4. The resistance of pancreatic cells.
Ans: 3

124. All of the following type of lymphoma are commonly seen in the orbit except.
1. Non Hodgkin’s lymphoma, mixed lymphocytic & histiocytic.
2. Non Hodgkin’s lymphoma, lymphocytic poorly differentiated.
3. Burkitt' lymphoma.
4. Hodgkin’s lymphoma.
Ans: 3

125. “Tophus” is the pathognomonic lesion of which of the following condition:


1. Multiple myeloma.
2. Cystinosis.
3. Gout.
4. Eale’s disease.
Ans: 3

126. Common ocular manifeststion in Trisomy 13 is:

www.EduMedWeb.com www.EduMedWeb.com www.EduMedWeb.com


www.EduMedWeb.com www.EduMedWeb.com Page 20

1. Capillary hemangioma.
2. Bilateral microphthalmos.
3. Neurofibroma.
4. Dermoid Cyst.
Ans: 1

127. Heamorrhagic pericacarditis occurs in all of the following condition except:


1. Transmural myocardial infarction.
2. Dissecting aneurysm of aorta.
3. Metastatic disease of pericardium.
4. Constrictive pericarditis.
Ans: 2

128. A 60-years man presented with fatigue weight loss and heaviness in left hypochodrium for
6 months. The hemogram showed Hb 10 gm/dl, TLC 5 laks/mm3, platelet count 4 laks/mm3,
DLC; neutrophil 55%, lymphocytes 4% montocytes 2% basophiles 6% metamyelocytes 10%
myeloytes 18%, promyelocytes 2% and blast 3%. The most likely cytogenetic abnormality in this
case is:
1. t (8; 21).
2. t (9; 22).
3. t (15; 17).
4. trisomy 21.
Ans: 2

129. Onsectioning of an organ at the time of autopsy, a focal, wedge-shaped firm area is seen
accompanied by extensive hemorrhage, with a red appearance. The lesion has a base on the
surface of the organ. This finding is typically of:
1. Lung with pulmonary thromboembolism.
2. Heart with coronary thrombosis.
3. Liver with hypovolemic shock.
4. Kidney with septic embolus.
Ans: 1

130. Upper GI endoscopy and biopsy from lower esophagus in a 48 year old lady with chronic
heart burn shows presence of columner epithelium with goblet. The feature is most likely
consistent with:
1. Dysplasia.
2. Hyperplasia.
3. Carcinoma in-situ.
4. Metaplasia.
Ans: 4

131. The light brown perinuclear pigment seen on F & F staining of the cardiac muscle fibers in
the grossly normal appearing heart of an 83-year oldman at autopsy is due to deposition of:
1. Hemosiderin.
2. Lipochrome.
3. Cholesterol matabolic.
4. Anthracotic pigment.
Ans: 2

www.EduMedWeb.com www.EduMedWeb.com www.EduMedWeb.com


www.EduMedWeb.com www.EduMedWeb.com Page 21

132. A renal biopsy from a 56 years old women with progressive renal failure for the past 3
years shows glomerular and vascular deposition of pink amorphous material. It shows apple-
green birefringence under polarized light after Congo red staining. These deposits are positive
for lambda light chains. The person is most likely to suffer from.
1. Rheumatoid arthritis.
2. Tuberculosis.
3. Systemic lupus erythematosus.
4. Multiple myeloma.
Ans: 4

133. A-40-year-old man has a chronic cough with fever for several months. The chest
radiograph reveals a diffuse reticulonodular pattern. Microscopically on transbronchial biopsy
there are focal areas of inflammation containing epithelioid cell granuloma, Langhans giant
cells, and lymphocytes. These findings are typical for which of the following type of
hypersentivity immunologic responses:
1. Type-I
2. Type-II
3. Type-III
4. Type-IV
Ans: 4

134. An adult old man gets a burn injury to his hands. Over few weeks, the burned skin heals
without the need for skin grafting. The most critical factor responsible for the rapid healing in this
case is:
1. Remnant skin appendages.
2. Underlying connective tissue.
3. Minimal edema and earthema.
4. Granulation tissue.
Ans: 4

135. A 12 year old girl complains of pain presisting in his left leg. For several weeks with a low
grade fever. A radiograph reveals a mass in the diaphyseal region of the left femur with
overlying cortical erosion and soft tissue extension. A biopsy of the lesion shows numerous
small round cells, rich in PAS positive diastase sensitive granules. The most likely histological
diagnosis is:
1. Osteogenic sarcoma.
2. Osteoblastoma.
3. Ewing’s sarcoma.
4. Chondroblastoma.
Ans: 3

136. In a 2 months old infant undergoing surgery for biliary atresia, you would avoid one of the
following anaesthetic:
1. Thiopentone.
2. Halothane.
3. Propofol.
4. Sevoflurance.
Ans: ?

137. The ideal muscle relaxant used for a neonate undergoing porto-enterostomy for biliary
atresia is:

www.EduMedWeb.com www.EduMedWeb.com www.EduMedWeb.com


www.EduMedWeb.com www.EduMedWeb.com Page 22

1. Altracurium.
2. Vecuronium.
3. Pancuronium.
4. Rocuronium.
Ans: ?

138. Visual analogue scale (VAS) is most widely used to measure:


1. Sleep.
2. Sedation.
3. Pain intensity.
4. Depth of Anaesthesia.
Ans: 3

139. Sodium 2-mercapto ethance sulfonate (mesna) is used as a protective agent in:
1. Radiotherapy.
2. Cancer chemotherapy.
3. Lithotrypsy.
4. Hepatic encephalopathy.
Ans: 2

140. Durring laryngoscopy and endo-tracheal intubation which of the maneuver is not
performed:
1. Flexion of the neck.
2. Extension of Head at the atlanto-occipital joint.
3. The laryngoscope is lifted upwards levering over the upper incisors.
4. In a straight blade laryngoscope, the epiglottis is lifted the tip.
Ans: 1

141. In a young patient who had extensive soft tissue and muscle injury, which of these muscle
relaxants used for endotracheal intubation might lead to cardiac arrest:
1. Atracurium.
2. Suxamethonium.
3. Vecuronium.
4. Pancuronium.
Ans: 2

142. The diuretic group that does not require access to the tubular lumen to induce diuresis is:
1. Carbonic anhydrase inhibitor.
2. Na-Cl symport inhibitor.
3. Mineralocorticoid antagonist.
4. Na-K symport inhibitor.
Ans: 3

143. A vasopressin analogue does not produce therapeutic effect through vasopression V-2
receptor in which of the following:
1. Central diabetes insipidus.
2. Bleeding esophageal varices.
3. Type I van Willebrand’s disease.
4. Primary nocturnal enuresis.
Ans: 2

www.EduMedWeb.com www.EduMedWeb.com www.EduMedWeb.com


www.EduMedWeb.com www.EduMedWeb.com Page 23

144. Regarding neonatal circumcision, which one of the following is true:


1. It should be done without anaesthesia, as it is hazardous to give anaesthesia.
2. It should be done without anaesthesia, as neonates do not perceive pain as adult.
3. It should be done under local anaesthesia only.
4. General anaesthesia should be given to neonates for circumcision as they also feel pain as
adult.
Ans: 3

145. The following statement is not true about the use of clonidine in the treatment of
hypertension..
1. Reduction in central sympathetic outflow.
2. Increase in LDL-cholesterol in prolonged use.
3. Sedation ad xerostomia are common side effects.
4. It can be combined with vasodilators.
Ans: 2

146. The following is not true about the use of beta-blockers in heart failure:
1. It should be initiated at a nery low dose.
2. It most effective in new-onset decompensated heart failure.
3. Slow upward titration of dose is required.
4. Carvedilol is most widely used in this condition.
Ans: 2

147. The following statement is not true about sotalol:


1. It is a non-selective beta-blocker.
2. It prolongs action potential duration troughout the heart.
3. It is excreted through bile following hepatic metabolism.
4. Polymorphic ventricular tachycardia is a common side effect.
Ans: 4

148. For drugs with first-order kinetics, the time required to achieve steady state levels can be
predicted from:
1. Volume of distribution.
2. Half life.
3. Clearance.
4. Loading dose.
Ans: 2

149. All of the following drugs are metabolised by acetylation except:


1. INH.
2. Sulfonamides.
3. Ketoconazole.
4. Hydralazine.
Ans: 3

150. All the following cephalosporins having good activity against Pseudomonas aerugenosa
except:
1. Cephadroxil.
2. Cefepime.
3. Cefoperazone.
4. Ceftazidime,

www.EduMedWeb.com www.EduMedWeb.com www.EduMedWeb.com


www.EduMedWeb.com www.EduMedWeb.com Page 24

Ans: 1

151. Low dose progestational contraceptives primarly act on:


1. Oviductal motility.
2. Uterine endometrium.
3. Cervix.
4. Pituitary.
Ans: 3

152. There is a mid-cycle shift in the basal body temterature (BBT) after ovulation in women.
This is coused by:
1. FSH-peak.
2. LH- peak.
3. Oestrsdiol.
4. Progesterone.
Ans: 4

153. Various cells respond differentially to a second messenger (such as increased CAMP)
because they have different.
1. Receptors.
2. Enzymatic composition.
3. Nuclei.
4. Membrane lipids.

154. A 60 year old hypertensive patient on Angiotensin II receptor antagonists (losartan) is


posted for hernia repair surgery. The anti- hypertensive drug should be:
1. Continued till the day of operation.
2. Discontinued 24 hrs. preoperatively.
3. Discontinued one week preoperatively.
4. Administered in an increased dosage on the day of operation.
Ans: 2

155. Infertility is a common feature in ‘Sertoli cell only’ syndrome, because:


1. Too many Sertoli cells inhibit spermatogenesis via inhibin.
2. Proper blood-testis barrier is not established.
3. There is no germ cells in this condition.,
4. Sufficient numbers of spermatoza are not produced.
Ans: 3

156. Most accurate measurment of extracellular fluid volume (ECF) can be done by using:
1. Sucrose.
2. Mannitol.
3. Inulin.
4. Aminopyrine
Ans: 3

157. A shift of posture from supine to upright posture is associated with cardiovasucular
adjustments. Which of the following is NOT true in this context:
1. Rise in central venous pressure.
2. Rise in heart rate.
3. Decrease in cardiac output.

www.EduMedWeb.com www.EduMedWeb.com www.EduMedWeb.com


www.EduMedWeb.com www.EduMedWeb.com Page 25

4. Decrease in stroke volume.


Ans: 3

158. Dystrophic gene mutation leads to:


1. Myasthenia gravis.
2. Motor neuron disease.
3. Poliomyelitis.
4. Duchenne Muscular Dystrophy.
Ans: 4

159. Osteoclast has specific receptor for:


1. Prathyroid hormone.
2. Calcitionin.
3. Thyroxin.
4. Vit D3.Ans: 2

160 Which of the following is not a ysual feature of right middle cerebral artery territory infarct?
1. Aphasia.
2. Hemiparese.
3. Facial weakness.
4. Dysarthria.
Ans: 1

161. The treatment of choice in Attention Deficit Hyperactivity Disorder is:


1. Haloperidol.
2. Imipramine.
3. Methylphenidate.
4. Alprazolam.
Ans: 3

162. The following is a Schneider’s first rank symptom:


1. Persecutory delusion.
2. Voices commenting on actions.
3. Delusion of guilt.
4. Incoherence.
Ans: 2

163. A middle aged man presented with pain in back, lack of interest I recreational activities, low
mood, lethargy, decreased sleep and appetite for two months. There was no history suggestive
of delusions of hallucinations. He did not suffer from any chronic medical illness. There was no
family history of psytchiatric illness. Routine investigations including haemogram, renal function
tests, liver function testsm electrocadiogram did not reveal any abnormality. This patient should
be treated with:
1. Haloparidol.
2. Sertraline.
3. Alprazolam.
4. Olanzapine.
Ans: 3

164. An elderly house wife lost her husband who died suddenly of Myocardial infarction couple
of years ago. They had been staying alone for almost a decade with infrequent visits from her

www.EduMedWeb.com www.EduMedWeb.com www.EduMedWeb.com


www.EduMedWeb.com www.EduMedWeb.com Page 26

son and grandchildren. About a week after the death she heard his voice clearly talking to her
as he would in a routine manner from the next room. She went to check but saw nothing.
Subsequently she often heard his voice conversing with her and she would also discuss her
daily matters with him. This however, provoked anxiety and sadness of mood where she was
preoccupied with his thought. She should be treated with:
1. Clomipramine.
2. Alprazolam.
3. Electroconvulsive therapy.
4. Haloperidol.
Ans: 4

165. Yawning is a common feature of


1. Alcohol withdrawal
2. Cocaine withdrawal
3. Cannabis withdrawal
4. Opioid withdrawal
Ans: 2

166. The differential diagnosis of retinoblastoma would include all except:


1. Persistent hyperplastic primary vitreous
2. Coat’s disease.
3. Retinal astro cytoma
4. Retinal detachment
Ans: 4

167. Which of the following imaging modality is most sensitive to detect early renal tuberculosis.
1. Intravenous urography
2. Ultrasound
3. Computed tomography
4. Magnetic resonance imaging
Ans: 1

168. A young man with tuberculosis presents with massive recurrent hemoptysis. For
angiographic treatment which vascular structure should be avaluated first.
1. Pulmonary
2. Bronchial artery
3. Pulomary vein
4. Superior vena cava
Ans: 2

169. Which one of the following imaging modalities is most sensitive for evaluation of extra-
adrenal phaeochromocytoma.
1. Ultrasound
2. CT
3. MRI
4. MIBG scan
Ans: 4

170. On 3 phase 99mTc-MDP bone scan, which of the following bone lesions will show least
osteoblastic activity.
1. Paget’s disease

www.EduMedWeb.com www.EduMedWeb.com www.EduMedWeb.com


www.EduMedWeb.com www.EduMedWeb.com Page 27

2. Osteoid Osteoma
3. Fibruous Displasia
4. Fibruous cortical defect
Ans: 4

171. Use of tomoxifen in carcinoma of breast patients does not lead to the following side effects:
1. Thromboembolic events
2. Endometrial carcinoma
3. Cataract
4. Cancer in opposite breast
Ans: 4

172. For the treatment of deep seated tumors, the following rays are used.
1. X-rays and Gamma-rays
2. Alpha rays and Beta-rays
3. Electrons and positrons
4. High power laser beams
Ans: 1 (4??)

173. Stereotactic Radio-surgery is a form of:


1. Radiotherapy
2. Radioiodine therapy
3. Robotic surgery
4.Cyro Surgery
Ans: 1 This is focused high dose radiation to destroy tumors less than 3 cm.

174. Which of the following ia not an oncological emergency


1. Spinal cord compression.
2. Superior venacaval syndrome
3. Tumorlysis syndrome
4. Carcinoma cervix stage-III”B” with Pyometra.
Ans: 4

175. The level of alpha fetoprotein is raised in all of the following except.
1. Cirrohosis of liver.
2. Hepatocellur carcinoma
3. Yolksac tumor
4. Dysgerminoma
Ans: 3

176. Aniseikonia means:


1. Difference in the axial length of the eyeballs.
2. Difference in the size of corneas
3. Difference in the size of pupils
4. Difference in the size of images formed by the two eyes.
Ans: 4

177. Ophthalmoplegic migraine means:


1. When headache is followed by complete paralysis of the IIIrd and VI nerve on the same side
as the hemicrania.

www.EduMedWeb.com www.EduMedWeb.com www.EduMedWeb.com


www.EduMedWeb.com www.EduMedWeb.com Page 28

2. When the headache is followed by partial paralysis of the IIIrd nerve on the same side as the
hemicrania with out any scotoma.
3. Headache associated with IIIrd, IVth and VIth nerve paralysis.
4. Headache associated with optic neuritis
Ans: 3

178. Surface ectoderm gives rise to all of the following structures except.
1. Lens
2. Corneal epithelium
3. Conjunctival epithelium
4. Anterior layers of iris.
Ans: 4

179. Enucleation of the eyeball is contraindicated in


1. Endophthalmitis
2. Panophthalmitis
3. Intraocular tumour
4. Painful blind eye.
Ans: 2

180. The tratment of congenital glaucoma is


1. Essentially topical medications
2. Trabeculoplasty
3. Trabeculotomy with trabeculectomy
4. Cyclocrytherapy.
Ans: 3

181. Hypochloremia hypokalemia and alkalosis are seen in:


1. Congenital hypertrophic pyloric stenosis
2. Hirschsprung’s disease.
3. Esophageal atresia
4. Jejunal atresia
Ans: 1

182. There is a high risk of renal dysplasia in


1. Posterior urethral valves.
2. Bladder extrophy
3. Anorectal maloformation
4. Neonatal sepsis
Ans: 1

183. Cells from the neural crest are involved in all except.
1. Hirscsprung’s disease
2. Neuroblastoma
3. Primitive neuroectodermal tumour
4. Wilm’s tumour
Ans: 4

184. A ‘Malignant pustule’ is a term used for


1. An infected malignant melanoma
2. A carbuncle

www.EduMedWeb.com www.EduMedWeb.com www.EduMedWeb.com


www.EduMedWeb.com www.EduMedWeb.com Page 29

3. A rapidly spreading redent ulcer.


4. Anthrax of skin
Ans: 4

185. A warthin’s tumour is:


1. An adenolymphoma of parotid gland
2. A pleomorphic adenoma of the parotid
3. A carcinoma of the parotid .
4. A carcinoma of submandibular salivary gland
Ans: 1 warthin’s

186. A newborn baby had normal APGAR score at birth and developed excessive frothing
andchoking on attempted feeds. The investigation of choice is:
1. Esophagoscopy
2. Bronchoscopy
3. MRI chest
4. X-ray chest and abdomen with the red rubber catheter passed per orally into esophagus
Ans: 4

187. A new born baby has been referred to the casualty as a case of congenital diaphragmatic
hernia. The first clinical intervention is to:
1. Insert a central venous pressure line.
2. Bag and mask ventilation
3. Insert a nasogastric tube.
4. Ventilate with high frequency ventilator
Ans: 3

188. One year old male child is presented with poor urinary stream since birth. The investigation
of choice for evaluation is:
1. Voiding cystourethrography (VCUG)
2. USG bladder
3. Intravenous urography
4. Uroflowmetry
Ans: 1

189. Which of the following statements about peptic ulcer disease is true.
1. Helicobacter pylori eradication increases the likelihood of occurrence of complications.
2. The incidence of complications has remained unchanged.
3. The incidence of Helicobacter pylori re-infection in India is very low.
4. Helicobacter pylori eradication does not alter the recurrence ratio.
Ans: ?

190. Which of the following is not a contraindication for extra corporeal Shockwave Lithotripsy
(ESWL) for renal calculi?
1. Uncorrected bleeding diathesis
2. Pregnancy
3. Ureteric stricture
4. Stone in a calyceal diverticulum.
Ans: 4

191. Which of the following is not an appropriate investigation for anterior urethral stricture?

www.EduMedWeb.com www.EduMedWeb.com www.EduMedWeb.com


www.EduMedWeb.com www.EduMedWeb.com Page 30

1. Magnetic Resonance Imaging


2. Retrograde urethrogram
3. Micturating cystourethrogram
4. High frequency ultrasound
Ans: 1

192. The recommended treatment for preputial adhesions producing ballooning of prepuce
during micturition in a 2-year-old boy is:-
1. Wait and watch policy
2. Circumcision
3. Dorsal slit
4. Preputial adhesions release and dilatation
Ans: 3

193. All are correct about potassium balance except:


1. Most of potassium is intracellular
2. Three quarter of the total body potassium is found in skeletal muscle.
3. Intracellular potassium is released into extra-cellular space in response to severe injury or
surgical stress.
4. Acidosis leads to movement of potassium from extracellular to intracellular fluid compartment.
Ans: 4

194. Hypocalcaemia characterized by all except.


1. Numbness and tingling of circumoral region.
2. Hyperactive tendon reflexes and positive Chvostek’s sign.
3. Shortening of Q-T interval in ECG.
4. Carpopedal spasm
Ans: 3

195. Which of the following not true of gas gangrene.


1. It is caused by clostridium perfringens
2. Clostridium Perfringens is a gram-negative spore-bearing bacillus.
3. Gas gangrene is characterized by severe local pain crepitys and signs of toxemia.
4. High does penicillin and aggressive debridement of affected tissue is the treatment of
established infection.
Ans: 2 they are gram + ve

196. “Sleep apnea”, is defined as a temporary pause in breathing during sleep lasting at least.
1. 40 seconds
2. 30 seconds
3. 20 seconds
4. 10 seconds
Ans: 4

197. In a blast injury, which of the following organ is least vulnerable to the blast wave.
1. G.I. tract
2. Lungs
3. Liver
4. Ear drum
Ans: 3

www.EduMedWeb.com www.EduMedWeb.com www.EduMedWeb.com


www.EduMedWeb.com www.EduMedWeb.com Page 31

198. Regarding testicular tumour, the following are false except.


1. They are commonest malignancy in older man
3. Seminomas are radiosensitive
3. Only 25% of stage 1 teratomas are cured by surgery alone.
4. Chemotherapy rerely produces a cure in those with Metastatic disease.
Ans: 3

199. A young patient presents with history of dysphagia more to liquid than solids. The first
investigation you will do is:
1. Barium Swallow
2. Esophagoscopy
3. Ultrasound of the chest
4. C.T. Scan of the chest
Ans: 1

200. A 45 years old hypertensive male presented with sudden onset severe headache, vomiting
and neck stiffness. On examination he didn’t have any focal neurological deficit. His CT scan
showed blood in the Sylvain fissure. The probable diagnosis is:
1. Meningitis
2. Ruptured aneurysm
3. Hypertensive bleed.
4. Stroke
Ans: 2? 3?

www.EduMedWeb.com www.EduMedWeb.com www.EduMedWeb.com


www.EduMedWeb.com www.EduMedWeb.com Page 1

AIIMS PG MAY 2012


(Based on Memory)

1. 50 year old female has undergone mastectomy for Ca Breast. After Mastectomy, Patient is
not able toextend, adduct and internally rotate the arm. There is damage to nerve supplying
which muscle?
A. Latissimus Dorsi
B. Pectoralis Major
C. Teres Minor
D. Long head of Triceps
Ans: A

2. Muscle spared by complete transection of cranial part of accessory nerve


A. Stylopharyngeus
B. Palatopharyngeus
C. Salpingiopharyngeus
D. Cricopharyngeus
Ans: A

3. Cervical vertebra is differentiated from the Thoracic vertebra by the presence of


A. Foramen Transversarium
B. Triangular Foramina
C. Upward facing facets
D. Larger Vertebral Body
Ans: A

4. In complete unilateral damage to Hypoglossal nerve, all are true EXCEPT


A. Atrophy on affected side
B. Deviation of tongue towards the site of lesion
C. Deviation ofLarynx to the contralateral side during swallowing
D. Loss of tactile sensation on tongue on affected side
Ans: D

5. Which of the following is True?


A. Hypothalamus is part of brainstem
B. Occipital lobe is part of Cerebral Cortex
C. Medulla ia part of Limbic System
D. All of the above
Ans: B

PHYSIOLOGY
6. CSF pressure is mainly regulated by
A. Rate of CSF formation

www.EduMedWeb.com www.EduMedWeb.com www.EduMedWeb.com


www.EduMedWeb.com www.EduMedWeb.com Page 2

B. Rate of CSF Absorption


C. Cerebral Blood Flow
D. Venous Pressure
Ans: B

7. On exposure to cold, a neonate shows all of the following mechanism EXCEPT


A. Shivering
B. Crying and flexion of body like fetus position
C. Cutaneous Vasoconstriction
D. Increased production of Noradrenaline for breakdown of brown fat in adipose tissue
Ans: A

8. Without external cues, the sleep-wake cycle in humans


A. Does not continue
B. Continue with cycle length of 24 hours
C. Continue with cycle length of less than 24 hours
D. Continue with cycle length of more than 24 hours
Ans: D

9. All of the following are true for ADH, EXCEPT


A. Post-operative increase in secretion
B. Neurosecretion
C. Increased secretion when plasma osmolality is low
D. Act on distal tubule and increase permeability
Ans: C

10. Critical Closing volume is


A. Volume at the end of forceful expiration
B. Volume at the end of forceful inspiration
C. Volume remaining after Functional Residual Capacity is measured
D. Close to Residual Volume
Ans: D

11. During moderate exercise, blood flow to brain


A. Increases
B. Decreases
C. No Effect
D. First increases then decreases
Ans: C

12. Corticospinal injury is associated with all EXCEPT


A. Babinski sign present
B. Loss of fine movements in fingers and hand
C. Superficial abdominal reflex absent

www.EduMedWeb.com www.EduMedWeb.com www.EduMedWeb.com


www.EduMedWeb.com www.EduMedWeb.com Page 3

D. Clasp knife rigidity


Ans: C

13. Ventricular Depolarization stars from


A. Posterobasal part of ventricle
B. Basal part of ventricle
C. Uppermost part of Interventricular septum
D. Left part of Interventricular septum
Ans: D

14. Left lobe is responsible for


A. Appreciation of Music
B. Spatial orientation

BIOCHEMISTRY
15. Fluorescence means
A. Spontaneous illumination in dark
B. Release of longer wavelength light on absorbing light of short wavelength
C. Release of shorter wavelength light on absorbing light of longer wavelength
D. Release of equal wavelength light at constant rate
Ans: B

16. Gluconeogenesis in Fasting state is indicated by


A. Citrate activation by acetyl co-a carboxylase
B. Pyruvate carboxylase activation by acetyl co-A
C. Fructose 1, 6 bisphosphate activation by Pyruvate Kinase
D. Fructose 2, 6 bisphosphate activation by PFK - 1
Ans: C

17. DNA estimation can be done by


A. Spirometer
B. Spectrophotometer
C. pH meter
D. Sphygmometer
Ans: B

18. Food with maximum Cholesterol content


A. Egg
B. Coconut Oil
C. Hydrogenated Fats
D. Ghee (Hydrogenated)
Ans: A

19. If more than one codon codes for same amino acid, this phenomenon is known as—

www.EduMedWeb.com www.EduMedWeb.com www.EduMedWeb.com


www.EduMedWeb.com www.EduMedWeb.com Page 4

A. Degenerac
B. Frame-Shift Mutation
C. Transcription
D. Mutation
Ans: A

FORENSIC MEDICINE
20. Last organ to be dissected during autopsy in asphyxia death (May 2008)
A. Neck
B. Head
C. Abdomen
D. Thorax

21. Active partner in lesbianism is called as


A. Femme
B. Bugger
C. Catamite
D. Dyke
Ans: D

22. A person was found dead with bluish green frothy discharge at the angle of mouth and
nostrils. What is thediagnosis?
A. Arsenic poisoning
B. Copper poisoning
C. Mercury poisoning
D. Lead poisoning
Ans: B

23. Heat stiffening in muscles occurs above temperature


A. 30
B. 40
C. 50
D. 60
Ans: D

24. A man was found with suicidal shot on right temple. The gun was in his right hand. The skull
was in his right hand. The skull was burst open. There was charring and cherry red coloration in
the track inside. What can be said about shot?
A. Contact Shot
B. Close shot
C. Shot within range of smoking
D. Shot within range of tattooing
Ans: A

www.EduMedWeb.com www.EduMedWeb.com www.EduMedWeb.com


www.EduMedWeb.com www.EduMedWeb.com Page 5

25. Which of the following anti-estrogen drug is used in estrogen receptor positive breast
cancer?
A. Tamoxifen
B. Clomiphene Citrate
C. Estrogen
D. Adriamycin
Ans: A

26. A woman consumes several tabs of Amytryptalline. All of the following can be done except?
A. Sodium Phenobarbitone infusion for alkalization of urine
B. Gastric Lavage
C. Diazepam for seizure control
D. Atropine as antidote
Ans: D

27. According to recent Supreme Court Judgment, Doctor can be charged for Medical
Negligence under 304-A, only if
A. If he is from corporate hospital
B. If negligence is from inadvertent error
C. Res Ipsa Loquitor
D. Gross Negligence
Ans: D

28. Aperson breaks someone’s mandible in alleged fight. Police can


A. Arrest with warrant
B. Arrest without warrant
C. Declare him hostile and put him in jail
D. Put him in mental asylum

29. Choking is seen in


A. Revolver
B. Pistol
C. Shotgun
D. Rifle
Ans: C

30. In Breslau’s second life test, organ tested is


A. Brain
B. Heart
C. Lung
D. Stomach and Intestine
Ans: D

www.EduMedWeb.com www.EduMedWeb.com www.EduMedWeb.com


www.EduMedWeb.com www.EduMedWeb.com Page 6

31. A patient comes with Pinpoint pupil, salivation, tremors and red tears. Cholinesterase level
was 30% of Norma. Probable Diagnosis is (?)
A. OP Poisoning
B. Dhatura
Ans: A

32. A case of murder with gunshot is reported. A metal bullet is recovered from the body.
Primary and Secondary markings on a metal bullet can be used for
A. Identification of weapon
B. To know the range of fring
C. Severity of tissue damage
Ans: A

PATHOLOGY
33. Edema in Nephrotic Syndrome is due to
A. Na + and Water retention
B. Increased venous pressure
C. Protein Loss
D. Hyperlipidemia
Ans: C

34. Fixative used in Histopathology


A. Ethyl Alcohol
B. Buffered Neutral 10% Formalin
C. Glutaraldehyde
D. Bouin’s Solution
Ans: B

35. In Barrett’s Esophagus


A. Intestinal Metaplasia
B. Intestinal Dysplasia
C. Gastric Dysplasia
D. Squamous Metaplasia
Ans: A

36. About Bombay Blood Group, all are true EXCEPTA


A. Lack of A, B and H antigen on RBCB. Lack of A, B and H antigen in Saliva
C. Lack of other Blood Group Antigens
D. Presence of Anti-A, Anti-B and Anti-H antibody in blood
Ans: C

37. Most common mechanism of GERD


A. Transient decrease in pressure at lower esophagus
B. Hypotension at lower esophageal sphincter

www.EduMedWeb.com www.EduMedWeb.com www.EduMedWeb.com


www.EduMedWeb.com www.EduMedWeb.com Page 7

C. Hiatus Hernia
Ans: A

38. Bradykinin in acute inflammation causes


A. Vasoconstriction
B. Pain at the sign of inflammation
C. Bronchodilatation
D. Decreased vascular permeability
Ans: B

A.B.C.D.
Mitochondrial Inheritance Autosonal DominantX-linked Recessive Autosomal Recessive
Ans: A
40. Which of the following suggest Irreversible Cell Injury?
A. Mitochondrial Swelling
B. Amorphous Density in Mitochondria
C. Ribosomal Disaggregation
D. Chromosomal
Ans: B

41. In Lewis Triple Response, redness when skin is scratched with a pointed object is seen due
to
A. Axon Reflex causing Vasoconstriction
B. Histamine Release due to local injury to mast cells by pointing device
C. Free Nerve Endings
D. Endothelial Damage leading to increase in permeability
Ans: A

PHARMACOLOGY
42. Drug causing acquired Nasolacrimal duct obstruction
A. Timolol
B. Pilocarpine
C. Dorzolamide
D. Brimonidine

43. Antibiotic associated with Colitis


A. Neomycin
B. Chloramphenicol
C.Clindamycin
D. Vancomycin
Ans: C

39. Inheritance is
44. Drugs banned by International Olympics Committee are all EXCEPT

www.EduMedWeb.com www.EduMedWeb.com www.EduMedWeb.com


www.EduMedWeb.com www.EduMedWeb.com Page 8

A.B.C.D.
SalbutamolSodium Cromoglycate Spironolactone Erythropoie
Ans: B

45. What is true about First Order Kinetics?


A. Constant amount of drug is eliminated
B. Rate of elimination is constant
C. Free drug concentration increased after successive doses
D. Rate of elimination is proportional to plasma concentration of the drug

46. Despite having short half-life, PPI acts longer because


A. Causes irreversible inhibition of proton pump. Acid secretion suppressed till new pumps are
generated
B. Get trapped in canaliculi
C. Enteric coated tablets causing slow release
D. Prodrugs and needs generation of active metabolites
Ans: A

47. False about Antiepileptics is


A. Phenytoin and Carbamazepine act by prolongation of inactivated state of Na+Channels
B. Diazepam is anti-epileptic
C. Carbamazepine used for Trigeminal Neuralgia
D. Lamotrigine acts by opening og GABA mediated CI-channels
Ans: D

MICROBIOLOGY
48. In a 5 year old boy who has history of pyogenic infections by bacteria with polysaccharide-
rich capsules, which of the following investigations should be done?
A. IgA deficiency
B. IgGI deficiency
C. IgG2 deficiency
D. IgA and IgG2 deficiency

49. Sputum can be disinfected by all EXCEPT


A. Autoclaving
B. Boiling
C. Cresol
D. Chlorhexidine
Ans: D

50. Which of the following factor is responsible for deciding whether an immunoglobin will
remain membranebound or get secreted
A. RNA Splicing
B. Class switching

www.EduMedWeb.com www.EduMedWeb.com www.EduMedWeb.com


www.EduMedWeb.com www.EduMedWeb.com Page 9

C. Differential RNA Processing


D. Allelic Exclusion
Ans: C

51. A patient presents with signs of pneumonia. The bacterium obtained from sputum was a
gram positive cocci which showed alpha hemolysis on sheep agar. Which of the following test
will hellp to confirm the diagnosis?
A. Optochin sensitivity
B. Bacterium sensitivity
C. Coagulase test
D. cAMP test
Ans: A

52. With reference to antibiotic resistance, all of the following statements are true EXCEPT
A. Most common mechanism is production of neutralizing substances
B. Plasmid mediated antibiotic resistance is always transmitted vertically
C. Complete elimination of the target is the mechanism of resistance to Vancomycin in some the
strains of enterococcus
D. Alteration of target is the mechanism of resistance in some of the Pneumococcal strains
Ans: B

53. A patient has Bilirubin = 0.9 mg/dl, elevated SGPT & SGOT HBaAg positive, Anti HBC Ig M
negative, Anti Hepatitis E Ig M positive. Patient has
A. Chronic Hepatitis B infection with Hepatitis E super infection
B. Hepatitis E infection
C. Co- infection with Hepatitis B & Hepatitis E
D. Infection with mutant Hepatitis B virus
Ans: A

54. A young male patient presented with urethral discharge. On urine examination pus cells
were found but noorganisms. Which method would be best for cultures?
A. McCoy Cell Line
B. Thayer Martin Medium
C. L.J. Medium
D. Levinthal Medium
Ans: A

55. A person working in abattoir with pustules on hand which turned into ulcer. Which of the
following would help best in diagnosis?
A. Polychrome Methvlene Blue
B. Carbol fuschin
C. Acid Fast Stain
D. Calcoflour white
Ans: A

www.EduMedWeb.com www.EduMedWeb.com www.EduMedWeb.com


www.EduMedWeb.com www.EduMedWeb.com Page 10

PREVENTIVE AND SOCIAL MEDICINE


56. In certain population, there were 4050 births in last one year. There were 50 still births. 50
infants died within 7 days where as 150 died within first 28 days. What is Neonatal Mortality
Rate?
A. 50
B. 62.5
C. 12.5
D. 49.5
Ans: A

57. Leprosy is not yet eradicated because


A. No effective vaccine
B. Highly Infectious but low pathogenicity
C. Only humans are reservoir
D. Long incubation period
Ans: D

58. A person found some correlation between fatty food intake and diseases due to obesity. He
did this bycollecting data from food manufacturers and hospitals respectively, such a study is
A. Ecological
B. Cross sectional
C. Psephlogical study
D. Experimental study
Ans: A

59. Bias can be eliminated by all EXCEPT


A. Matching
B. Blinding
C. Randomization
D. Multivariate Analysis
Ans: D

60. A study revealed lesser incidence of carcinoma colon in pure vegtarians than non-
vegetarians; by which it was concluded that Beta-carotene is protective against cancer. This
may not be true because the vegetarian subjects may be consuming high fiber diet which is
protective against cancer. This is an example of
A. Multifactorial Causation
B. Causal Association
C. Confounding factor
D. COMMON association
Ans: C

61. According to The Workmen’s Compensation Act, 1923, which of the following is considered
an occupational disease?

www.EduMedWeb.com www.EduMedWeb.com www.EduMedWeb.com


www.EduMedWeb.com www.EduMedWeb.com Page 11

A. Typhoid
B. Anthrax
C. Tetanus
D. Dengue
Ans: B

62. True about Indian Reference Male is


A. Age 20-39 years
B. Weight 65 kg
C. Work is mainly dentary
D. Works for 10 hours
Ans: C

63. Application of Incubation period is all EXCEPT


A. To differentiate co-primary cases from secondary cases
B. To find out time for isolation
C. To find out time for Quarantine
D. To prevent infection to the contacts of the infected person
Ans: B

64. Common to both Acute and ChMalnutrition is


A. Weight for Age
B. Height for Age
C. Weight for Height
D. BMI
Ans: A

65. True about Rashtriya Swasthya Bima Yojna


A. Insurance company runs the scheme
B. Government run Insurance scheme for employees
C. Government run Insurance scheme for poor’s
Ans: C

66. Natural Disaster causing maximum deaths


A. Hydrological
B. Geological
C. Meteorological
D. Climatological
Ans: A

67. In 13-15 year female child, Recommended Daily Protein Intake is


A. 0.68
B. 0.95
C. 1

www.EduMedWeb.com www.EduMedWeb.com www.EduMedWeb.com


www.EduMedWeb.com www.EduMedWeb.com Page 12

D. 1.33
Ans: D

68. In Acute Flaccid Paralysis Surveillance, Re-Evaluation of post-polio residual Paralysis is


done at
A. 60 days
B. 90 days
C. 6 weeks
D. 6 months
Ans: A

69. Which of the following is best suited for the role of social worker?
A. Health professional involved in physiotherapy
B. Health professional involved in coping strategies, interpersonal skills, adjustment with family
C. A person involved in finding jobs and economic support for disabled
D. Health professional
Ans: B

70. A sexually active, long distance truck driver’s wife comes with vaginal discharge. Under
Syndromic Approach, which drug should be given
A. Metronidazole, Azithromycin, Fluconazole
B. Metronidazole
C. Azithromycin
D. Metronidazole and Fluconazole
Ans: A

71. All of the following are helpful for elimination of Filariasis, EXCEPT
A. Microfilarias do not multiply in vectors
B. They multiply in humans
C. Larvae are deposited on skin surface where they can’t survive

72. In WHO “Road to Health” chart, upper and lower limit of represents
A. 30 percentile for boys and 3 percentile for girls
B. 50 percentile for boys and 3 percentile for girls
C. 30 percentile for boys and 5 percentile for girls
D. 50 percentile for boys and 5 percentile for girls
Ans: B

OPHTHALMOLOGY
73. On Fundoscopic Examination a patient, a red dot is seen. He has a history of being hit by a
tennis ball. What is the probable cause?
A. Berlin’s Edema
B. Macular Tear
C. Macular Hole

www.EduMedWeb.com www.EduMedWeb.com www.EduMedWeb.com


www.EduMedWeb.com www.EduMedWeb.com Page 13

D. Macular Bleed

74. In Vision 2020, the target for Secondary Service Center is for how much population?
A. 10000
B. 50000
C. 1 lac
D. 5 lac

75. In Vision 2020,Ophthalmologist per population ratio is


A. 5000
B. 10000
C. 50000
D. 1 lac
Ans: C

76. Occular Dendritic Cells have


A. HLA 1
B. HLA 2
C. Both
D. None
Ans: C

77. Corneal Dystrophies are usually


A. Primary Bilateral
B. Primary Unilateral
C. Primary Bilateral with Systemic Disease
D. Primary Unilateral Systemic Disease
Ans: A

78. Weakness of both Adduction and Abduction is seen in


A. Duane’s Retraction Syndrome Type 1
B. Duane’s Retraction Syndrome Type 2
C. Duane’s Retraction Syndrome Type 3
Ans: C

79. A patient presented with unilateral proptosis, which was compressible and increases on
bending forward. Northrill or bruit was present. MRI shows a retroorbital mass with
enhancement. The likely diagnosis is
A. A V Malformation
B. Orbital Encephalocele
C. Orbital Varix
D. Neurofibromatosis
Ans: C

www.EduMedWeb.com www.EduMedWeb.com www.EduMedWeb.com


www.EduMedWeb.com www.EduMedWeb.com Page 14

80. In patient with anterior uveitis, decrease in vision due to posterior segment involvement can
occur because of
A. Visual Floaters
B. Inflammatory Disc Edema
C. Exudative Retinal Detachment
D. CME
Ans: D

81. Area of retina seen under Direct Ophthalmoscope


A. 1 DD
B. 2 DD
C. 3DD
D. 4DD
Ans: b

82. A 40 year male with diabetes presents with vitreous hemorrhage. What is the cause?
A. Neovascularization at disc
B. Posterior Retinal Detachment
C. Central Retinal vein Occlusion
D. Trauma to Central Retinal Artery
Ans: A

83. Gyrate Atrophy patient with defective Ornithine Transcarbamoylase will be benefited by
A. Ornithine free diet
B. Arginine free diet
C. Pyridoxine and Vit B12
D. Vitamin B1, B6 and B12
Ans: A

84. Iritis, Vitritis. Unilateral white patches on retina


B. Toxoplasmosis
C. CMV
D. ENT

85. In Electrocochleography
A. Probe, stimulation
B. Summation of microphonics
C. AP of cochlear nerve
D. Outer hair cells

86. Vestibular Evoked Myogenic Potential detects


A. Cochlear Nerve
B. Superior Vestibular Nerve
C. Inferior Vestibular Nerve

www.EduMedWeb.com www.EduMedWeb.com www.EduMedWeb.com


www.EduMedWeb.com www.EduMedWeb.com Page 15

D. Inflammatory Myopathy

87. Second Primary Tumor of Head & Neck most commonly suspected in malignancy of
A. Oral cavity
B. Larynx
C. Hypopharynx
D. Paranasal sinuses
Ans: A

88. Initial screening test for newborn hearing disorder Hearing test used in newborn
A. ABR – Auditory Brainstem Response
B. Autoacoustic Emissions
C. Free Field Audiometry
D. AABR

MEDICINE
89. All of the following are true for Hepatitis B EXCEPT?
A.
B.
C.
D.
Vertical transmission more important than horizontal Age of onset determines prognosis Period
of Communicability lasts several months Virus can be found in blood 1 month before jaundice
Ans: A

90. The acid base status of a patient reveals a pH=7.45 and and pCO2=30 mmHg. The patient
has partially compensated.
A. Metabolic Acid
B. Metabolic Alkalosis
C. Respiratory Alkalosis
D. Respiratory Acidosis
Ans: C

91. In Alternative Medicine, which of the following can be used for Heart Failure?
A. Gingko Biloba
B. Neem tree extract
C. Terminalia Arjuna
D. St. John’s wort
Ans: C

92. SLE, Anti TGFC antibody given. What can be seen in dermoepidermal junction?
A. Anti-Nuclear antibody
B. Immune-complex deposition
C. Anti-Collagen antibody

www.EduMedWeb.com www.EduMedWeb.com www.EduMedWeb.com


www.EduMedWeb.com www.EduMedWeb.com Page 16

D. Anti-Epithelial Cell antibody


Ans: B

93. Post-transplantation lymphoma is due to


A. CMVB
2. EBVC
3. HPV
Ans: B

94. Most common type Spinocerebral Ataxia in India is


A. SCA 1
B. SCA 2
C. SCA 3
D. SCA 4
Ans: B

95. In Posterier Intercommunicating Artery Aneurysm, which structure has most chances to be
compressed
A. Occulomotor Nerve
B. Trochlear Nerve
C. Hypophysis
D. Cerebellum
Ans: A

96. In Tension Pneumothorax


A. Chest wall expansion
B. Negative intraleural pressure
C. Decreased surfactant
D. Increased compliance of lung
Ans: A

97. Intracorpuscular Defect of Erythrocyte


A. Autoimmune Hemolytic Anemia
B. Hereditary Spherocytosis
C. Microangiopathic Hemolytic Anemia
Ans: B

98. A 23 year old female presenting with anemia, jaundice for 2 years. Peripheral smears
showing spherocytes. The best investigation to be done is
A. Osmotic Fragility Test
B. Coomb’s Test
C. Reticulocyte Count
D. Bone Marrow Aspiration
Ans: B

www.EduMedWeb.com www.EduMedWeb.com www.EduMedWeb.com


www.EduMedWeb.com www.EduMedWeb.com Page 17

99. Not a side-effect of Growth Hormone Therapy?


A. Gynecomastia
B. Hypoglycemia
C. Pseudotumor of Brain
D. Slipped Capital Femoral Epiphysis
Ans: B

100. A male come with anemia, weakness and constipation since 2 years. He has abdominal
pain. Mostprobable diagnosis?
A. Lead toxicity
Ans: A

101. A patient with Hypertension, comes with severe unilateral headache and vomiting. On MRI,
lateralized convex lesion seen. He has neck stiffness. The diagnosis is
A. Intracerebral Hemorrhage
B. Subarachnoid Hemorrhage
C. Meningitis
Ans: B

102. Drug induced Lupus seen in all EXCEPT


A. Sulfonamides
B. Hydralazine
C. Isoniazid
D. Penicillin
Ans: D

103. All about Leukemia are FALSE EXCEPT


A. CML occurs after 50 years of age
B. All occurs before 1 year in neonates
C. All in before 1 year has poor prognosis
D. Hairy Cell Leukemia occurs before 50 years of age
Ans: A

104. Patient with Pneumococcal Brain Abscess. Culture is sent for Antibiotic sensitivity. Which
empiricalantibiotic is given till culture sensitivity result come?
A. Penicillin G
B. Ceftriaxone + Metronidazole
Ans: B

105. Psedotumor Cerebri is seen in A.B.C.D. Obese Female of 20-40 years


B.Obese male of 20-40 years
C. Thin Female of 50-60 years
D. Thin male of 50-60 years

www.EduMedWeb.com www.EduMedWeb.com www.EduMedWeb.com


www.EduMedWeb.com www.EduMedWeb.com Page 18

Ans: A

106. Photosensitivity is a symptom of Porphyria. All of the following enzyme deficiencies have
photosensitivity EXCEPT
A. Uroporphyrinogen decarboxylase
B. Uroporphyrinogen oxidase
C. Coproporphyrinogen oxidase
D. Protoporphyrinogen oxidase
Ans: C

107. A 60 year old has progressive slowing of movements since 2 years. She has Gigidity and
rectangular slow wave jerking movements. Most probable diagnosis is
A. Superanuclear Palsy
B. Lewy-Body Dementia
C. Parkinsonism
D. Multiple System Atrophy
Ans: A

108. A Penicillin allergic patient has Rheumatic Fever. Which of the following grug should be
prescribed?
A. A Penicillin G
B. Sulfoxazole
C. Sulfasalazine
D. Streptomycin

109. A neonate present with bleeding from umbilical stump, rest of the examination is normal.
Probable diagnosis is
A. Factor X deficiency
B. von Willebrand Disease
C. Glanzman Thromboasthenia
D. Bernard Soulier Syndrome
Ans: A

110. Which scientific principle is the basis for T hermodilution method used in measurment
of cardiac output by Pulmonary Catheter?
A.
B.
C.
D.
Hagen-Poisseuille Principle Stewart-Hamilton Principle Bernoulli’s Principle Universal Gas
Equation
Ans: B

111. Most common of Immuno proliferative small intestinal disease

www.EduMedWeb.com www.EduMedWeb.com www.EduMedWeb.com


www.EduMedWeb.com www.EduMedWeb.com Page 19

A. Malabsorption
B. Obstruction
C. Bleeding
D. Abdominal Pain
Ans: D

112. “Fever Blister” is due to


A. Primary HSV-1 Infection
B. Vericella Zoaster
C. Reactivation of HSV-1 Infection
D. CMV
Ans: C

113. An alcoholic with poor judgment and decreased in skilled movements. Blood alcohol level
would be
A. 30-80 mg/dl
B. 80-200 mg/dl
C. 200-300 mg/dl
D. > 300 MG/DL
Ans: B

114. A 56 year old man presents with unilateral headache, diplopia and fever. He had difficulty
in chewing. MRI of brain was normal and ESR was raised. Treatment is
A. Prednisolone
B. High Dose Aspirin
C. Coxibs
D. Valproate
Ans: A

115. Type of wave in Metabolic Encephalopathy


A. Alpha
B. Beta
C. Gamma
D. Delta

116. In 2 patients with Atherosclerosis, one is diabetic and other is non- diabetic. In relation to
non- diabetic, diabetic patient has 100 times increased risk of
A. MI
B. Cerebrovascular Stoke
C. Lower Limb Ischemia
D. Vertebrobasilar Insufficiency
Ans: C

117. In “Pneumonia Severity Scale”, most important factor is

www.EduMedWeb.com www.EduMedWeb.com www.EduMedWeb.com


www.EduMedWeb.com www.EduMedWeb.com Page 20

A. Age
B. Congestive Heart Failure
C. Hypothermia
D. Hyponatremia
Ans: A

118. A boy is suffering from Acute Pyelonephritis. Most specific urinary finding
A. WBC Cast
B. Gram Staining for Bacteria
C. Leucocyte Esterase positive
D. Positive Nitrite Test
Ans: A

119. A lady had Meningioma with inflammatory edematous lesion. She was planned for surgery.
Juniorresident’s mistake in writing pre-op notes is
A. Stop Steroids
B. Wash head with shampoo
C. Antibiotic Sensitivity
D. Antiepileptic
Ans: A

120. A lady had brethlessness, arthralgia and uveitis. Diagnosis is


A. Ocular Sarcoid
B. HLA B27 related Uveitis
C. Tuberculosis
Ans: A

121. A 46 year old man with constipation, abdominal pain, mucous in stool and sense of
incomplete evacuation since 4 years. On examination, there was left iliac fossa tenderness.
Diagnosis is
A. Ca Colon
B. Diverticular disease
Ans: C

122. Hematopoietic Stem Cells differ Committed Progenitor Cells in


A. Can reconstitute Bone Marrow
B. Secrete Growth Factor
C. Terminal Cells are different
D. Nuclear Receptor present
Ans: A

123. In an ablated animal, aresearcher injects Myeloid Stem Cells, after the incubation period
he finds which type of cells?
A. T-Lymphocyte

www.EduMedWeb.com www.EduMedWeb.com www.EduMedWeb.com


www.EduMedWeb.com www.EduMedWeb.com Page 21

B. Erythrocyte
C. Fibroblast
D. Hematopoietic Stem Cells

124. Due to decelerations, Aorta can be ruptured at places at places where it is fixed EXCEPT
A. At Aortic Valve
B. Behind Esophagus
C. Behind Crura of the Diaphragm
D. At Ligamentum Arteriosus

125. A lady with cold intolerance and hoarseness of voice was detected with cardiomegaly.
Investigation to bedone to know the cause is
A. Coronary Angiography
B. Left Ventricle Angiography
C. Right Ventricle Angiography
D. Echocardiography
Ans: D

126. Midsystolic Click is heard in a female. Pathology seen in mitral valve is (AI 10)
A. Myxomatous degeneration
B. Aschoff Nodules
C. Sub Acute Bacterial Endocarditis
D. Ruptured Chordae Tendinae
Ans: A

127. Most significant association for Hepatocellular Carcinoma


A. Hepatitis A
B. Hepatitis B
C. EBV
D. CMV
Ans: B

128. A patient comes with transmural thrombi. Streptokinase and Warfarin is started. After
which further testthrombolysis should be stopped
A. Pericardial Friction Rub
B. Pericardial Effusion
C. Mobitz Type 2 Block
D. Leg vein thrombosis
Ans: C

129. Thrombolytics can be given in treatment of AMI, if patient comes within


A. 6 hours
B. 12 hours
C. 18 hours

www.EduMedWeb.com www.EduMedWeb.com www.EduMedWeb.com


www.EduMedWeb.com www.EduMedWeb.com Page 22

D. 24 hours
Ans: B or C

130. A n old hypertensive man, on treatment with Aspirin, Sorbitrate, and Atenolol suddenly fell
in bathroom. Relatives noted black stools. He presented in ER with hypotension, tachycardia.
This diagnosis is (?)
A. Gastric Ulcer
B. Cerebrovascular Accident
Ans: A

131. All are true EXCEPT


A.
B.
C.
D.
Amyloid Angiopathy is associated with Small Vessel Disease Alzheimer’s Disease is associated
with Amyloid Angiopathy Alzheimer’s Disease is related to small vessel Disease Alzheimer’s
Disease is not related to small vessel Disease
Ans: D

132. A patient with Atrial Fibrilation has clot in left Atrium. All of the following should be done in
management EXCEPT? (?)
A. DC Cardioversion with percutaneous Clot removal
B. Walfarin therapy
C. Open Commisurotomy and Clot removal
Ans: A

133. Malignant Hyperthermia, Metabolic Acidosis, Start treatment with


A. Iv Dantrolene
B. Na Bicarbonate
Ans: A

134. Most reliable test for diagnosing spinal TB is


A. MRI
B. CT guided Biopsy
C. PPD
Ans: B

135. All of the following can be seen in Massive Acute Blood Loss EXCEPT
A. Increase in Reticulocyte count
B. Increase in PCV
C.Increase in MCV
D. Increase in Neutophils
Ans: C

www.EduMedWeb.com www.EduMedWeb.com www.EduMedWeb.com


www.EduMedWeb.com www.EduMedWeb.com Page 23

136. Which of the following is incorrectly matched


A. Neurofibromatosis – Renal Artery Stenosis
B. Moyamoya disease – Aortic Aneurysm
C. Marfan’s Syndrome – Dural Ectasia
D. Muilbrey nanism – Constricitve Pericarditis
Ans: B

SURGERY
137. Patient is tenderness in left iliac fossa. There was 3 cm stone in renel pelvis and no
Hydronephrosis. Mostappropriate management is
A. Percutaneous Nephrolithostomy
B. ESWL
C. Diuretics
D. Medical dissolution therapy with Potassium Citrate
Ans: A

138. A 40 year male presents with hematemesis. On examination his BP was 90/60 mmHg and
Heart Rate was 120/min. Splenomegaly was also present. The most probable cause of his
bleeding is
A. Portal Hypertension
B. Gastric ulcer
C. Duodenal ulcer
D. Drug-induced GI injury
Ans: A

139. After removal of the parotid gland, patient is having sweating on cheeks while eating.
Auriculotemporalnerve which contains parasympathetic secretomotor fibers to parotid gland is
fused with
A. Greater petrosal Nerve
B. Facial Nerve
C. Greater Auricular Nerve
D. Buccal Nerve
E. Glossopharynalgeal Nerve
Ans: None or B

140. A patient of Crohn’s Disease, underwent resection anastomosis. Now presents on 7 th post-
op day withanastomotic site leak from a fistula. Everyday leakage volume adds up to 150-200
ml. There is no intra-abdominal collection and the patient is stable without any complains. What
will be the next line of management?
A. Do conservative treatment and leave hi hope for the spontaneous resolution
B. Perform Laparotomy and check for leakage site and healthy margins
C. Perform Laparotomy and completely exteriorize the fistula
D. Perform Laparotomy and place lateral drains and leave

www.EduMedWeb.com www.EduMedWeb.com www.EduMedWeb.com


www.EduMedWeb.com www.EduMedWeb.com Page 24

Ans: A

141. A lady has dysphagia, intermittent epigastric pain. On endoscopy, Esophagus was dilated
above and narrow at the bottom. Treatment is
A. Heller’s Cardiomyotomy
B. Esophagectomy
C. Dilatation
D. PPIs
Ans: A

142. Which jaw cyst is pre-malignant?


A. Radicular Cyst
B. Dental Cyst
C. Odontogenic kerato Cyst
D. Dentigerous Cyst
Ans: D

143. After doing a Thoracoabdominal graft, a person developed weakness in legs. It is due to
A. Lumbosacral nerve injury
B. Thoracic splanchnic nerve injury
C. Decreased blood supply to lower limbs
D. Discontinuation of Arteria Radicularis Magna

Ans: A or D
144. True about Epigastric Hernia is
A. Located below the umbilicus and always in the midline
B. Located above the umbilicus and always in the midline
C. Located above the midline and on either side
D. Can be seen anywhere on abdomen
Ans: B

145. External Hemorrhoids, distal to dentate line is


A. Painful
B. Skin Tag is same
D. Should be ligated
Ans: A

146. Early complication of lieostomy


A. Diarrhoea
B. Obstruction
C. Necrosis
D. Prolapse
Ans: A

www.EduMedWeb.com www.EduMedWeb.com www.EduMedWeb.com


www.EduMedWeb.com www.EduMedWeb.com Page 25

147. Ca Breast Stage 4b involves all EXCEPT


A. Nipple Retraction
B. Skin ulcer over the swelling
C. Dermal Edema
D. Satellite Nodule
Ans: A

148. Due to advances in cancer treatment the prognosis of which of the following has become
better?
A. Glioblastoma Multiforme
B. Esophageal Carcinoma
C. All in children
D. Cholangio Carcinoma
Ans: C

OBSTETRICS & GYNECOLOGY


149. After full term normal vaginal delivery, lady goes to sudden collapse. What should be
suspected?
A. PPH
B. Amniotic fluid embolism
C. Uterine Inversion
D. Eclampsia
Ans: C

150. A young lady has history of recurrent abortion. Which test should be done?
A. Russel Viper Venom Test
B. Prothrombin Time
C. Bleeding time
D. Clotting Time
Ans: A

151. A 20 year old average weight female complains of oligomenorrhoea along with facial hair.
Preliminary investigation reveals raised free testosterone levels. On Ultrasound, ovaries were
normal. Which of the following could be likely etiology?
A. Idiopathic Hirsutism
B. PCOD
C. Adrenal Hyperplasia
D. Testosterone secreting tumor
Ans: C

152. Which test is used to differentiate maternal and fetal blood cells?
A. Apt Test
B. Kleihauer-Betke Test
C. Bubblin Test

www.EduMedWeb.com www.EduMedWeb.com www.EduMedWeb.com


www.EduMedWeb.com www.EduMedWeb.com Page 26

D. Osmotic Fragility Test


Ans: A

153. Lymph nodes negative with positive peritoneal cytology is staged as-
A. Stage IIIA
B. Stage IIIB
C. Stage IIIC1
D. Stage IIIC2
Ans: B

154. Treatment of Ca Cervix IIIB include


A. Wertheims Hysterectomy
B. Schauta’s Hysterectomy
C. Chemotherapy
D. Intracavity Brachytherapy followed by External Beam Radiotherapy
Ans: D

155. At which time in the pregnancy, woman has maximum cardiac strain?
A. 2nd Trimester
B. At the time of delivery
C. Immediately following delivery
Ans: C

156. A Pregnant female feeling dizziness in supine position; feels on lateral position and after
getting up. This must be due to
A. IVC Compression
B. Raised intra-abdominal pressure
C. Decreased blood flow to placenta
D. Compression of Sympathetic Chain
Ans: A

157. Awoman presented with carcinoma cervix FIGO STAGING II-III, which of the following
treatment is recommended?
A. Trichelectomy
B. Radiotherapy plus HPV vaccine
C. Chemo-radiotherapy
D. Hysterectomy

PEDIATRICS
158. Which of the following will favor the diagnosis of RDS in new born?
A. Receipt of antenatal steroids
B. Air bronchogram in chest X-ray
C. Manifests after 6 hours
D. Occurs after term gestation

www.EduMedWeb.com www.EduMedWeb.com www.EduMedWeb.com


www.EduMedWeb.com www.EduMedWeb.com Page 27

Ans: B

159. A neonate is diagnosed with Bell’s Stage 1 Necrotizing and he is stable. Treatment is
A. Conservative and Antibiotics
B. IV Antibiotics and Removal of necrosed portion of Intestine
C. Drainage
D. Resection Anastomosis
Ans: A

160. A Toddler has few drop of blood coming out of rectum. Probable diagnosis is
A. Juvenile Rectal Polyp
B. Adenoid Poliposis Coli
C. Rectal Ulcer
D. Piles
Ans: A

161. In Juvenile Myoclonic Epilepsy, most common presentation is (Twisted Repeat)


A. Absent Seizure
B. Myoclonus
C. Generalized Tonic-Clonic Seizure on going to sleep
D. Generalized Tonic-Clonic Seizure on going on awakening
Ans: B

162. A child was taken for CECT Chest and contrast was injected; child had swelling which
gradually increased. There is numbness. There is pain on passive extension of fingers. He is
not allowing you to touch the arm. Pluse was present. What will you do?
A. High Dose Prednisolone
B. Arterial Thrombectomy
C. Immediate Fasciotomy
D. Antihistaminics / Anticoagulants
Ans: A

163. Recurrent URTI in 5 year old child with ear problems, mouth breathing. Treatment is
A. Myringotomy
B. Myringotomy with Grommet insertion
C. Adenoidectomy with Grommet insertion
D. Tonsillectomy
Ans: C

164. A child with recurrent seizures, palpable plaques in the Ophthalmic and Maxillary
distribution. Mother complaints, it is present since birth and not changed since then. Diagnosis
is
A. Congenital Hemangioma
B. Sturge-Weber Syndrome

www.EduMedWeb.com www.EduMedWeb.com www.EduMedWeb.com


www.EduMedWeb.com www.EduMedWeb.com Page 28

C. Infantile Hemangioma
D. Tuberous Sclerosis
Ans: B

SKIN
165. Which of the following is characteristic of Basal Cell Carcinoma?
A. Four Cells
B. Nuclear Palisades
C. Keratin Pearls
Ans: B

165. A 24 year old man had multiple small hypopigmented patches around hair follicles on
upper chest and back. Macules were with fine scaling which coalesced later. Patient has
received treatment before 1 year andit was treatted. Investigation to be done is
A. KOH preparation
B. Tzank test
C. Slit Skin Smear
D. Skin biopsy of coalesced lesion
Ans: A

167. A child with itchy erythematous lesion on face and palm & soles. Possible diagnosis is
A. Scabies
Ans: A

168. A patient has multiple hypoaesthetic and hypopigmented patches on lateral aspect of
forearm. Abundance of acid fast bacilli and Granulomatous inflammation is seen. Diagnosis is
A. Lepromatous Leprosy
B. Tuberculoid Leprosy
C. Indeterminate Leprosy
D. Borderline Leprosy
Ans: D

169. Characteristic of Borderline Leprosy is


A. Inverted Saucer Appearance
B. Erythema Nodosum Leprosum
C. Hypopigmented macules/plaques all over the body
D. Glove and Stocking Anesthesia
Ans: A

170. A patient has 2 finger nails and 1 toenail tunneling since 1 year. Rapid Diagnosis is done
by
A. Woods Lamp Examination
B. KOH mount
C. Slit Smear Examination

www.EduMedWeb.com www.EduMedWeb.com www.EduMedWeb.com


www.EduMedWeb.com www.EduMedWeb.com Page 29

Ans: A

171. Aboy had itchy, excoriated papules on the forehead and the exposed parts of the arms and
legs since 3 years. The disease was most severe in rainy season and improved completely in
winter. The most likely diagnosis is (May 2004)
A. Insect Bite Hypersensitivity
B. Scabies
C. Urticaria
D. Atopic Dermatits
Ans: A

172. Which of the following is related to sunlight exposure?


A. Actinic Keratosis
Ans: A

173. A 23 year lady develops brownish pigmentation on cheeks and bridge of nose on exposure
to sun. Diagnosis is
A. Photodermatitis
B. SLE
C. Chloasma
D. Rosacea

ANESTHESIA
174. Maximum dose of Lignocaine given with Adrenaline for Occular Surgery
A. 3 mg/kg
B. 5 mg/kg
C. 7 mg/kg
D. 10 mg/kg
Ans: C

175. Lady with a history of previous C Section told the anesthetist that in spite of spinal
anesthesia, she wasaware of the procedure. What should be done to monitor depth of
anesthesia?
A. Pulse Oximetry
B. Train of Four
C. Bispectral Imaging
D. End Tidal CO2
Ans: C

176. During ET tube Intubation, by right handed person, Laryngoscope is held by which hand?
A. Left
B. Right
C. Both
Ans: A

www.EduMedWeb.com www.EduMedWeb.com www.EduMedWeb.com


www.EduMedWeb.com www.EduMedWeb.com Page 30

177. Patient with ruptured spleen is taken for laparotomy. BP is 80/50 and HR is 125/min.
Induction agent of choice
A. Sodium Thiopentone
B. Opioids
C. Ketamine
D. Halothane
Ans: C

178. A 25 year old primigravida has Mitral Stenosis and Regurgitation and is under labor. She
says she Wantsnatural delivery. Which would be the best way to provide analgesia in the
woman?
A. Neuraxial Anesthesia
B. Spinal Anesthesia
C. Inhalational Analgesia
D. Intravenous Opioids
Ans: C

179. A patient has severe Mitral Stenosis. Anesthetic agent is


A. Etomidate
B. Propramide
C. Morphine
D. Remifentanyl
Ans: A

180. All are true about “Scoline Asphyxia” EXCEPT


A. It can be inherited
B. It is due to deficiency of Cholinesterase
C. Succinylcholine can produce it
D. Patient can be saved if properly treated
Ans: B

181.Component of Sodalime for CO2 absorption in closed circuit in anesthesia is


A. NaOH
B. Ca(OH)2
C. KOH
D. Ba(OH)2
Ans: B

182. In Pregnant female, ther is decreased requirement of the anesthetic agent because of all of
the following EXCEPT
A. Exaggerated Lumbar Lordosis
B. Increased Congestion in Spinal Veins
C. Decreased Subarachnoid space

www.EduMedWeb.com www.EduMedWeb.com www.EduMedWeb.com


www.EduMedWeb.com www.EduMedWeb.com Page 31

D. Increased Sensitivity of Nerves


Ans: A or C

RADIOLOGY
183. Amifostine is
A. Radio-sensitizer
B. Radio-protector
C. Radio-modifier
D. Radio-mimetic
Ans: B

184. “Bracket Calcification” on Skull X-Ray is seen in


A. Tuberous Sclerosis
B. Sturge-Weber Syndrome
C. Lipoma of Corpus Callosum
D. Meningioma
Ans: C

185. Maximum Radiation exposure occurs in


A. Bone Scan
B. CT Scan
C. X-Ray
D. MRI
Ans: B

186. Non-iodine containing Contrast is


A. Gadolinium
B. Visipaque
C. Iohexanol
D. Diatrozoate
Ans: A

187. Stereotactic Radiotherapy is used in


A. Miliary Lung Metastasis
B. Inoperable Stage 1 Lung Tumor
C. Lymphangitis Carcinomatosa
D. Tumor at the base of tongue with Lymph Node enlargement
Ans: B

PSYCHIATRY
188. Which of the following drug is a available as depot preparation?
A. Fluphenazine
B. Aripriprazole

www.EduMedWeb.com www.EduMedWeb.com www.EduMedWeb.com


www.EduMedWeb.com www.EduMedWeb.com Page 32

C. Ziprosidone
D. Trifluperzine
Ans: A

189. All of the following are disorders of “form of thought” EXCEPT


A. Loosening of Association
B. Tangentially
C. Thought Block
D. Circumferentiality / Derealisation
Ans: C

190. PTSD is differentiated from all other disorders by


A. Recall of events and avoidance of trauma
B. Nightmares about events
C. Autonomic arousal and anxiety
D. Depression
Ans: A

191. Brain is considered responsive because


A. It has control centers for all motor actions and sensory inputs
B. It can be profoundly affected by external environment
C. Every cell of brain has specifiic function
D. It acts as a closed system, responding to internal stimuli but not the external stimuli
Ans: B

ORTHOPEDICS
192. Gallow’s Traction is used for
A. Shaft of Femur
B. Neck of Femur
C. Shaft Tibia
D. Tibial Tuberosity
Ans: A

193. A patient met with Road Traffic Accident with injury to left knee. Dial test was positive.
What could be the cause?
A. Medical Collateral Ligament Injury
B. Posterolateral Corner Injury
C. Lateral Meniscus Tear
D. Medial Meniscus Injury
Ans: B

194. After lifting something heavy from ground, a patient complaints of back pain, which is
radiating to lateralleg and great toe of lower limb. Most probable diagnosis would be
A. L5-S1 disc prolapse

www.EduMedWeb.com www.EduMedWeb.com www.EduMedWeb.com


www.EduMedWeb.com www.EduMedWeb.com Page 33

B. L4-L5 disc prolapse


C. L3-L4 disc prolapse
D. L5 fracture
Ans: B

195. A person is able to abduct his arm, internally rotate it, place the back of hand
onlumbosacral joint, but is not able to lift it from back. What is etiology?
A. Subscapularis tendon tear
B. Teres major tendon tear
C. Long head of biceps tendon tear
D. Acromioclavicular joint dislocation
Ans: A

196. A new born presents with inverted foot and the dorsum of the foot cannot touch the anterior
tibia. The most probable diagnosis is
A. Congenital Vertical Talus
B. Arthrogryposis Multiplex
C. CTEV
D. Flat Foot
Ans: A

197. 8 year old child has fever with pain and swelling in mid-thigh. Codman’striangle is seen on
X-Ray. Histopathology shows small round cell positive for MIC-2. Diagnosis is
A. Osteosarcoma
B. Ewing’s Sarcoma
Ans: B

198. A patient comes to the emergency department after alcohol binge previous night and
sleeping on armchair. In the morning he is unable to move his hand and diagnosis of ulnar
nerve palsy is made. What is the next line of management?
A. Wait and Watch
B. Knuckle Bender Splint
C. Immediately operate and explore the nerve
D. Do EMG study after 2 days
Ans: D

199. An old lady had a history of fall in bathroom once couldn’t move. Afterwards, she had legs
inexternally rotated position. There was tenderness in Scarpa’s triangle and limb movement
could not been done due to pain. No hip fracture was seen on X-Ray. Next step?
A.MRI
B. Repeat X-Ray after one week
C. Joint Aspiration
D. Give analgesic and Manipulate
Ans: A

www.EduMedWeb.com www.EduMedWeb.com www.EduMedWeb.com


www.EduMedWeb.com www.EduMedWeb.com Page 1

AIIMS PG MAY 2014

ANATOMY

1. All of the following are derived from ectoderm except


A. Hair follicles
B. Erector pilae
C. Sebaceous glands
D. Mammary glands

2. Which of the following muscles is not a derivative of mesoderm?


A. Skeletal muscles
B. Smooth muscles
C. Cardiac myocyte
D. Dilators of pupil

3. Buccopharyngeal membrane develops from:


A. Ectoderm + Mesoderm
B. Ectoderm + Endoderm
C. Mesoderm + Endoderm
D. Ectoderm + Mesoderm + Endoderm

4. Epithelioid cells include Which of the following?


A. Islet cells of pancreas
B. Theca Iutein cells
C. Cells of Brunner’s gland
D. Interstitial cells of Leydig

5. Venous drainage of esophagus is into the


A. Azygous and inferior thyroid veins
B. Azygous, inferior thyroid and left gastric veins
C. Azygous, inferior thyroid and right gastric veins
D. Superior thyroid, , inferior thyroid veins, azygous and hemi- azygous veins

6. The most common variant in the blood supply of colon is


A. Absent right colic artery
B. Absent middle colic artery
C. Absent left colic artery
D. Absent superior rectal artery

7. All of the following are part of mesorectal fascia except


A. Superior rectal vein
B. Inferior rectal vein
C. Para rectal nodes
D. Inferior mesenteric plexus

8. Which of the following is not true about blood test is barrier?


A. It is formed by Sertoli cells
B. It is formed before primary spermatogenesis

www.EduMedWeb.com www.EduMedWeb.com www.EduMedWeb.com


www.EduMedWeb.com www.EduMedWeb.com Page 2

C. Any rupture of barrier can cause immune response to germ cells


D. Germ cells are not necessary for formation of the barrier

9. Which of the following is not true about duar matter?


A. It is the outermost of the cranial meninges
B. It has two layers-periosteal and meningeal
C. Dural venous sinuses lie below the meningeal layer
D. It is supplied by a branch of the V nerve

10. Which of the following structures is seen in the cavernous sinus?


A. 1ST part of V nerve
B. 2nd part of V nerve
C. ICA
D. 4th nerve

11. The medulla receives its blood supply from all of the following arteries except
A. Anterior spinal
B. Posterior inferior cerebellar
C. Vertebral
D. Superior cerebellar

12. Which of the following is not supplied by ipsilateral nerve nucleus?


A. Superior rectus
B. Inferior Superior
C. Medial Superior
D. Inferior oblique

13. Trigeminal nerve supplies all of the following muscles except


A. Stylohyoid
B. Medial pterygoid
C. Lateral pterygoid
D. Tensor veli palatine

14. Facial nerve gives out secretomotor fibres to all of the following except
A. Lacrimal gland
B. Parotid gland
C. Submandibular gland
D. Nasal glands

PHYSIOLOGY

15. In an encephale is ole preparation, the transsaction is done at:


A. First cervical spinal segment
B. Level of medulla
C. Midpontine level
D. Midcollicular level

16. If a supramaximal stimulus is applied to an excitable tissue like a nerve or a muscle and it
elicits a response, then the tissue is said to be in
A. Absolute refractory period
B. Relative refractory period

www.EduMedWeb.com www.EduMedWeb.com www.EduMedWeb.com


www.EduMedWeb.com www.EduMedWeb.com Page 3

C. Latent period
D. After-depolarization

17. Nuclear bag fibres are related to


A. Force
B. Length
C. Tone
D. Length and velocity

18. True about Golgi tendon organ is


A. Sence the dynamic length of the muscle
B. Are involved in reciprocal innervations
C. Are stimulated by alpha motor neuron
D. Sense muscle tension

19. The mechanostretch receptors joints and ligaments are


A. Fast adapting
B. Slow adapting
C. Non adapting
D. Adapt ifferentially for different stresses

20. During hip replacement, loss of joint and ligament receptors leads to
A. No loss of sensation
B. Loss of all sensations
C. Joint sensation lost in stable position
D. Decrease in movements and decrease in sensation

21. Fast fatigue fibres are recuited during walking


A. In the beginning
B. In the end
C. Throughout the walking process
D. When small neurons are excited

22. Which of the following can cause a shift of the oxygen-hemoglobin dissociation curve to the
left?
A. pH
B. Temperature
C. 2,3-BPG
D. pCO2

23. In the given circuit, the inward flow pressure is 100 mm Hg and the outward flow pressure is
10 mm Hg/mL/min. Calculate the flow across the circuit
A. 45 mL
B. 90 mL
C. 3.6 mL
D. 135 mL

24. Which of the following defines the pressure in the vascular system in the absence of blood
flow?
A. Pulse pressure
B. Critical closing pressure

www.EduMedWeb.com www.EduMedWeb.com www.EduMedWeb.com


www.EduMedWeb.com www.EduMedWeb.com Page 4

C. Mean circulatory pressure


D. Perfusion pressure

25. Which of the following defines the pressure in the vascular system in the absence of blood
flow?
A. Pulse pressure
B. Critical closing pressure
C. Mean circulatory pressure
D. Perfusion pressure

26. Which of the following is true about measurment of blood pressure?


A. The length of the bladder in the blood pressure cuff should cover more than 80% of the arm
area
B. The person should be comfortably sitting and blood pressure cuff should be at higher level
than the heart
C. Caffeine intake induces decrease in blood pressure
D. Blood pressure increases during sleep.

27. The left atrial ‘v’ wave is larger than ‘a’ wave in comparison to right atrium because
A. Left side is high pressure system
B. Left atrium is more compliant
C. Left atrium is compressed posteriorly by pulmonary veins
D. Right atrial pressure is transmitted into the pulmonary circulation

28. The right atrium chronic overload is indicated by a P wave of more than
A. 2.5 mm
B. 3.5 mm
C. 4.5 mm
D. 5.5 mm

29. The cells of the human body most sensitive to ischemia are
A. Neurons
B. Nephrons
C. Cardiac myocyte
D. Hepatocytes

30. The most important hormone that increases gallbladder contraction after a fatty meal is
A. Gastrin
B. Secretin
C. CCK
D. GIP

31. The most serious complication of prolonged sitting position is


A. Venous air embolism
B. Dysrhythmias
C. Hypotension
D. Nerve palsies

BIOCHEMISTRY

www.EduMedWeb.com www.EduMedWeb.com www.EduMedWeb.com


www.EduMedWeb.com www.EduMedWeb.com Page 5

32. Not an intermediate product of citric acid cycle is


A. Acyl CoA
B. Succinyl CoA
C. Citrate
D. α – ketoglutarate

33. The enzyme common to both glycogenolysis and glycogenesis is


A. Glycogen phosphorylase
B. Glycogen synthase
C. Glucotransferase
D. Carboxykinase

34. In the entire human genome, coding DNA constitutes


A. 0.02
B. 0.1
C. 0.25
D. 0.4

35. Methylation of cytosine leads to


A. Increased expression of gene
B. Decreased expression of gene
C. No effect on gene expression
D. Mutation

36. 2,3 – BPG binds to --- sites of hemoglobin and ------ the affinity for oxygen
A. 4, decreases
B. 1, decreases
C. 4, increases
D. 1, increases

37. The last receiver of electrons in electron transport system is


A. Coenzyme Q
B. FADH2
C. O2
D. Cytochrome b

PHARMACOLOGY

38. Which of the following is not converted into an active metabolite?


A. Lisinopril
B. Fluxetine
C. Cyclophosphamide
D. Diazepam

39. Methacholine is a
A. M1 receptor agonist
B. M2 receptor agonist
C. M3 receptor agonist
D. M4 receptor agonist

40. α2 agonists cause all of the following except

www.EduMedWeb.com www.EduMedWeb.com www.EduMedWeb.com


www.EduMedWeb.com www.EduMedWeb.com Page 6

A. Analgesia
B. Hyperalgesia
C. Sedation
D. Anxiolysis

41. Which of the following is a mixed alpha and beta agonist?


A. Dobutamine
B. Fenoldopam
C. Epinephrine
D. Phenylephrine

42. Peripheral vasospasm is observed with which of the following anti-Parkinsonian drugs?
A. Ropinirole
B. Levodopa
C. Bromocriptine
D. Entacapone

43. Peripheral vasospasm is observed with which of the following anti-Parkinsonian drugs?
A. Ropinirole
B. Levodopa
C. Bromocriptine
D. Entacapone

44. Lithium potentiates the action of non-depolarizing muscle relaxants. How many days before
administration of the muscle relaxants should lithium be stopped?
A. 1 day
B. 2 days
C. 3 days
D. 4 days

45. True statement regarding methadone are all except


A. It is a long acting µ receptor agonist
B. It is rapidly absorbed from the gastrointestinal tract and is detected in plasma 30 minutes
after oral administration
C. The primary use of methadone is relief of chronic pain
D. The onset of analgesia is 30-60 minutes after parenteral administration and 1-2 hours after
oral administration

46. The site of action of the loop diuretic furosemide is


A. Thick ascending limb of loop of Henle
B. Descending limb of loop of Henle
C. Proximal tubule
D. Distal tubule

47. Which of the following pairs of drug and its indication is matched incorrectly?
A. Carbamazepine
B. Octreotide – treatment of diarrhea associated with vasoactive intestinal peptide tumours
C. Desmopressin-treatment of diabetes insipidus
D. hCG-treatment of infertility in men and women

48. Time dependent killing and prolonged post-antibiotic effect is seen with

www.EduMedWeb.com www.EduMedWeb.com www.EduMedWeb.com


www.EduMedWeb.com www.EduMedWeb.com Page 7

A. Fluoroquinolones
B. Beta lactam antibiotics
C. Clindamycin
D. Erythromycin

49. Non – nucleoside reverse transcriptase inhibitors (NNRTIs) include all of the following
except
A. Nevirapine
B. Delavirdine
C. Etavirine
D. Lamivudine

50. Which of the following antitubercular drugs is associated with hypothyroidism?


A. Rifampicin
B. PZA
C. Etionamide
D. Streptomycin

51. The most common side effect of cancer chemotherapy is nausea with or without vomiting.
The anticancer drugs vary in their ability to cause nausea and vomiting. Which of the following
anti-cancer drugs is least likely to cause nausea and vomiting?
A. Chlorambucil
B. Cisplatin
C. Doxorubicin
D. Daunorubicin

52. Which of the following medications is essential for ameliorating the toxicity of pemetrexed?
A. Folinic acid and vitamin B6
B. Folic acid and vitamin B12
C. Vitamin B6 and vitamin B12
D. Folic acid and dexamethasone

53. Which of the following drugs is useful for the treatment of advanced prostate cancer?
A. Ganirelix
B. Cetrorelix
C. Abarelix
D. Goserelin

PATHOLOGY

54. A 50-year old patient presents with weakness and bleeding episodes. His hemoglobin was
10.5 g/dL. His leukocyte count was 48*109 cells/L and platelet count was 50*109 cells/L. There
was dysplasia of neutrophilia with a sifferential count showing 76% neutrophils, 8% blast cells,
12% myelocytes and metamyelocytes and 4% other cells. Bone marrow examination showed
14% blasts. Cytogenetics showed t(8,21). The most likely diagnosis is
A. Acute myeloid leukemia
B. Chronic myeloid leukemia
C. Chronic lymphoid leukemia
D. Myelodysplastic syndrome

55. Which of the following regarding cellular events in acute inflammation is not correct?

www.EduMedWeb.com www.EduMedWeb.com www.EduMedWeb.com


www.EduMedWeb.com www.EduMedWeb.com Page 8

A. PECAM/CD31 is responsible for neutrophil activation


B. Components of complement can assist in chemotaxis
C. Neutrophil margination is assisted by selectins
D. ICAM-1/VCAM-1 is responsible for neutrophil adhesion

56. Which of the following factors in morphogenic as well as mitogenic?


A. Fibroblast growth factor
B. Platelet derived growth factor
C. Bone morphogenetic protein
D. Insulin-like growth factor

57. Which of the following is not an example of a syndrome caused by uniparental disomy?
A. Prader-Will syndrome
B. Angelman syndrome
C. Russell-Silver syndrome
D. Bloom syndrome

58. Anti-neutrophil cytoplasmic antibodies (ANCA) is seen in


A. Wegener’s granulomatosis
B. Diabetes mellitus
C. Rheumatoid arthritis

59. ARDS is due to a defect in


A. Type 1 pneumocytes
B. Type 2 pneumocytes
C. Clara cells
D. Endothelial cells

60. A 50 –year old male presents with large bowel type diarrhea and rectal bleeding. On
sigmoidoscopy, a cauliflower like growth is seen in the rectum. Colectomy specimen on
histopathology shows adenocarcinoma. Which of the following prognostic investigations is not
required?
A. Microsatellite instability
B. c-myc mutations
C. EGFR mutations
D. k-RAS mutations

61. Verocay bodies are see in


A. Meningioma
Hemamgioma
C. Glioma
D. Schwannoma

62. Histopathologically, rosettes are not seen in


A. Retinoblastoma
B. Neurocysticercosis
C. PNET
D. Medulloblastoma

MICROBIOLOGY

www.EduMedWeb.com www.EduMedWeb.com www.EduMedWeb.com


www.EduMedWeb.com www.EduMedWeb.com Page 9

63. Which of the following is correct regarding staining of microbes?


A. Gram positive bacteria stain black
B. Gram negative bacteria stain red
C. Gomorri’s methenamine silver stains fungi green
D. Calcoflour white stains acanthamoeba red

64. Both DNA and RNA are present in


A. Bacteria
B. Prions
C. Virioids
D. Plasmid

65. Which of the following is not true regarding IgE antibodies


A. It mediates release of histamine and other chemical mediators
B. It is the primary antibody involved in allergic reactions
C. It is involved in anti-parasitic immune responses
D. May cross the placenta and fix complement

66. A 10-year old child presented with severe sore throat and a grey pseudomembrane over
pharynx and tonsils. The organism causing this infection is most likely
A. A gram positive bacillus
B. A gram negative bacillus
C. A Catalase-positive gram positive coccus that grows in clusters
D. A single-standed positive sense RNA virus

67. The hematoxylin and eosin staining of a lung section during autopsy of a patient suffering
from AIDS shows desquamation of type 1 pneumocytes with prominent intranuclear basophilic
inclusion bodies surrounded by a clear halo. The CD4 count was less than 100/mm3 before
demise. The most likely diagnosis causing these features is
A. Acute respiratory distress syndrome
B. Pneumocystis jiroveci pneumonia
C. CMV pneumonia
D. Mycobacterium avium-intracellulare pneumonia

68. Giemsa stained smear of epithelial cells obtained from a newborn with hepatosplenomegaly
is shown below. What is the most likely cause of this congenital infection?
A. CMV
B. Rubella
C. HIV
D. Ebstein-Barr virus

69. An elderly male presents 2 months after renal transplantation with nephropathy. Which of
the following can be a viral etiological agent?
A. Polyoma virus BK
B. Human herpes virus type 6
C. Hepatitis C
D. Human papilloma virus, high risk types

70. The causative organism for AIDS was identified in the year
A. 1983
B. 1976

www.EduMedWeb.com www.EduMedWeb.com www.EduMedWeb.com


www.EduMedWeb.com www.EduMedWeb.com Page 10

C. 1994
D. 1969

71. The term recrudescence in malaria refers to


A. Same as relapse in P. vivax and p. ovale malaria
B. Resistance to antimalarial drugs
C. Recurrence of sexual parasitemia after completion of treatment
D. Reinfection with the same species

FORENSIC MEDICINE AND TOXICOLOGY

72. The legal responsibilities of an intoxicated person is given in section ----- IPC
A. 82
B. 83
C. 84
D. 85

73. A person is liable for punishment for perjury which is defined under?
A. Section 190 IPC
B. Section 191 IPC
C. Section 192 IPC
D. Section 193 IPC

74. The most important sign of age determination as per Gustafson’s method is
A. Root transparency
B. Root attrition
C. Root resorption
D. Paradentosis

75. A person of eonism derives pleasure from


A. Wearing clothes of opposite sex
B. Fondling female body parts
C. Rubbing genitalia against body of other person
D. Seeing the opposite partner nude

76. Cannabis is the most commonly used illicit drug in India. Which of the following is not
derived from cannabis?
A. Bhang
B. Charas
C. Ganja
D. Afeem

77. The characteristic burnt odour is caused by toxin of


A. Cannabis
B. Tobacco
C. Strychnine
D. Chloral hydrate

78. The active principle of white oleander is


A. Nicotine
B. Nerin

www.EduMedWeb.com www.EduMedWeb.com www.EduMedWeb.com


www.EduMedWeb.com www.EduMedWeb.com Page 11

C. Abrine
D. Pilocarpine

79. A doctor is treating a patient with viper snake bite. He should not forget that viper venom is
A. Histotoxic
B. Vasculotoxic
C. Musculotoxic
D. Neurotoxic

PREVENTIVE AND SOCIAL MEDICINE

80. The prospectively evaluated, double-blinded, randomized clinical trial represents the gold
standard for providing evidence for therapeutic decision making. This was first proposed by the
father of evidence-based medicine:
A. Tostoy
B. Sackett
C. Hippocrates
D. da Vinci

81. True statements regarding the calculation of HDI are all of the following except
A. The minimum value for combined gross enrolment ratio is fixed at 0%
B. The minimum value for adult literacy rate is fixed as 100
C. The minimum value of life expectancy at birth is taken as 100
D. The minimum value for per capita income is fixed at 40,000

82. Regarding education in HDI, true statement is


A. In gross enrolment, only secondary educationis considered, not primary education
B. 1/3rd weightage is given to adult literacy
C. 2/3rd weightage is given to gross enrolment
D. Country’s achievement is computed on the basis of adult literacy

83. Immune thrombocytopenic purpura (ITP) is a complication observed with which of the
following vaccines?
A. MMR
B. HiB conjugate polysaccharide vaccine
C. Typhoid Vi polysaccharide vaccine
D. Influenza vaccine

84. Immune thrombocytopenic purpura (ITP) is a complication observed with which of the
following vaccines?
A. MMR
B. HiB conjugate polysaccharide vaccine
C. Typhoid Vi polysaccharide vaccine
D. Influenza vaccine

85. Chemoprophylaxis is indicated for all of the following except


A. Cholera
B. Measles
C. Meningococcal meningitis
D. Conjunctivitis

www.EduMedWeb.com www.EduMedWeb.com www.EduMedWeb.com


www.EduMedWeb.com www.EduMedWeb.com Page 12

86. All of the following statements regarding INH chemoprophylaxis are correct except
A. It is cheap
B. It is not feasible to apply on a large scale
C. It has risk of drug-induced hepatitis
D. It is not highly clinically effective

87. Positive predictive value is a function sensitivity, specificity and


A. Absolute risk
B. Relative risk
C. Incidence
D. Prevalence

88. All of the following are correct regarding the period of isolation in the following disease
except
A. Chicken pox – 6 days after the onset of rash
B. Measles – s days after the onset of rash
C. German measles – 7 days after the onset of rash
D. Herpes zoster – 6 days after the onset of rash
89. Which of the following regarding oral polio vaccine (OPV) is not true?
A. Useful in epidemics
B. Excreation of virus in stools may cause disease to the unimmunized
C. Rapid antibody response
D. Protective even in the presence of maternal antibodies

90. HIV sentinel surveillance provides data


A. To monitor trends
B. To identify high risk population
C. To estimate disease incidence
D. To assess the quality of related services

91. A boy complained of unprovoked bite by a local dog in the community. The dog was alter
caught by the local animal authorities and appeared to be healthy. The most appropriate caurse
of action would be
A.Give post-exposure prophylaxis to the bitten person with cell-culture derived vaccine
B. Withhold vaccination, keep the dog under observation for ten days for signs of rabies
C. Test the dog for rabies antibodies
D. Immediately carry out euthanasia of the dog

92. Evidence of decreased risk of cardiovascular disease is associated with all of the following
except
A. Vitamin E supplementation
B. Low to moderate daily alcohol consumption
C. Regular physical activity
D. Potassium

93. Under the school vision screening programme, vision screening in schools is done by
A. Ophthalmologist
B. Optometrist
C. NGO worker
D. Teacher

www.EduMedWeb.com www.EduMedWeb.com www.EduMedWeb.com


www.EduMedWeb.com www.EduMedWeb.com Page 13

94. Under the National Rural Health Mission, an ASHA will receive financial remuneration for all
of the following except
A. Institutional deliveries
B. Measuring the birth weight of neonates
C. 1st dose of DPT and OPV
D. Registration of births

95. The provisions under Janani Shishu Suraksha Karyakram (JSSK) include all of the following
except?
A. Free food to the mother in the hospital
B. Cash incentive for institutional deliveries
C. Free transport to the mother and baby to/from hospital
D. Free treatment of the neonate and infant in all public health facilities.

96. Calculate the neonatal mortality rate for a population according to the data of 2012. Number
of neonatal deaths: 450. Number of still births: 212. Total number of live births 12,450
A. 36
B. 15
C. 90
D. 56

97. Infant mortality rate includes all of the following except


A. Early neonatal deaths
B. Late neonatal deaths
C. Post neonatal deaths
D. Still births

98. A girl is having white hair. 40% of her classmates are also suffering from the same problem.
None of the other people from the same village have white hair. Her brother who is staying with
an aunt in the neighbouring village is also normal and does not have white hair. Which of the
following explains the probable cause of her white hair?
A. Environmental
B. Genetic
C. Nutritional

99. Severe Acute Malnutrition (SAM) in India is defined as


A. Very low weight for age (below-3 Z scores of the mean WHO standards for growth), by visible
severe wasting or presence of nutritional edema
B. Very low weight for age (below-3 Z scores of the mean WHO standards for growth), only
C. Visible severe wasting, or presence of nutritional edema
D. Very low weight for height (below-3 Z scores of the mean WHO standards for growth), by
visible severe wasting, or presence of nutritional edema

100. The most sensitive indicator of environmental iodine deficiency is


A. Serum T4 levels
B. Serum T3 levels
C. Urine iodine excretion
D. Neonatal hypothyroidism

101. All of the following can cause epidemic in post-disaster period except
A. Leishmaniasis

www.EduMedWeb.com www.EduMedWeb.com www.EduMedWeb.com


www.EduMedWeb.com www.EduMedWeb.com Page 14

B. Leptospirosis
C. Rickettsiosis
D. Acute respiratory infections

102. The hemoglobin levels of pregnant females in a community was found to have a mean of
10.6 g/dL and a standard deviation of 2 g/dL. What is the minimum hemoglobin level below
which 5% of the pregnant females will have their hemoglobin levels?
A. 7.32
B. 8.64
C. 6.68
D. 8.96

103. Universal Health coverage in India was recommended by which of the followinh
committees?
A. Multipurpose worker committee
B. Health development and planning committee
C. High level expert committee
D.Medical education committee

104. All of the following provisions are included in the primary health care according to the Alma
Ata declaration except
A. Adequate supply of safe drinking water
B. Provision of food supply
C. Provision of free medicines
D. Basic sanitation

OPHTHALMOLOGY

105. The critical angle of cornea-ir interface is


A. 36O
B. 46O
C. 56O
D. 66O

106. Which diagnostic procedure is not done in a dilated pupil?


A. Gonioscopy
B. Laser inferometry
C. Fundus examination
D. Electroretinography

107. A 59-year old male presents with dimness of near vision. On examination, the media was
clear in both the eyes. What would be the next step?
A. Refraction with near add
B. Refraction under atropine
C. Radial keratotomy
D. Cataract surgery

108. Which organismal infection is highly virulent and may cause corneal ulcer perforation within
48 hours?
A. Herpes simplex
B. Pseudomonas

www.EduMedWeb.com www.EduMedWeb.com www.EduMedWeb.com


www.EduMedWeb.com www.EduMedWeb.com Page 15

C. Staphylococci
D. Aspergillus

109. Circumcorneal vascularization is observed in deficiency of


A. Vitamin D
B. Thiamine
C. Riboflavin
D. Biotine

110. A patient gives a history of chuna falling into his eyes. All of the following would be a part of
his immediate management except
A. Irrigation of both eyes with copious amounts of normal saline
B. Frequent sodium citrate drops
C. Thorough slit-lamp examination on presentation
D. Removal of chuna particles by double eversion of upper eyelids

111. Which of the following drugs acts on trabecularmeshwork and affects the aqueous outflow?
A. Timolol
B. Pilocarpine
C. Brimonidine
D. Brinzolamide

112. Ex-press glaucoma implant is made up of


A. Silicone
B. Titanium
C. Gold
D. Stainless steel

113. Ascorbate and α-tocopherol are maintained in a reduced state in the lens by
A. Glucose
B. Glycoprotein
C. Glutathione
D. Fatty acid

114. A 7-year old male child presents with normal vision 6/6 in the right eye and hand
movement perception close to the face in the left eye. On fundoscopy, his right eye was normal
and left eye showed retinal detechment, subretinal yellowish exudates and telangiectatic
vessels. The most likely diagnosis is
A. Coats disease
B. Sympathetic ophthalmitis
C. Familial exudative vitreoretinopathy
D. Retinopathy of permaturity

115. A young male presents with central scotoma in left eye. His right vision showed 6/6 vision.
On examination, in the left eye, there was focal foveal detachment. What would bethe next
step?
A. Examine retrolental cells
B. Inquire about the use of steroids
C. Ask for history of trauma in the other eye

www.EduMedWeb.com www.EduMedWeb.com www.EduMedWeb.com


www.EduMedWeb.com www.EduMedWeb.com Page 16

116. A 28-week baby suffered from respiratory distress syndrome at birth. On day 14 of life, he
developed sepsis. No other morbidity was seen. He should be evaluated for retinopathy of
prematurity at what post natal age?
A. 2 weeks
B. 4 weeks
C. 6 weeks
D. 8 weeks

117. A person is diagnosed to be a diabetic on his 45th birthday. You will recommend a dilated
fundoscopic examinatio
A. Immediately
B. Before his 50th birthday
C. When he turns 50 years of age
D. When he complains of dimness of vision

118. Microaneurysms are the earliest manifestation of diabetic retinopathy. Which of the
following layer is involved in diabetic retinopathy?
A. Outer plexiform layer
B. Inner nuclear layer
C. Layer of rods and cones
D. Retinal pigment epithelium

119. Pneumoretinopexy is an outpatient procedure in which retinal detachment is sealed with air
insufflations. Which of the following gas is used in pneumoretinopexy?
A. Sulfur hexafluoride
B. Carbon dioxide
C. Nitrous oxide
D. Oxygen

120. Cells affected inglaucomatous optic neuropathy are


A. Amacrine cells
B. Bipolar cells
C. Ganglion cells
D. Rods and cones

121. Lipogranulomatous inflammation is seen in


A. Fungal infection
B. Tuberculosis
C. Chalazion
D. Viral infection

EAR, NOSE AND THROAT

122. Which of the following does not show negative Rinne test in the right ear?
A. Sensorineural hearing loss of 45 dB in left ear and normal right ear
B. Profound hearing loss
C. Conductive hearing loss 40 dB in both ears
D. Conductive hearing loss 40 dB in right ear and left ear normal

123. The screening investigation of high risk neonates in ICU for suspected hearing loss is
A. Otoacoustic emissions

www.EduMedWeb.com www.EduMedWeb.com www.EduMedWeb.com


www.EduMedWeb.com www.EduMedWeb.com Page 17

B. Free field audiometry


C. Stapedial reflex testing
D. Pure tone audiometry

124. The wave V in BERA represents activity in which of the following?


A. Lateral lemniscus
B. Superior olivary complex
C. Cochlear nerve
D. Cochlear nucleus

125. Which of the following is not resected in stapedotomy?


A. Anterior crus of stapes
B. Posterior crus of stapes
C. Stapedial ligament
D. Lenticular process of incus

126. Which of the following part of cochlear implant is implanted during surgery?
A. Receiver stimulator
B. Transmitting coil
C. Microphone
D. Speech processor

127. A 75-year old diabetic patient presents with severe ear pain and granulation tissue at
external auditory canal with facial nerve involvement. The most likely diagnosis is:
A. Skull base osteomyelitis

128. A 14-year old boy presents with nasal bleeding. His Hb was found to be 6.4 g/dL and
peripheral smear showed normocytic hypochromic anemia. The most probable diagnosis is
A. Juvenile nasopharyngeal angiofibroma
B. Hemangioma
C. Antrochonal polyp
D. Carcinoma of nasopharynx

129. A patient presents withantrochoanal polyp arising from the medial wall of the maxilla.
Which of the following would be the best management for the patient?
A. FESS with polypectomy
B. Medial maxillectomy (TEMM)
C. Caldwell-Luc procedure
D. Intranasal polypectomy

130. An elderly male presents with T3 No laryngeal carcinoma. What would be the
management?
A. Neo-adjuvant chemotherapy followed by radiotherapy
B. Concurrent chemoradiotherapy
C. Radical radiotherapy followed by chemotherapy
D. Radical radiotherapy without chemotherapy

MEDICINE

131. Which of the following is not a characteristic feature of multiple myeloma?


A. Increased Ig levels in serum

www.EduMedWeb.com www.EduMedWeb.com www.EduMedWeb.com


www.EduMedWeb.com www.EduMedWeb.com Page 18

B. Positive ANA
C. Plasmacytosis
D. M spike on electrophoresis

132. A 60 year old obese male with a history of chronic smoking since childhood presents in CU
with pelvic fracture due to fall from height. On the 4th day of the ICU stay, he developed sudden
tachypnoea, fall in SpO2 and hypotension. On 2D echo, there was a dilation of right ventricle
and tricuspid regurgitation. What would be the next immediate step?
A. D-dimer assay
B. Administration of heparin
C. Pericardiocentesis
D. Systemic thrombolysis

133. A 4-year old girl presents with severe vomiting after a viral fever of 5 days. She is
hospitalized and develops cerebral edema. Liver biopsy is most likely to demonstrate which of
the following?
A. Centrizonal hemorrhagic necrosis
B. Non-alcoholic steatohepatitis
C. Ring granulomas
D. Marked microvesicular steatosis

134. A 40-year old obese female presents with fullnes of right upper quadrant of abdomen. Her
medical history is significant for type 2 DM and hyperlipidemia. Liver biopsy is most likely
suggestive of which of the following diagnoses?
A. Non-alcoholic steatohepatitis
B. Peliosis hepatis
C. Autoimmune hepatis
D. Primary biliary sclerosis

135. A young female patient presents with jaundice and elevated liver enzymes. She has a
history of 2 similar episodes in the past. Her serum ANA was 1:40 and IgG was 2400 IU. His
serum copper levels were normal and viral markers were negative. Aliver biopsy was carried out
and based on the pathology report, you treated her with immunosuppressants and the patient’s
condition improved. What was the most likely finding on pathology report to suggest the
treatment?
A. Non-alcoholic stestohepatitis
B. Hemosiderosis
C. Autoimmune hepatitis
D. Primary biliary cirrhosis

136. Laxative abuse is associated with


A. Hypokalemia
B. Hypomagnesemia
C. Hypoglycemia
D. Colonic spasticity

137. A 27-year old male presents with low backache, that occurs early in the morning,
associated with stiffness, and persists for more than 30 minutes. On examination, his chest
expansion is also restricted. The most probable diagnosis is
A. Rheumatoid arthritis
B. Osteoarthritis

www.EduMedWeb.com www.EduMedWeb.com www.EduMedWeb.com


www.EduMedWeb.com www.EduMedWeb.com Page 19

C. Gouty arthritis
D. Ankylosing spondylitis

138. One of the following is not a diagnostic criteria for declaring brainstem death
A. Absence of brainstem reflexes
B. A positive apnea test
C. Lack of cerebromotor response to pain in all extremities
D. Absence of stretch reflex all extremities is essential

139. A vascular lesion in the mid-brainstem level in the area depicted in the figure can cause all
of the following except
A. Deviation of the tongue to the same side
B. Contralateral loss of pain and temperature below the neck
C. Nystagmus
D. Paralysis of soft palate

140. Two persons A and B were told to draw a square from the blank and filled circle,
respecitively. The square drawn by A is irregular. What could be the possible cause?
A. Lesion in basal ganglia
B. Lesion in subthalamic nucleus
C. Lesion in cerebellum
D. Physiological tremors

141. One of the following drugs is used in acute bacterial meningitis


A. Erythromycin
B. Sulfamethoxazole
C. Ceftriaxone
D. Streptomycin

SURGERY

142. The first prioriti in management of a case of head injury with open fracture of shaft of femur
is
A. Neurosurgery consultation
B. Give IV fluids
C. Intubation
D. Splintage of fracture

143. Neurogenic shock is characterized by


A. Hypertension and tachycardia
B. Hypertension and bradycardia
C. Hypotension and tachycardia
D. Hypotension and bradycardia

144. During resuscitation, artefats of fractured ribs most commonly involve


A. 2nd – 4th ribs
B. 3rd – 5th ribs
C. 4th – 6th ribs
D. 5th – 7th ribs

www.EduMedWeb.com www.EduMedWeb.com www.EduMedWeb.com


www.EduMedWeb.com www.EduMedWeb.com Page 20

145. A 12-year old girl presents with nodular goiter. Which of the following statements regarding
her evaluation and management incorrect?
A. 99 m-Tc scan should be performed to determine whether the nodules are hypofunctioning or
hyperfunctioning
B. Functional thyroid nodules are ususally benign
C. All nodules > 4cm should be resected irrespective of cytology
D. FNAC should be performed for all nodules > 1 cm in diameter

146. Which of the following incisions is taken for diaphragmatic surgery?


A. Transverse
B. Circumferential
C. Vertical
D. Radial

147. Which of the following is not an etiological factor for pancreatitis?


A. Abdominal trauma
B. Hyperlipidemia
C. Islet cell hyperplasia
D. Germline mutations in the cationic trypsinogen gene

OBSTETRICS AND GYNECOLOGY

148. All of the following indicate superimposed pre-eclampsia in a pregnant female of chronic
hypertension except
A. New onset proteinuria
B. Platelet count < 75,000
C. Increase in systolic BP by 30 mm Hg and diastolic by 15 mm Hg
D. Fresh retinal hypertensive changes

149. All of the following indicate superimposed pre-eclampsia in a pregnant female of chronic
hypertension except
A. New onset proteinuria
B. Platelet count < 75,000
C. Increase in systolic BP by 30 mm Hg and diastolic by 15 mm Hg
D. Fresh retinal hypertensive changes

150. Which of the following antihypertensives is not given in pregnancy?


A. Enalapril
B. α – methyldopa
C. Labetalol
D. Nifedipine

151. All of the following can be administered in acute hypertension during except
A. IV labetalol
B. IV nitroprusside
C. IV dihydralazine
D. IV diazoxide

152. Which of the following is not a part of HELLP syndrome?


A. Hemolysis
B. Elevated liver enzyme

www.EduMedWeb.com www.EduMedWeb.com www.EduMedWeb.com


www.EduMedWeb.com www.EduMedWeb.com Page 21

C. Thrombocytopenia
D. Reteroplacental hemorrhage

153. A Cesarian section was done in the previous pregnancy. All of the following would be
indications for elective Cesarean section except
A. Breech
B. Macrosomia
C. Polyhydramnios
D. Post-term

154. Which of the following is responsible for pubertal growth in females?


A. Decreased level of adrenal androgens at puberty
B. High level of estrogen at puberty
C. Pulsatile release of GnRH during sleep
D. Increased sensitivity of HPO axis to estrogen

155. One of the following forms the basis for sex chromatin testing?
A. Barr body
B. Testosterone receptors
C. Hormone levels
D. Phenotypic features

156. Hysteroscopic myomectomy scores over open myomectomy in all of the following except
A. Less recurrence
B. Less post-operative pain
C. Less bleeding
D. Early ambulation

PAEDIATRICS

157. A 32-wee, 1400 g neonate is born to a primigravida. The baby did not require resuscitation
and showed stable vitals. The baby was transferred to the feeding of the patient?
A. Start total enteral feeding and IV feeding not required
B. Start IV feeding with minimal enteral feeding
C. Start IV feeding and introduce feeding on 2nd day of life
D. Start parenteral feeding and institute oral feeding on 2nd day of life

158. Which of the following is not a component of Kangaroo mother care (KMC)?
A. Skin to skin contact
B. Supplementary nutrition
C. Exclusive breast feeding
D. Exclusive discharge and follow-up

159. A 32 week baby is born to a mother with eclampsia, who was given IV magnesium
sulphate. The baby was resuscitated and transferred to the NICU. 12 hours later, the baby
showed hypotonia, lethargy, constricted pupils and two episodes of seizures. The staging of HIE
is
A. 1
B. 2
C. 3
D. There is no HIE

www.EduMedWeb.com www.EduMedWeb.com www.EduMedWeb.com


www.EduMedWeb.com www.EduMedWeb.com Page 22

160. Asymmetric Moro’s reflex at birth is indicative of


A. HIE
B. Brain damage
C. Erb’s palsy
D. Kernicterus

161. A very preterm baby on 30 mL/.kg of enteral feeding developed sudden servere abdominal
distension with visible bowel loops on day 6 of life. The baby also showed temperature
instability and lethargy. X-ray of the abdomen showed portal venous gas. The staging of NEC is
A. Ib
B. 2a
C. 2b
D. 3a

162. Which of the following babies has the least risk of developing hypoglycemia?
A. A baby born to mother treated with beta blockers
B. Infant of diabetic mothers
C. Appropriate for gestational age babies
D. IUGR babies

163. The most common fungal infection in the neonates transmitted by caregiver’s hands is
A. Candida albicans
B. Candida glabrata
C. Candida tropicalis
D. Candida parapsilosis

164. A child has deficient bone mineralization with low serum calcium, high serum phosphorus,
with decreased urinary excretion of calcium and phosphorus and elevated levels of alkaline
phosphatase. The most likely diagnosis is
A. Nutritional rickets
B. Renal tubular rickets
C. Renal glomerular rickets
D. Celiac rickets

165. Bull-neck is seen in severe cases of which of the following?


A. Diphtheria
B. Tubercular lymphadenitis
C. Mumps
D. Goitre

166. In a rural clinic, a 3-year old girl child is brought by her mother and is emaciated. Her
hemoglobin was 5g/dL. The girl also has edema over her knees and ankles with discrete rash
on her knees, ankles and elbows. Th emost likely worm infeststion causing these manifestations
is
A. Hook worm
B. Round worm
C. Whip worm
D. Pin worm

www.EduMedWeb.com www.EduMedWeb.com www.EduMedWeb.com


www.EduMedWeb.com www.EduMedWeb.com Page 23

167. A 6-week baby presents with cough and cold for the past 3 days. Respiratory rate is
48/min. On examination, patient is febrile with wheezing but no chest in drawing. Which of the
following is not true?
A. Child has pneumonia
B. Antibiotics are not required
C. Wheezing to be treated
D. Fever to be treated

168. A 10-year old presents with edema, oliguria and frothy urine. HE has no past history of
similar complaints, On examination, his urine was positive for 3 + proteinuria, no RBCs/WBCs
and no casts. His serum albumin was 2.5 g/L and serum creatinine was 0.5 mg/dL. The most
likely diagnosis is
A. IgA nephropathy
B. Minimal change disease
C. Acute interstitial nephritis
D. Membraneous nephropathy

169. A 6 – year old girl presents with fever for the past 5 days, generalized erythematous rash,
strawberry tongue and cervical lymphadenopathy. The most likely diagnosis is
A. Kimura disease
B. Kawasaki disease
C. Scarlet fever
D. Rosie-Dorfman syndrome

170. A 6-year old girl presents with fever for the past 5 days, generalized erythematous rash,
strawberry tongue and cervical lymphadenopathy. The most likely diagnosis is
A. Kimura disease
B. Kawasaki disease
C. Scarlet fever
D. Rosie-Dorfman syndrome

171. Enzyme replacement therapy is available for


A. Gaucher’s disease
B. Pompe disease
C. San Philipo disease
D. Fabry’s disease

172. A child presents will albinism. He should be evaluated for


A. ENT consultation
B. Eye consultation
C. Electrocardiography
D. Neurosurgery

173. A 34-week female fetus was born as shown below. Identify the congenital anomaly
A. Anencephaly
B. Iniencephaly
C. Anen- Iniencephaly
D. Complete craniospinal rachischisis

174. A 7-year old boy presents with a right sided hemangioma and left sided focal seizures. The
most likely diagnosis is

www.EduMedWeb.com www.EduMedWeb.com www.EduMedWeb.com


www.EduMedWeb.com www.EduMedWeb.com Page 24

A. Neurofibromatosis
B. Incontinetia pigmenti
C. Hypermelanosis of Ito
D. Sturge-Weber disease

176. A 1.5 year old female is brought to the clinic with complaints of excessive enlargement of
head, intolerance to feeds and severe malnourishment. MRI imaging was suggestive of a
medulloblastoma causing obstructive hydrocephalus. Which of the following is an example of
irrational management of the patient?
A. Craniotomy and sub-total excision of the tumour. Surgeon leaves the layer of the tumour
adherent with colliculus
B. First ventriculoperitoneal shunt was done
C. CCNU and vincristine were given as chemotherapy
D. Radiotherapy 35-40 Gy was given to the whole craniospinal axis

ORTHOPAEDICS

176. In a case of tuberculosis of the thoracic spine, the earliest sign of cord compression is
A. Bladder dysfunction
B. Extensor plantar
C. Motor weakness
D. Sensory loss

177. A 5-year old boy presents with painand swelling in diaphysis of tibia. He also complains of
fever. On examination, his ESR was raised, Which of the following is the likely diagnosis?
A. Ewinng arcoma
B. Chondrosarcoma
C. Osteogenic sarcoma
D. Fibrosarcoma

178. The signs of malignant transformation in osteochondroma are all except


A. Pain
B. Weight loss
C. Increase in size
D. Increase in thickness of cartilage cap

DERMATOLOGY

179. Olympian brow and rhagades are seen in


A. CMV inclusion disease
B. Ectodermal dysplasia
C. Congenital syphilis
D. Hyper IgE syndrome
180. A 60-year old male presents with painful, grouped vesicles over erthematous plaques in T3
dermatome region of trunk, which of the following would be the likely causative organism?
A. Varicella zoster
B. Herpes simplex
C. Pox virus
D. Papilloma virus

www.EduMedWeb.com www.EduMedWeb.com www.EduMedWeb.com


www.EduMedWeb.com www.EduMedWeb.com Page 25

181. A 19-year old male presents with several comedones, papules and pustules on face and
trunk. The appropriate drug of choice for the patient would be
A. Topical retinoic acid
B. Topical retinoic acid + Oral doxycycline
C. Topical clindamycin
D. Topical azithromycin

182. A 37 year female presents with multiple, linear, itchy wheals, with itching for 30 minutes at
the site. The most probable diagnosis is
A. Dermatographic urticaria
B. Pressure urticaria
C. Acute urticaria
D. Chronic urticaria

183. A young 8 year old boy presents with multiple discrete, shiny, pin-head papules on dorsal
aspect of hand, forearms and shaft of penis. The diagnosis is
A. Molluscum contagiosum
B. Scabies
C. Lichen planus
D. Lichen nitidus

184.A 30-year old male presents with flaccid bullae on an erythematous base and erosions over
the oral nmucous membrane. The blisters developed painful erosions on rupture. What would
bethe most likely finding on an immunofluorescent examination of the skin biopsy of this
patient?
A. Linear IgA deposition in dermopeidermal junction
B. Linera IgA deposition in dermopeidermal junction
C. Fish net IgG deposition in epidermis
D. Granular deposits of IgA in dermal papillae

185. A patient presented with thinning of nails and onychyloysis. What else finding can be
observed in him?
A. Violaceous papules
B. Suprabasal split
C. Basal degeration
D. Acantholysis

186. A patient presents with focal alopecia areata. All of the following are associations of
alopecia areata except
A. Nail pitting
B. Atopy
C. Geographical tongue
D. Exclammatory mark

187. A man presents with rashes on face and also complains of decreased mental function. He
is also having few macular lesions on his skin. On CT scan, intracranial calcification was seen.
His wife is normal. His 10-year old daughter is also normal but his 6 year old son is also having
similar skin lesions. What would be the most likely diagnosis?
A. Neurofibromatosis-1
B. Neurofibromatosis-2
C. Xeroderma pigmentosum

www.EduMedWeb.com www.EduMedWeb.com www.EduMedWeb.com


www.EduMedWeb.com www.EduMedWeb.com Page 26

D. Autosomal dominant inheritance

ANAESTHESIA

188. A patient of head injury is intubated and ventilated. The ideal mode of ventilation in him
wound be
A. CMV
B. CPAP
C. AMV
D. SIMV

189. The appropriate size of LMA for an average adult patient weighing 50 kg is
A. 2.5
B. 3.0
C. 4.0
D. 5.0

190. At supra MAC concentrations, anaesthetics lead to shift of EEG wave from
A. α to β waves
B. β to δ waves
C. δ to θ waves
D. θ to α waves

191. The plane of surgical anesthesia during either anesthesia is defined as


A. Loss of consciousness
B. Loss of consciousness to the onset of spontaneous respiration
C. From onset of regular respiration to cessation of spontaneous breathing
D. Absence of reflexes

192. Which of the following drugs does not affect absorption and secretion of cerebrospinal
fluid?
A. Halothane
B. Nitrous oxide
C. Ketamine
D. Thiopentone sodium

193. Which of the following intravenous anesthetic agents is contrsindicated in epileptic patients
posted for general anesthesia?
A. Ketamine
B. Thiopentone
C. Propofol
D. Midazolam

194. Midazolam causes all except


A. Anterograde amnesia
B. Retrograde amnesia
C. Cause tachyphylaxis during high dose infusions
D. Decreased cardiovascular effects as compared to propofol

RADIOLOGY

www.EduMedWeb.com www.EduMedWeb.com www.EduMedWeb.com


www.EduMedWeb.com www.EduMedWeb.com Page 27

195. Puff of smoke appearance on cerebral angiography is seen in


A. ACA aneurysm
B. Cavernous sinus thrombosis
C. Moyamoya desease
D. Vein of Galen malformation

196. A 7-year old patient presents with severe headache, paralysis of upward gaze, loss of light
perception and accommodation, nystagmus and failure of convergence. CT scan showed
homogenous hyperdense lesion above the sella and in the posterior part of the third ventricle.
MRI showed that the lesions were homogenous and isointense on T1 weighted imaging, and
isointense on T2 weighted imaging with intense contrast enhancement. The most likely
diagnosis is:
A. Teratoma
B. Germinoma
C. Dermoid
D. Choroid plexus carcinoma

PSYCHIATRY

197. All of the following statements are true about blackouts except
A. The person appears confused to the onlookers
B. Remote memory is relatively intact during the blackout
C. It is a discrete episode of anterograde amnesia
D. It is associated with alcohol intoxication

198. According to the ICD-10 revision, for establishing a diagnosis of mania, the symptoms
should persist for at least
A. 1 week
B. 2 weeks
C. 3 weeks
D. 4 weeks

199. The evidence-based psychological therapy of choice for depression is


A. Group discussion therapy
B. Counselling
C. Cognitive behavior therapy
D. Psychological psychotherapy

www.EduMedWeb.com www.EduMedWeb.com www.EduMedWeb.com


www.EduMedWeb.com www.EduMedWeb.com Page 28

ANSWER KEY

1 B 41 C 81 C 121 C 161 C
2 D 42 C 82 D 122 A 162 C
3 B 43 C 83 A 123 A 163 D
4 D 44 B 84 A 124 A 164 C
5 B 45 D 85 B 125 D 165 A
6 B 46 A 86 A 126 A 166 A
7 B 47 A 87 D 127 A 167 A
8 B 48 C 88 C 128 A 168 B
9 C 49 D 89 B 129 A 169 B
10 C 50 C 90 A 130 B 170 B
11 D 51 A 91 A 131 B 171 A
12 A 52 B 92 A 132 D 172 B
13 A 53 D 93 D 133 D 173 D
14 B 54 B 94 B 134 A 174 D
15 A 55 A 95 B 135 C 175 D
16 B 56 D 96 A 136 A 176 B
17 D 57 D 97 D 137 D 177 A
18 D 58 A 98 A 138 D 178 B
19 B 59 B 99 D 139 A 179 C
20 A 60 C 100 C 140 C 180 A
21 B 61 D 101 A 141 C 181 B
22 A 62 B 102 C 142 C 182 A
23 B 63 B 103 C 143 D 183 D
24 B 64 A 104 C 144 C 184 C
25 B 65 D 105 B 145 C 185 C
26 A 66 A 106 A 146 B 186 C
27 A 67 C 107 A 147 C 187 D
28 A 68 A 108 B 148 C 188 A
29 A 69 A 109 C 149 C 189 C
30 C 70 A 110 C 150 A 190 B
31 A 71 C 111 B 151 D 191 C
32 A 72 D 112 D 152 D 192 B
33 C 73 D 113 C 153 C 193 A
34 A 74 A 114 A 154 C 194 B
35 B 75 A 115 B 155 A 195 C
36 B 76 D 116 B 156 C 196 B
37 C 77 A 117 A 157 B 197 A
38 A 78 B 118 A 158 B 198 A
39 B 79 B 119 A 159 B 199 C
40 B 80 B 120 C 160 C 200

www.EduMedWeb.com www.EduMedWeb.com www.EduMedWeb.com


www.EduMedWeb.com www.EduMedWeb.com Page 1

AIIMS PG MAY 2015


(Based on Memory)

1. Recurrent respiratory papillomatosis is treated locally by


A. acyclovir
B. cidofovir
C. zinc
D. ranitidine
Ans: B

2. A child has disc battery in the nose. Which is most appropriate concern?
A. Keep on instilling nasal drops intermittently till the foreign body dislodges
b. Keep the battery as it is and refer to higher expertise for removal
C. Battery contents might leak resulting into chemical damage of the surrounding tissue
D. Elective removal as the child has high risk of contracting tetanus.
Ans: B

3. HPV do not cause which type of carcinoma


A. Base of tongue carcinoma
B. Tonsillar carcinoma
C. Oropharyngeal carcinoma
D. Laryngeal papillomatosis
Ans: C

4. Artery supplying nasopharyngeal angiofibroma?


A. Ascending pharyngeal
B. Facial
C. Internal masseter
D. ?

5. Kashima operation done for


A. Vocal cords
B. Cholesteatoma
C. Industry surgery
Ans: A

6. Most reliable test for eustachian tube dysfunction:


A. Politzer test
B. Tympanometry
C. VEMP
D. Rhinomanometry
Ans: B

Pathology

www.EduMedWeb.com www.EduMedWeb.com www.EduMedWeb.com


www.EduMedWeb.com www.EduMedWeb.com Page 2

1.Oil red 0: is fat stain used on frozen sections (cryostat sections)


2. Lymphoma with starry sky appearance: C Myc (seen in burkitt’s)
3. Multiple factor deficiency: treatment is FFP (not Cryo because cryo has only factor 8, 13, vwf
and fibrinogen) (not whole blood also because plasma in stored whole blood is deficient in factor
8 & 5)
4. Platelets transfused pre operatively in thrombocytopenia to keep the levels at? 50000
(minimum)
5. Cause of caseous necrosis in TB: Delayed type of Hypersensitivity (t cells kill mycobacteria)
6. Female with suspected ovarian mass, on laparoscopy was found to have a smooth surface
tumors with signet rings and mucin secreting cells: Krukenberg
7. Factor responsible for adhesion in platelets? : vwF
8. Factor increasing strength of clot: factor 13
9. Inheritance of ABO: codominant
10. Severity of disease increases with every generation: Anticipation-
11. Not autoimmune disease: UC
12. Crumpled tissue paper appearance: glucocerebroside accumulation (gaucher’s disease)
13. Fibroblasts in incised wound: 4-5 days
14. Forward scatter in Flow cytometry: indicates size of cell (side scatter indicates granularity of
cell)

15. Microscopic view of fluorescent green RBCs on a dark background …. Mechanism by which
image is viewed?
A. By cathode ray tube
B. By dark field/phase condenser
C. Dichroic mirror
D. Phase plate
Ans: C

16. Crumpled tissue paper appearance of a cell on bone marrow examination –


glucocerebroside accumulation in a cell.

17. Most abundant collagen in basement membrane – type 4


18. Carcinoma thyroid spresds mostly by lymphatics – papillary
19. Forward scatter in flow cytometry – cell size
20. Receptor for lgE is present on – mast cell
21. A karyogram with 2X and 1Y chromosome was given. Diagnosis is Klinefelter syndrome –
gynecomastia with thin extremities.
22. Ovarian mass with smooth surface tumor with signet ring cells and mucin secreting cells –
Krukenbergs tumor.
23. Diagnosis of Langerhan cell histiocytosis – Birbeck granules

24 Reticular fibres of collagen tissues re present in all except


A. Thymus
B. Bone marrow

www.EduMedWeb.com www.EduMedWeb.com www.EduMedWeb.com


www.EduMedWeb.com www.EduMedWeb.com Page 3

C. Spleen
D. Lymph node
Ans: B

25. Brown fat is not found in


A. scapula
B. subcutaneous
C. adrenal
D. around blood vassel

26. Two barr bodies are seen in??


A. XXX
B. 47 XXY
C. X0
D. 46 XX
Ans: A. Barr body is equal to no of x chromosome minus one

OBG
1.Dexa dose for lung maturity
A. 6 mg 4 time 12 hrs apart
B. 6 mg 2 time 24 hrs apart
C. 12 mg 2 time 24 hrs apart
D. 12 mg 4 time 12 hrs apart
Ans: A

2. Dose of carbetocin 100 mcg over 1 min I/v


3. Dose of FA 4mg
4. Fistula question is ureterovaginal
5. Amenorrhea galactorrhoea syndrome hcg
6. Amenorrhea breast development AIS
7. Proteinuria severe preeclampsia is 2gm
8. 6wk preg safe and accurate is Usg
9. Supports all except are recto ago all septum
10. Sperm count: 1.5 ml 15 million 4% and 32%
11. Unprotected intercourse: copper T
12. Signet ring cells: Krukenberg
13. 40 year old infertility and pcos: ocp
14. Oocyte prophase
15. Recurrent abortion Vdrl

16. Drug of choice in hypertension in pregnency


A. Enalapril
B. Methyldopa
C. Diuretics

www.EduMedWeb.com www.EduMedWeb.com www.EduMedWeb.com


www.EduMedWeb.com www.EduMedWeb.com Page 4

D. Captopril
Ans: B

17. Drug contraindicated in hypertension in pregnancy


A. Methyldopa
B. Labetalol
C. Nifedipine
D. Enalapril
Ans: D

18. Drug contraindicated in pregnancy? Methotrexate, azathioprine, acclosporine, retinoids


PSM
1. Diastolic BP of 200 persons, values of 1st and 3rd quartile is 90 and 102 respectively. How
many persons have diastolic BP above 102.
A. 25
B. 50
C. 90
D. 102

2. Nikshay software of Govt. of India is for


A. TB
B. High risk neonate
C. High risk pregnancy
D. Malaria / accident or HIV

3. Sensitivity is
A. True positive
B. True negative
C. False positive
D. False negative

4. Mean GFR of 100 pt is 85 ml/min. SD is 25 ml/min. 90% confidence interval values lies
between
A. 80 and 90
B. 81 and 89
C. 75 and 95
D. 83 and 87

5. In one study, highest value of 58 was mistakenly taken as 85.. This mistake will lead to
A. High mean, high median
B. High mean, same median
C. Low mean, same median
D. Same mean, high median

www.EduMedWeb.com www.EduMedWeb.com www.EduMedWeb.com


www.EduMedWeb.com www.EduMedWeb.com Page 5

6. HDI includes
A. Life expectancy at birth, knowledge, descent life style
B. Life expectancy at one year, knowledge, descent life style
C. Life expectancy at one year,…

Anatomy

1. Which of the following is not a branch of Ext carotid – transverse cervical


2. Which of the following does not have microvilli – CD
3. Fusion of xiphi and sternal body occurs at what age? – 8/12/16 years?
4. Which mucleus deep to facial colliculus – Abducens
5. Which of the following is not derived from mesonephric duct – Glomerulus
6. Which of the following is not a support of uters – urogen diap/pelvic diap/perineal body/?
7. Nerve not having GVE – Olfactory

Physiology

1.Graph on cardiac cycle – point E


2. Extracellular fluid calculation – 18L
3. If resistance is double what’s the flow in vessel
4. Voluntary output fibres – Alpha/Gamma/both
5. Brown fat in all except – Subcut/Scapula/Adrenal cortex/along vessel
6. Beta cell also produces – Amylin
7. EMG + EOG + EEG – NREM/REM/??

Biochemistry

1. Common enz btwn Glycogenesis and Glycogenolysis

2. si RNA – Knock out/in/down

3. Q on glycogen storage diseases – history given

4. Cerebroside/galactosidase?

5. on total cholesterol HDL, LDL and something to calculate

6. A 6 month old child started vomiting and ceased to gain weight. At 8 months, he had to be
admitted to the hosipital. After one week, he started getting drowsy. He couldn’t tolerate feeding
even by gastrostomy and had to be given intravenous glucose, following which he improved
dramatically and came out of coma in 24 hours. Physical examination was normal, but his urine
had high levels of glutamine and uracil, and increased levels of ammonia in blood. What is the
enzyme defect he has?
A. Ornithine transcarbamoylase

www.EduMedWeb.com www.EduMedWeb.com www.EduMedWeb.com


www.EduMedWeb.com www.EduMedWeb.com Page 6

B. Carabamoyl phosphate synthase 1


C. Argininosuccinate synthase
D. Arginase

Pharmacology

1.Methacholine – M2 agonist

2. Which of the following is protease inhibitor


A. enfuvirtide
B. saquinavir
C. adefovir

3. Haloperidol – starts with tremor – doc – carbamazepine

4. Some muscle twitching over eyes – drug to be used?

5. Drug not acting by insulin release? Pioglitazone

6. Lipolysis by which adrenoceptor


A. Alfa 1
B. Beta 2
C. Alfa 2
D. Beta 3

7. Methacholine agonist at
A. M2
B. M1
C. M4
D. M3
Answer: A. Agonist of muscarinic receptors: M1 – Oxotremorine, M2 – Methacholine, M3 –
Bethanechol

8. Which of the following topical agents causes heterochromia iridis?


A. Latanoprost
B. Prednisolone
C. Timolol
D. Olopatadine
Ans: A. Latanoprost

9. Which of the following diuretic is most appropriate for mild or moderate HTN?
A. loop diuretic
B. thiazide
C. potassium aparing

www.EduMedWeb.com www.EduMedWeb.com www.EduMedWeb.com


www.EduMedWeb.com www.EduMedWeb.com Page 7

D. osmotic diuretics

10. Carbetocin for postpartum hemorrhage?


A. 10 ml in 2 mins
B. 100 ml in 2 minutes
C. 20 ml in 1 minute
D. 200 ml in 2 minutes

Microbiology

1. Vibrio toxin acts by – desmosomes

2. Q on respiratory papillomatosis (one was HPV causes all cancer except – Ca tongue, Cw
nasophar, Ca tonsil, Ca Respiratory mucosa?)

3. Hepatitis virus having retrovirus property?

4. Rheumatic fever suspected due to chorea but pharyngeal swab neg – best test now should b
– ASO?/?

5. Person HBs pos but anti HBc neg – should tell pt that he is normal/do PCR DNA/repeat HBs
after 6 months/?

6. Reverse transcriptase virus


A. Hep.A
B. Hep.B
C. Hep.C
D. Hep.E

7. Glyceryl Trinitrate is given sublingual as it is a


A. Nonionic compound with high lipid solubility
B. Iionic compound with high lipid solubility
C. Iionic compound with low lipid solubility
D. Nonionic compound with lipid solubility

FMT

1. Car falling down – diff between driver and bajuwala, seat belt

2. Wound on shin of tiba – patient was asked to do dresing, patient did not do the dressing,
Doctor was in a busy schedule negligence?

3. Site of knot in classical hanging?

www.EduMedWeb.com www.EduMedWeb.com www.EduMedWeb.com


www.EduMedWeb.com www.EduMedWeb.com Page 8

4. Mr.X shot mr.Y with shotgun, but mr.Y got a small injury on thigh, and mild bleeding which
stopped spontaneously. Bt still Mr.Y filed case against mr.X. What will be the IPC for penalty to
Mr.X
A. 302
B. 304
C. 324
D. 326

5. Judicial hanging knot at


A. Angle of jaw
B. At mandible
C. Behind occiput
D. Depend on hangman

6. Criminal act 2013. Conset age limit of Sexual Offences


A. 16yr
B. 18yr
C. 14yr
D. 15yr
Ans: B

7. Delayying of death sentence by high court in case of pregnant female is delayed till delivery is
under IPC
A. 316
B. 317
C. 318
D. 319
Ophthalmology

1.Stain for Granular stromal dystrophy – masson’s trichrome

2. Trochanteric nerve palsy- ?

3. Which of the following drugs won’t cause whorls like opacities in cornea?
A. Chloroquine
B. Amiodarone
C. Chlorpromazine
D. Indomethacin

2. Characteristic marker of limbal epithelial cell


A. keratin
B. ABCG2
C. enolase
D. collagen

www.EduMedWeb.com www.EduMedWeb.com www.EduMedWeb.com


www.EduMedWeb.com www.EduMedWeb.com Page 9

Ans: B

3. The retina receives its blood supply from all except:


A. posterior ciliary artery
B. central retinal artery
C. retinal arteries
D. plexus of zinn and haller artery
Ans: D

Medicine

1. RTA with hypoglycemia – i. v. Dextrose


2. Hypertensive bleed – Basal gangila
3. Some lead poisoning case?????
4. First sign of alcohol withdrawal – tremors
5. DLCO increased in all except – Asthma
6. 1-2 syndromes(hurler?)???????
7. Anomic aphasia – cannot name things but comprehension and all normal
8. Regarding TRALI, which is not true?
A. Present in 24 hrs

2. Hypoxemia and noncardiogenic pul edema are cardinal feat

3. Assoc wid all blood products, characteristically with plasma


D. Mc in pt with sepsis and cardiac transplants
Ans: TRALI is mostly associated with fresh frozen plasma and platelets, so the best answer look
like occurs within 24 hrs.

9. Which of the following complications is likely to result after several units of blood transferred?
A. Metabolic alkalosis.
B. Metabolic acidosis.
C. respiratory alkalosis.
D. respiratory acidosis.

10. Most commonly involved part in hypertension


A. Basal gangila
B. Thalamus
C. Hemisphere

11. Bosentan – used in pulmonary artery hypertension.

Surgery

1. Oesophageal ca prognosis depends on?

www.EduMedWeb.com www.EduMedWeb.com www.EduMedWeb.com


www.EduMedWeb.com www.EduMedWeb.com Page 10

A. T stage
B. Length of segment involved
C. Age of presentation
D. O
Ans: A

2. A child with billiary atresia – prep bili 12, hepatojejunostomy done, postop 2 weeks bili6,
cause?

3. Incision for diaphragm surg – circumlinear

4. Screening not useful in which cancer?


A. colon
B. prostate
C. testicular
D. breast
Ans: C

5. Absent microvilli?
A. Pct
B. Duodenum
C. CT
D. Gall bladder
Ans: C

6. Prophylactic splenectomy was done. Resistant case of ITP to steroids, patient develops
fever, chills and rigor. Isnt it? Penumococcal pneumonia.. Lower lobe consolidation on cxr.?
Lung atelectasis

7. Preptic ulcer is asso with all except one??


A. cirrhosis
B. zollinger ellison syndrome
C. primary hyperparathyroidism
D. pennicious anemia

8. 3 yr old child with hydrocoele hernia sac.. Management?


A. Herniotomy
B. Herniorrhaphy
C. Observation
D. 9
Ans: A

9. Mallory weiss tear mc artery involved?


A. Short gastric

www.EduMedWeb.com www.EduMedWeb.com www.EduMedWeb.com


www.EduMedWeb.com www.EduMedWeb.com Page 11

B. Left gastric
C. Coronary
D. Phrenic
Ans: B

10. Oncocyte dx test is done for?


A. Chemotherapy in HR – ve
B. Hormone therapy in HR + ve
C. Herceptin in HER2 + ve
D. Hormone therapy in HR – ve
Ans:

11. 2 month neonatal hepatitis with conjugated bilirubin.. liver biopsy showed granular
cytoplasm which is pas positive..diagnosis A. cong hepatic fibrosis B. extrahepatic biliary atresia
C. malformation of duct

12. patient after splenectomy developed chill and rigor on 3 rd day..what correlates with the
patient. A – consolidation in lung B – uti C – renal failure..D – port site infection

13. MESORECTUM CONTAINS ALL EXCEPT .sup rectal vein, inf rectal vein pararectal nodes,
some plexus inferior rectal artery

14. the q 60 yr old female with blood stained nipple discharge with + ve family h/o breast ca
NEXT investigation after clinical assessment?
a. MRI
b. Ductography
c. Cytology of discharge
d. Sono mammogram

15. a young girl with a mass in lower abdomen involvibg hypogastrium, unable to insinuate
fingers from pelvic bone??
A. duplication of intestine
B. mesenteric cyst
C. omental cyst
D. ovarian cyst

16. m/c bariatric surgery


A. laparo band
B. sleeve gastrectomy
C. roux en y

17. during inguinal hernia operation a track was left in the lateral part of iliopubic tract…pt
complaints of paraesthesia & pain in rt. Thigh, nerve involved?
a. illioinguinal nv.

www.EduMedWeb.com www.EduMedWeb.com www.EduMedWeb.com


www.EduMedWeb.com www.EduMedWeb.com Page 12

b. genitofemoral nv.
C lat. Cutaneous nv. Of thigh…
D obturator nv
Was site of paresthesia mentioned-
Anterior thigh – Genitofemoral nerve
Lateral thigh – Lat cutaneous nerve of thigh
I guess here answer should be lat cutaneous of thigh

18. Best imaging modality to diagnose acute appendicitis in children. Answer is CT scan.

Ortho

1. Young adult with irreparable rotator cuff injury, which of the following procedures can be
done?
A.Acromioplasty
B. Tendon transfer
C. Toatal shoulder replcement
D. Reverse shoulder replacement
Ans: B

3. Card test for?


A. Dorsal interossei
B. Palmar interossei
C. Lumbricals
D. Thenar muscles
Ans: B

4. Which traction nt used in lower limb?


A. perkins
B. bryants
C. dunlop
Ans: C

5. Jersey finger is due to ruture of?


A. extensor digiti minimi
B. extensor pollicis
C. FDP
D. FDS
Ans: A Jersey finger (also called Rugby finger or Sweater finger) describes a type of injury
where there is avuulsion of the flexor digitorum profundus (FDP) at the base of the distal
interphalangeal joint (DIP)

6. What is not done while removing intervertebral disc?


A. Laminectomy

www.EduMedWeb.com www.EduMedWeb.com www.EduMedWeb.com


www.EduMedWeb.com www.EduMedWeb.com Page 13

B. Laminotomy
C. Laminoplasty
D. Hemilaminectomy

7. Most active part of bone


A. Cortical bone
B. Cancellous bone
C. Periosteun
D. Endosteum

8. osteoporosis
A. low calcium high alk phosphate
B. normalalcium high alk phosphate
C. low calcium low alk phosphate
D.?

9. A person falls from building and lands on foot. What #’s correspond to this injury
A. Cervical #
B. Pond #
C. Lumbar #
D. base of skull #.

10. Judet view is for


A. Pelvis
B. Calcaneum
C. Scaphoid
D. ?
Ans: B

11. muscle not attached at greater tubercle?


1. suspraspinatus
2. subscapularis
3. infraspinatus
4. teres minor
Ans: B

Dermatology

1. A 65 year old male itchy bullae over erythematous areas and normal skin
A. bullous perphigoid
B. dermatitis herpetiformis
C. lga pemphigus
D. pemphigus vulgaris
Ans: A

www.EduMedWeb.com www.EduMedWeb.com www.EduMedWeb.com


www.EduMedWeb.com www.EduMedWeb.com Page 14

2. A 23 yr old pregnant diabetic female comes with lake of pus lesions on skin. Treatment is
A. methotrexate
B. azathioprine
C. retinoids
D. cyclosporine
Ans: D

3. Young female with blue macules on forehead and ocular macules. The diagnosis is
A. Mongolian spot
B. Nevus of ota
C. Nevus of ito
D. Beckers nevus
Ans: A

4. 50 year old male from bihar came with symmetrical juicy papules and nodules with
hypopigentation on face and trunk. There was no hypoaesthesia and nerve thickening. There
was fever in childhood
A. LL
B. Post kala azar dermal leishmaniasis
C. BL
D. Mycosis fungoides
Ans: B

5. patient comes with more than 30% skin blistering with lip involvement. Common cause.
A. bacteria
B. virus
C. drugs
D. malignancy
Ans: C

6. Picture was that of a lady with two white patches over the neck and chin associated with
leucotrichia options were
A. Segmental vitiligo
B. Acne vulgaris
C. Focal vitiligo
D. Piebaldism
Ans: A

7. Papular lesions on dorsum of hands, shaft of penis in a child (confusion as to what this
question was-itchy? Non-itchy? Since 7 days?). Hence not answering this.
A. Lichen planus
B. Lichen niditus
C. Lichen scrofulosorum

www.EduMedWeb.com www.EduMedWeb.com www.EduMedWeb.com


www.EduMedWeb.com www.EduMedWeb.com Page 15

D. Scabies
Ans: B

8. A picture with hyperpigmented margins with central hypopigmentation whose margins looked
ike punched out lesions:
A. BB
B. BL
C. LL
D. Histoid hansens
Ans: A

9. A face of a lady was shown with red macular lesions on central part of face around the cheek
and chin with flushing on sun exposure and increasing with otional disturbance
A. Acne vulgaris
B. Acne rosacea
C. SLE
D. DLE
Ans: B

10. 70 yr old man with vesicular eash on lower limb after a course of steroids. Tzanck smear
shows multinucleate giant cells.
A. VZV
B. Vaccinia
C. Molluscum
D. M. Tb
Ans: A

Radiology

1. Judet view of X-ray is for: Acetabulum (pelvis is best answer)


2. Shenton line is seen in X ray of Hip
3. IOC for acute appendicitis in children: USG
4. Investigation of choice in strees fracture: MRI
5. Investigation of choice for biliary atresia in a 2 month old is: Hepatic scintigraphy
6. Patient with h/o tachyarrhythmias is on Implantable cardioverter defibrillator. He develops
shock. Best method to know the integrity of ICD is to do: Plain Radiograph
7. Expansion of the subarachnoid space is seen in intradural extramedullary tumours
8. Safe & accurate for viability at 6 weeks is USG
9. Best non invasive investigation for myocardial viability is FDG PET
10. Hypertensive hemorrhage most commonly affects: Basal ganglia

Anatomy

12. Lower part of sternum fused by age

www.EduMedWeb.com www.EduMedWeb.com www.EduMedWeb.com


www.EduMedWeb.com www.EduMedWeb.com Page 16

A. 12 years
B. 14 years
C. 18 years
D. 10 years
Ans: B

13. Reticular fibres of collagen tissues are present in all except


A. Thymus
B. Bone marrow
C. Spleen
D. Lymph node
Ans: A

14. Cranial nerve nucleus lying deep to facial colliculus


A. Facial
B. Abducent
C. Glosso-pharyngeal
D. Trigeminal
Ans: B

15. Collagen typical of basement membrane


A. Type I
b. Type V
C. Type IV
D. Type III
Ans: C

16. Muscle not inserted on greater trochanter


A. Teres minor
B. Supra-spinatus
C. Infra- spinatus
D. Sub-scapularus
Ans: D

17. Not a branch of external carotid artery


A. Trans-cervical artery
B. Lingual
C. Superior thyroid artery
D. Ascending pharyngeal
Ans: A

18. Card test done for


A. Lumbricals
B. Dorsal interossei

www.EduMedWeb.com www.EduMedWeb.com www.EduMedWeb.com


www.EduMedWeb.com www.EduMedWeb.com Page 17

C. Palmar interossei
D. Adductor pollicis
Ans: C

19. Microvilli are seen in all; except


A. Duodenum
B. Gall bladder
C. PCT
D. Collecting duct
Ans: D

20. Metabokically active part in bone


A. Periosteum
B. Endosteum
C. Cancellous bone
D. Cortical bone

21. Special visceral efferent carries all the cranial nerve except
A. Facial
B. Vagus nerve
C. Olfactory
D. Glosso-pharyngeal
Ans: C

22. Meso-nephric ducts and tubules give rise to all except


A. Glomeruli
B. Paroophoron
C. Epididymis
D. Vas deference
Ans: A

23. Meso-rectal fascia doesn’t contain which of the following


A. Inferior rectal vein
B. Para- rectal node
C. Superior rectal vein
D. Inferior mesenteric plexus
Ans: D

24. Which of these is not a support of the uterus?


A. Urogenital diaphragm
B. Pelvic diaphragm
C. Perineal body
D. Recto-vaginal septum
Ans: D

www.EduMedWeb.com www.EduMedWeb.com www.EduMedWeb.com


www.EduMedWeb.com www.EduMedWeb.com Page 1

AIIMS PG NOV 2016


(Based on Memory)
1. Postherpetic neuralgia is defined as pain lasting beyond how many weeks
(a) 1 week
(b) 2 weeks
(c) 3 weeks
(d) 4 weeks

2. A 24-years-female presents with patchy hair loss in the right temporal and occipital region.
Examination reveals non-scarring alopecia with multiple small broken hairs. Scrapings from
scalp show mild inflammation, perifollicular hemorrhage and surrounding mild lymphocytes
infiltration. What is the likely diagnosis?
(a) Alopecia areata
(b) Androgenic alopecia
(c) Loose anagen hair
(d) Trichotillomania

3. What is the ratio of chest compressions and breaths when a lone person is giving
cardiopulmonary resuscitation
(a) 30:1
(b) 30:2
(c) 15:1
(d) 10:1

4. Why 100% oxygen is administered after nitrous oxide is discontinued on emergency from
anesthesia?
(a) Second gas effect
(b) Diffusion hypoxia
(c) For adequate analgesia
(d) As air is costlier then oxygen

5. Non visualization of gallbladder in hepatic scintigraphy is suggestive of:


(a) Chronic cholecystitis
(b) Carcinoma gallbladder
(c) Acute cholecystitis
(d) Gallstones obstructing CBD

6. A surgeon to decide to operate a patient under epidural anesthesia. 3% xylocaine with


adrenaline is used for administering epidural anesthesia. The patient suddenly develops
hypotension after 3 minutes to administration.
What is the likely cause for this?
(a) Systemic absorption of the drug
(b) Vasovagal effect
(c) Allergy to the drug preparation
(d) Penetration into the subarachnoid space

7. Pigmentation of nail is caused by all of these drugs except:


(a) Cyclophosphamide
(b) Chlorpropamazine

www.EduMedWeb.com www.EduMedWeb.com www.EduMedWeb.com


www.EduMedWeb.com www.EduMedWeb.com Page 2

(c) Chloroquine
(d) Amiodarone

8. A 12-years-old boy presented with gradually progressive annual plaque on his buttocks with
central scarring. What is the most likely diagnosis?
(a) Granuloma annular
(b) Lupus vulgaris
(c) Tinea cruris
(d) Annular psoriasis

9. During laryngoscopy and intubation procedure, all of these are true except:
(a) A sight pressure may be applied at the cricoids cartilage
(b) The laryngoscope is held in the right hand introduced from the right side of the patient.
(c) The neck is fixed with extension at the atlanto occipital joint
(d) After incisor to pull up the tongue and visualize the vocal cords

10. An elderly female had her house destroyed in an earthquake. Following this, she presented
to your office with complaints of anxiety, sadness, lack of sleep, anger palpitations and despair,
consider the following statements:
(a) The lady is suffering from acute stress reaction.
(b) The defense mechanism involved is projection
(c) Drug of choice in this situation is risperidone
(d) She needs referral to a psychiatrist for psychotherapy which of the following statement are
true?
(a) a and c
(b) b and d
(c) a, and c
(d) a and d

11. A-22-years-old male comes to your with complains of frequenting checking of doors even
when they are locked. He is distressed about this fact. He is subsequently diagnosed to have
obsessive compulsive disorder. Consider the following statements:
(a) Repression and reaction formation are the defense mechanisms involved
(b) SSRIS are the drug of choice
(c) Risperidone may be used in SSRIS resistant cases to augment the response
(d) Systemic desensitization is the psychotherapy of choice which of the above are correct
statements:
(a) a and b
(b) b and c
(c) b, c and d
(d) a, b, c and d

12. Which of these is the correct sequence of Maslow’s hierarchy of needs?


(a) Safety – physiological needs – self – actualization – Belonging – self – esteem
(b) Physiological needs – Safety – Belonging – self – esteem– self – actualization
(c) Safety – self – actualization – Belonging – physiological needs – self – esteem
(d) Self – actualization – physiological needs – Safety – Belonging – self – esteem

13. A person has been referred to you by the court. You find a discrepancy between history and
examination findings. Which of these conditions you should be aware of in this situation?
(a) Malingering

www.EduMedWeb.com www.EduMedWeb.com www.EduMedWeb.com


www.EduMedWeb.com www.EduMedWeb.com Page 3

(b) Factitious
(c) Somatization syndrome
(d) Dissociative fugue

14. According to the Who definition of hearing loss, what is the value to classify as profound
hearing loss?
(a) 61 – 71 dB
(b) > 81 dB
(c) > 91 dB
(d) > 101 dB

15. Maximum contribution to the refractive power of the eye is by which part of the eye?
(a) Anterior surface of cornea
(b) Anterior surface of lens
(c) Posterior surface of lens
(d) Posterior surface of cornea

16. What is the usual weight of rabbit used in ophthalmologist experiments?


(a) 1.5 kg
(b) 5 – 7 kg
(c) 2.5 kg
(d)10 – 12 kg

17. Which of these is the correct definition of blindness as per NPCB?


(a) Diminution of field vision to 20. Or less in better eye
(b) Inability of a person to count fingers from a distance of 3 meters
(c) Vision 6/60 or less with best possible spectacle correction in the better eye
(d) Vision 4/60 or less with best possible spectacle correction in the better eye

18. What is the power of the lens that should be used with indirect ophthalmoscope to visualize
the retina?
(a) + 90 D
(b) + 58 D
(c) + 60 D
(d) + 20 D

19. A woman present with complete wound dehiscence 4 – days after a laparotomy.After
prescribing antibiotics for the infection, the surgeon decides to suture the wound. Which of these
suture material should he use?
(a) Vicryl
(b) Mersilic
(c) Catgut
(d) Ethilon

20. Which of the following causes formation?


(a) Movement at fracture site
(b) Rigid immobilization
(c) Compression plating
(d) Intraosseous naliling

www.EduMedWeb.com www.EduMedWeb.com www.EduMedWeb.com


www.EduMedWeb.com www.EduMedWeb.com Page 4

21. Partograph represents various stages of labor with respect to time. True about partograph is
all except –
(a) Each small square represents one hour
(b) Alert and action lines are separated by a difference of 4 hours
(c) Partograph recording should be started at a cervical dilation of 4 cm
(d) Send the patient to first referral unit if the labor progression line crosses the alert line

22. The following graph represents the stages of labor. Which of the following statements is true
about the graph C?
(a) Secondary arrest after progression of labor
(b) Prolonged active phase of labor
(c) Prolonged latent phase / primary arrest of labor
(d) Normal labor in a primigravida

23. Which of the following statement is not true about cervical cancer screening guidelines
according to who?
(a) Pap smear should repeated yearly in woman of reproductive age group
(b) HPV test should be done five yearly in woman between age of 30 to 49 years
(c) VIA/VILI is more reliable at older age as it becomes easier to identify the transformation one
with age
(d) Pap smear can be repeater less frequency if it comes out negative for 3 consecutive years

24. What is done for screening of Down’s syndrome in first trimester?


(a) Beta HCG and PAPP – A
(b) Unconjugated estradiol and PAPPA
(c) AFP and Inhibin A
(d) AFP and Beta HCG

25. What does of misoprostol is used orally to control bleeding in partum heamorrhage?
(a) 1000 micrograms
(b) 400 micrograms
(c) 800 micrograms
(d) 600 micrograms

26. The following graph represents the stages of labor. What is the maximum capacity of bakri
balloon which used in post partum heamorrhage?
(a) 200 ml
(b) 300 ml
(c) 500 ml
(d) 1000 ml

27. A 38 week primigravida presented to the labor room with minimal labor pains and
contraction. On heart rate of the patient is 86 / min and blood pressure is 126 /76 mm Hg. What
should be done next?
(a) Give oxytocin to augment labor
(b) Observe the patient and wait for increase in uterine contractions
(c) Sedate the patient by and give phenergan to decrease labor pains
(d) Induce labor by artificial rupture of membranes

28. What is the drug of choice for precocious puberty in girls?


(a) GNRH analogues

www.EduMedWeb.com www.EduMedWeb.com www.EduMedWeb.com


www.EduMedWeb.com www.EduMedWeb.com Page 5

(b) Danazol
(c) Cyproterone acetate
(d) Medroxy progesterone acetate

29. A young female presents to you with primary amenorrhea. Examination reveals normal
breast development and absent axillary hairs. Pelvic examination shows a normally developed
vagina with citomegaly. On ultrasound, gonads are visible in the inguinal region. What is the
likely diagnosis?
(a) Complete androgen insensitivity syndrome
(b) Partial androgen insensitivity syndrome
(c) Grandal dysgenesis
(d) Mayer Rakitansy Kuster Huser syndrome

30. Which of the following statements is true regarding medical abortion?


(a) Can only be done up to 72 days
(b) Ultrasound should be done in all cases
(c) Only a person certifield under MTP act can perform medical termination of pregnancy
(d) If the patient has an LUCD in – situ, it doesn’t need to be removed

31. What should be the ideal temperature in delivery room?


(a) 20 – 23 degrees C
(b) 25 – 28 degrees C
(c) 30 – 35 degrees C
(d) 37 degrees C

32. Ideal route of drug delivery in neonatal resuscitation is


(a) Intraosseous
(b) Through umbilical artery
(c) Through peripheral vein
(d) Through umbilical vein

33. An unresponsive patient has been brought to you by the police. What is the first thing you
will do?
(a) Start chest compressions immediately
(b) Check carotid pulse
(c) Check for response and cell help
(d) Start resuce breathes

34. In which of the diseases is the following appearance of calve seen in a child?
(a) Spinal muscular atrophy
(b) Muscular dystrophy
(c) Myopathy
(d) Peripheral neuropathy

35. Ponderal index is:


(a) Head circumference to abdominal circumference ratio
(b) Mid – upper arm circumference to head circumference ratio
(c) Weight in kilograms by cube of height in meters
(d) Square root of height in feet by weight in grams

36. A neonate presented with jaundice on first day of life. How will you manage?

www.EduMedWeb.com www.EduMedWeb.com www.EduMedWeb.com


www.EduMedWeb.com www.EduMedWeb.com Page 6

(a) Observe only as it is mostly physiological jaundice


(b) Exchange transfusion
(c) Phototherapy
(d) Liver function tests and liver biopsy as it is mostly due to cholestasis

37. Which of the following is a true diverticulum of esophagus?


(a) Zenker’s diverticulum
(b) Meckel diverticulum
(c) Epiphremic diverticulum
(d) Parabronchial diverticulum

38. What is the recommended does of steroid for altaining fatal lung maturity?
(a) inj. Betamethasona12 mg for 2 does 12 hours apart
(b) inj. Betamethasone12 mg for 2 does 24 hours apart
(c) inj. Dexanmethasone6 mg for 4 does 24 hours apart
(d) inj. Dexamethasona12 mg for 2 does 12 hours apart

39. A-45-year-old patient presented to you with ongoing massive hematemesis. The patient is
alert and hemodynamically stable. What will be the first step in management?
(a) Do an urgent upper GI endoscopy
(b) Put the patient in recovery position and secure airway
(c) Insert a cannula and start IV
(d) Send for blood transfusion

40. Which one of these is a muscle splitting incision?


(a) Kocher’s incision
(b) Lanz incision
(c) Rutherford – Morrison incision
(d) Pfannenstiel incision

41. In a child surgery was done for biliary stricture with hepatojejunal anstomosis. Postoperative
bilirubin level after 2 weeks was 6 mg / dl from a preoperative level of 12 mg / dl. The reason for
this could be
(a) Normal lowering of bilirubin takes time
(b) Delta bilirubin
(c) Anastomotic stricture
(d) Mistake in job technique

42. Which of these Scoring system is helpful in assessing serverity of wound infection?
(a) South anapton grading scale
(b) ASA classification
(c) Glasgow score
(d) APGAR

43. After midline laparotomy you have been asked to suture the incision. What length of suture
material will you choose?
(a) 2x incision length
(b) 4x incision length
(c) 6x incision length
(d) 8x incision length

www.EduMedWeb.com www.EduMedWeb.com www.EduMedWeb.com


www.EduMedWeb.com www.EduMedWeb.com Page 7

44. In a patient with obstructive jaundice, the bilirubin level is 40 mg/dl. What is the possible
explanation for such a high level?
(a) Liver Failure
(b) Renal Failure
(c) Complete obstruction of common bile duet
(d) Malignant obstruction

45. A-44-years-old male underwent & VATS thymectomy for Myasthenia graves. During
surgery, the pleural was accidentally injured. The surgeon decided to put a drain in the pleural
cavity. Which of these statement is correct about the timing of removed of intercostals chest
tube?
(a) After partial expansion of lungs and < 50 ml output from drain for 2 consecutive days
(b) After complete expansion of lungs and < 30 ml output from drain for 2 consecutive days
(c) On 4th day, irrespective of the output from the drain and lung expansion
(d) After complete expansion of lungs and < 20 ml output from drain for 2 consecutive days

46. Identify the instrument shown below:


(a) DPL catheter
(b) Laparoscopic port trocar
(c) Peritoneal dialysis catheter
(d) Endoscopic ultrasound probe

47. A neonate presented with cicatrizing skin lesions all over the body with hypoplasia of all
limbs. An MRI of the brain reveled diffuse cerebral atrophy. An ophthalmologic evaluation
reveals chorioretinitis. Which of these tests is must likely to show a positive result in this
patient?
(a) Anti – HCMV antibodies
(b) Anti – toxoplasma antibodies
(c) Anti – VZV antibodies
(d) Anti – rubella antibodies

48. A neonate on routine examination at birth was found to have hepatomegaly. Rest of the
examination was antibodies were found to be positive. What sequelae in later life is the child at
risk of?
(a) Renal failure
(b) Sensorineural hearing loss
(c) Hepatic fibrosis
(d) Mental Retardation

49. A child presented at 2 years of age with delayed motor development mental retardation and
finger biting. He was normal at birth. He subsequent develops cerebral palsy arthritis and dies
due to renal failure at age of 25 years. What is the likely enzyme deficiency implicated?
(a) HGPRT deficiency
(b) ADA deficiency
(c) Hexosaminidase deficiency
(d) Ornithine transcarbamyl deficiency

50. Corebellar lesion can produce all of these except?


(a) Ataxic Gait
(b) Past Pointing
(c) Resting tremor

www.EduMedWeb.com www.EduMedWeb.com www.EduMedWeb.com


www.EduMedWeb.com www.EduMedWeb.com Page 8

(d) Nystagmus

51. All of these muscles have parallel oriented fibers except:


(a) Sartorius
(b) Rectus abdominis
(c) Tibialis anterior
(d) Sternohyoid

52. Calcium absorption from gut is in increased by all except:


(a) Phytates
(b) Vitamin D
(c) Alkaline PH in gut
(d) Protein in diet

53. Which of these nerve root is the control center for the Stapedial reflex?
(a) Superior olivary complex
(b) Lateral lemnisus
(c) Inferior callicullis
(d) Medical geniculate body

54. Countercurrent mechanism in not seen in


(a) Eyes
(b) Testes
(c) Kidney
(d) Intestine

55. Diffusion capacity of carbon monoxide is decreased in all except:


(a) Polyeythemia
(b) Interstital lung disease
(c) Emphysema
(d) Pulmonary vascular disease

56. Anterior 2/3rd of the tongue is demarcated from the posterior 1/3rd by:
(a) Passavant’s ridge
(b) Circumvallate papillae
(c) Sulcus terminalis
(d) Epiglottis

57. Parasympathetic nervous system comprises of?


(a) Cranial nerves 3, 5, 7, 10 and sacral nerves 51 – 55
(b) Cranial nerves 3, 7, 9, 10 and sacral nerves 52 – 54
(c) Cranial nerves 5, 7, 9, 10 and sacral nerves 52 – 54
(d) Cranial nerves 3, 5, 7, 10 and sacral nerves 52 – 54

58. Endo – cochlear potential is –


(a) + 45 MV
(b) – 45 MV
(c) – 60 MV
(d) + 85 MV

59. Common intermediate in synthesis of steroid hormones is:

www.EduMedWeb.com www.EduMedWeb.com www.EduMedWeb.com


www.EduMedWeb.com www.EduMedWeb.com Page 9

(a) 7 Dihydrocholoesterol
(b) Cortisone
(c) Pregnenolone
(d) 7 Hydroxyeholssterol

60. The PH of a solution containing 5millimole per liter of H+ ions is closest to what value?
(a) 2.3
(b) 1.7
(c) 3.5
(d) 4.2

61. RQ of a person on pure carbohydrate diet will be:


(a) 1
(b) 0.7
(c) 1.2
(d) 1.5

62. A farmer ingested some unknown poisonous seeds and had pain and vomiting. Soon he
developed paralysis of his lower limbs which ascends till it affected the respiratory muscles and
he died within two days. Identify the likely cause:
(a) Strychnine nux – vomica
(b) Conium maculatum
(c) Solanum lycopersicum
(d) Abrusprecatorium

63. Which vitamin is synthesized in vivo in the body by human?


(a) Folic acid
(b) Niacin
(c) Panthothenic acid
(d) Cyanocobalamin

64. Polysomngraphy contains all of the following tests except?


(a) Electroencephalography
(b) Pulse oximetry
(c) Electroculography
(d) Arterial pCo2 measurement

65. Potential developed due to movement of freely diffusible ions across a semi permeable
membrane is calculated using:
(a) Nernst equation
(b) Gibbs equation
(c) Goldman – Hodgkin – Katz equation
(d) Fick principle
66. Migratory motor complexes in the gut reappear after intervals of:
(a) 90 min
(b) 120 min
(c) 60 min
(d) 150 min

67. Hyperpolarizing cyclienucleotide (HCN) gated channels have a role in:


(a) Cardiac rhythm generation

www.EduMedWeb.com www.EduMedWeb.com www.EduMedWeb.com


www.EduMedWeb.com www.EduMedWeb.com Page 10

(b) Generation of mitochondrial ascion potential


(c) Memory formation
(d) Myocardial muscle contraction

69. Correctly math the following:


Declaration Motive
(1) Declaration of Geneva (i) Global health
(2) Declaration of Helsinki (ii) Hippocratic oath
(3) Declaration of Tokyo (iii) Medical Ethics
(4) Declaration of Oslo (iv) Inhuman practices and torture
(a) 1 – iii, 2 – ii, 3 – iv, 4 – i
(b) 1 – ii, 2 – iii, 3 – iv, 4 – i
(c) 1 – iv, 2 – i, 3 – ii, 4 – iii
(d) 1 – ii, 2 – iv, 3 – iii, 4 – i

70. Hco3/H2co3 is the best butter because it is?


(a) PKa near physiological PH
(b) Its components can be increased or decreased in the body as needed
(c) Good acceptor and donor of H+ ions
(d) Combination of a weak acid and weak base

71. At physicological PH, which of these amino acids has a positive charge?
(a) Arginine
(b) Aspartic acid
(c) Valine
(d) Isoleucine

72. All of these processes take place in mitochondria except:


(a) Beta – oxidation of fatty acids
(b) DNA synthesis
(c) Fatty acid synthesis
(d) Protein synthesis

73. Ammonia from brain is removed as:


(a) Glutamine
(b) Alanine
(c) Urea
(d) Glutamate

74. CG islands in our DNA are important for:


(a) Acetylation
(b) Methylation
(c) T-RNA synthesis
(d) DNA replication

75. A patient has normal blood glucose level as estimated by Glucose – Oxidase Peroxidase
method, shows positive Benedicts test in urine. Which of the following is likely cause?
(a) Fruetosemia
(b) Galactosemia
(c) Denaturation of glucose
(d) Faulty tests

www.EduMedWeb.com www.EduMedWeb.com www.EduMedWeb.com


www.EduMedWeb.com www.EduMedWeb.com Page 11

76. Which of these is most effective for gluconeogenesis?


(a) Citrate stimulation of acetyl carboxylase
(b) Acetyl – COA stimulation of pyruvate carboxylase
(c) Fructose – 2, 6 – piphosphate stimulation of phosphofrukinase – 2
(d) Fructose – 1, 6 – piphosphate stimulation of phosphofrukinase – 1

77. Which of the following is maximum in HDL as compare to other lipoproteins?


(a) Cholesterol
(b) Triglycerides
(c) Apoproteins
(d) Fatty acids

78. Change in sex hormone binding globulin will most affect levels of which of these hormones?
(a) Testosterone
(b) Estrogen
(c) Progesterone
(d) DHEA

79. Rapid source of energy by resynthesizing ATP for excening museles is:
(a) Glycalysis
(b) TCA cycle
(c) Glycogenolysis
(d) Phosphocreatine

80. A 59 year old presents with pain, numbnees and impaired sensation over half of the face
along with ataxia, nystagmus, dysphagia and hoarseness of voice. His pain and thermal
sensations over opposite half are impaired, Horner’s syndrome is present. Likely cause of the
disease is thrombosis of which vessel?
(a) AICA (Anterior inferior cerebellar artery
(b) PICA (Posterior inferior cerebellar artery
(c) Basilar
(d) Pontine vessels

81. All of these structures lie in relation to the left ureter except:
(a) Mesentery of sigmoid colon
(b) Bifurcation of common iliac artery
(c) Quadratus lumborum
(d) Gonadal vessels

82. Extraembryonic mesoderm is derived from:


(a) Epiblast
(b) Primary yolk sac
(c) Secondary yolk sac
(d) Hypoblast

83. According to the new RNTCP guidelines, the following is not suspect of tuberculosis?
(a) Conformed extrapulmonary tuberculosis patient with cough of 2 weeks or more
(b) HIV-Positive patient with cough of any duration
(c) Contacts of sputum Positive tuberculosis patient with cough of any duration
(d) Diabetic person having cough of any duration

www.EduMedWeb.com www.EduMedWeb.com www.EduMedWeb.com


www.EduMedWeb.com www.EduMedWeb.com Page 12

84. Which of these is an obligate intracellular pathogen?


(a) Listeria monosytogenes
(b) Coxiellaburnetili
(c) Legionella pneumophila
(d) Klebsiella

85. What is the most common clinical manifestation who larvae of ascaris, hookworm and
strongyloides migrates through the body?
(a) Asymptomatic
(b) Pneumonitis
(c) Acute dermal reaction
(d) Anemia

86. Why are schizot and late trophozoite stages of plasmodium falciparum not seen in
peripheral blood smear?
(a) They are sequestered in the spleen
(b) Due to adherence to the capillary endothelium, they are not seen in peripheral blood
(c) Due to antigen – antibody reaction and removed
(d) They are seen in mosquito blood

87. Which of the following is not used in osteoporosis?


(a) Milnacipran
(b) PTH
(c) Strontium ranelate
(d) Denosumab

88. Which of the following statements in incorrect about rifabutin as compared to rifampicin?
(a) It has higher bioavailability and stronger activity against all mycobacterium
(b) It has a longer half – life
(c) It has lasser drug interactions
(d) It is preferred in patients with HIV

89. A patient presented to the hospital with serves hydrophobia. You suspect rabies obtain
corneal scraping from the patients. What test should be done on this specimen for a diagnosis
of Rabies?

(a) RT – PCR for rabies virus


(b) Negri bodies
(c) Antibodies to rabies virus
(d) Indirect immunofluorescence

90. In a suspected patient of dengue, all of these are acceptable investigations at day 3 of
presentation except?
(a) NSI antigen detection
(b) Viral culture and isolation in C6/36 cell line
(c) RT – PCR
(d) Elisa for antibody against Dengue virus

91. All of these are seen in hemolytic anemia except:


(a) Yelowing of eyes and sclera

www.EduMedWeb.com www.EduMedWeb.com www.EduMedWeb.com


www.EduMedWeb.com www.EduMedWeb.com Page 13

(b) Increased LDH


(d) Low reticulocyte count

92. The resolving power of a Microscope upon all except:


(a) Size of the aperature
(b) Focal length of the eyepiece
(d) Wavelength of light source used

93. During autopsy of a patient died due to suspected myocardial infarction, the heart was
stained with triphenyletrazolium tetrachloride dye. What will be the color of the viable
myocardium?
(a) Blue
(b) White
(c) Red
(d) Dark Brown

94. MHC antigens are absent on:


(a) Monocyte
(b) Platelet
(c) Erythrocyte
(d) Neutrophil

95. A-26 year female presents with pallor with a hemoglobin of 9.5 mg/Dl, PCV 30 mm Hg count
of 2 million per cubic millimeters. What is the likely diagnosis?
(a) Sideroblasticanemia
(b) Iron deficiency anemia
(c) Thalassemia
(d) Folic acid deficiency

96. Fine needs aspiration cytology is not enough to diagnose.


(a) Papillary Carcinoma of thyroid
(b) Carcinoma breast
(c) Adeno carcinoma lung
(d) Follicular Carcinoma of thyroid

97. How is follicular carcinoma differentiated from bening thyroid disease?


(a) Invasion of basement membrane
(b) Presence of mitotic figures
(c) Orphan Annie eye nuclei
(d) Staining with calcitonin

98. All of these are seen in MEN – 1 syndrome except:


(a) Posterior pituitary tumors
(b) Pancreatic neuroendocrine tumors
(c) Foregut carcinoid
(d) Parathyroid hyperplasia

99. Most consistent feature of RPGN is:


(a) Crescent formation
(b) Mesangial cell proliferation
(c) IGA deposition

www.EduMedWeb.com www.EduMedWeb.com www.EduMedWeb.com


www.EduMedWeb.com www.EduMedWeb.com Page 14

(d) Loss of foot processes

100. Stain useful for identifying premalignant lasions of the lip is:
(a) Hematoxylon and eosin
(b) Toluidine blue
(c) Giemsa
(d) Alizarin red

101. All of these strains are used in tattooing except:


(a) India ink
(b) Vermillion
(c) Prussian
(d) Osmin blue

102. Digoxin is obtained from a plant product and a half of 36 hours. How does this information
help us in formulating treatment?
(a) To adjust maintenance does of digoxin required to keep the blood levels within therapeutic
range
(b) Intravenous administration in emergency and urgent dosing
(c) Long half-life permits altrernate day dosing
(d) It requires a high loading does to be administered

103. Which of these sites is least commonly preferred for insulin injection?
(a) Anterior thihg
(b) Lateral thihg
(c) Dorsum of arm
(d) Around umbilicus

104. All of these are potentially serious side effects of thinamide group of antithyroid drugs
except:
(a) Agranulocytosis
(b) Hepatic dysfunction
(c) Severe rash
(d) Anaphylaxis

105. The correct sequence of systemic absorption if a drug is administered through skin is:
(a) Scrotum > foream > scalp > postauricular skin > plantar surface
(b) Scrotum > postauricular skin > foream > scalp > plantar surface
(c) Plantar surface > scalp > foream > postauricular skin > scrotum
(d) Postauricular skin > foream > scrotum > scalp > plantar surface

106. An 80 kg male patient presented to the emergency with hypotension and have been
instructed to start him on an inotrope at a dose of 10 mcg / kg / min. Each 5ml amo of th drug
contains 200 mg drug.
You choose 2 ampules of the drug and decide to mix it with saline to make a 250 ml solution.
What should be the flow rate of the drug solution to maintain the BP of the patient (assuming 16
drops = 1ML)?
(a) 16 drops / min
(b) 8 drops / min
(c) 4 drops / min
(d) 10 drops / min

www.EduMedWeb.com www.EduMedWeb.com www.EduMedWeb.com


www.EduMedWeb.com www.EduMedWeb.com Page 15

107. About anti – epileptic drugs, which of the following is an incorrect match?
1. Focal Lamotrigine, Valproate
Seizures Carbamazepine,
Phenytoin
2. Absence Carbamazepin Ethosuximide,
Seizure Valproate
3. Myoclonic Vaiproate Clonazepam
Seizures Lamotrigine
4. Generalized Vaiproate Phenytoin,
Tonic clonic Lamotrigine Carbamzaepine
Topiramate

(a) 1
(b) 2
(c) 3
(d) 4

108. An 86-years-old lady presented with severe constipation, she is a known hypertensive on
medications for 10 years. In clinic, her BP is 157/98 mm Hg with a heart rate of 58/min. On
taking here BP in the supine position it was found to be 90/60 mm Hg. She also has a recent
history of depression. She is taking atenolol, Thaizide, impramine, haloperidol and Docusate.
What will be the next best step in management?
(a) Change atenolol and Thaizide to calcium channel blocker and ACF inhibitor and add
bisacodyl for constipations
(b) Change impramine and haloperidol to fluoxetine and respeirdone and add bisacodyl for
constipations
(c) Only add bisacodyl for constipations add continue rest of the medications
(d) Discontinue all her madications and start her on steroids

109. A-25-years-old unidentified male from roadside is brought by police to emergency room
with disorientation, altered sensorium and vomiting. He has a BP of 90/70 mm Hg. Heart rate of
110/min, temperature – 36.40c and respiratory rate of 11/min. On examination, he had bilateral
pin- point pupils. What is the suspected diagnosis?
(a) Pontine hemorrhage
(b) Opioid poisoning
(c) Dhatura poisoning

(d) Hypothermia

110. Continuous murmur can be heard in all except:


(a) Patent ductus arteriosus
(b) VCD with aortic regurgitation
(c) Pulmonary AV Fistula
(d) Coronary AV Fistula

111. A patient presents to the emergency with altered sensorium. All of these tests should be
done except?
(a) Non – contrast CT head
(b) Lumbar puncture
(c) Random blood sugar

www.EduMedWeb.com www.EduMedWeb.com www.EduMedWeb.com


www.EduMedWeb.com www.EduMedWeb.com Page 16

(d) Complete blood counts

112. A patient diagnosed with carcinoma of lung presented with a serum calcium level of 16.4
mm 01/l. What will be the first step in management?
(a) IV fluids and frusemide
(b) Immediate hemodialysis
(c) Disphosphonates
(d) Chemotherapy with gemcitabine and carboplatin

113. About Transfusion related acute lung injury (TRLI), all are true except
(a) Mortality is less than 10%.
(b) Signs and symptoms usually subsides within 2-3 weeks of onset
(c) Supportive care is the mainstay of treatment
(d) Steroids have adoubtful role in management

114. A-40-years female is diagnosed to have epilepsy and is started of phenytoin and valproate.
4 weeks later, she develops a diffuse rash all, over her body, which gradually disappears on
stopping both the drugs. In how many weeks, will the rash reapkear when she is rechalenged
with phenytoin and valproate?
(a) 1 week
(b) 2 weeks
(c) 3 weeks
(d) 4 weeks

115. Which of these infections cause a non-distributed rash?


(a) Epidemic typhus
(b) Typhoid
(c) Meals
(d) Secondary syphilis

116. Which of these is not true regarding iron supplementation in iron deficiency anemia?
(a) Administer a small diluted does first prior to infusion to look for any allergy to iron preparation
(b) Oral iron therapy should be stoped once the patient achives a hemoglobin of > 12 mg / dl
(c) If gastric intolerance to oral iron therapy occurs, all patients should be administered
parenteral forms.
(d) Parenteral iron supplementation is required in a patient with HB of less then7 mg / Dl

117. Which of the following drugs can be given in renal failure safety?
(a) Saxagliptin
(b) Sitaglipitin
(c) Vidaglipitin
(d) Linaglipitin

118. Thrombocytopenia causing drugs are all except


(a) Cephalsporin
(b) Multivitamins Containing Vitamin K
(c) NSAIDS
(d) Anti – depressants

119. Which of the following drugs can be used in benazodiazepine toxicity?


(a) Protamine sulphate

www.EduMedWeb.com www.EduMedWeb.com www.EduMedWeb.com


www.EduMedWeb.com www.EduMedWeb.com Page 17

(b) Flumazenil
(c) Barbiturate
(d) Ethyl alcohol

120. Metformin causes a servers, sometimes life – threatening side effect of lactic acidosis. All
of the following factors increase the risk of lactic acidosis except
(a) Liver dysfunction
(b) Advanced age
(c) Smoking
(d) Renal failure

121. Drug advertisement letter is a necessary component of each drug formulation and contains
various information about the drug like drug dosing, frequency and half life which of the
following information need not be given in the drug advertisement latter
(a) Research papers and other articles preparing efficiency of the drug
(b) Date of expiry of the drug
(c) Race, but serious life threatening adverse effects
(b) Common, not so serious adverse effects

122. A patient presented with history of vomiting for last three days. On examination, signs of
dehydration were present. On further investigation, the patient was found to have raised
creatinine, a ph = 7.22, pco2 = 30mm Hg, Hco3 = 9 meq, Na+ = 138 meq and k+ = 3.4meq/l
identify the acid base abnormality observed in this patient?
(a) Metabolic acidosis
(b) Metabolic alkalosis
(c) Mixed metabolic acidosis and metabolic alkalosis
(d) Mixed metabolic acidosis and respiratory acidosis

123. Calculate the base deficit in a patient of weight 75 kg with aph = 6.96, pco2 = 30mm Hg
and Hco3 = CmEq/l?
(a) 300mEq
(b) 400mEq
(c) 500mEq
(d) 800mEq

124. Metered dose inhalers are used in management of asthma. To use a MDL, it is
recommended to shake will before use breath out, bring the inhaler to your mouth, start to
breath in slowly, press the tap of you inhaler over and keep breathing in slowly until you have
taken a fall breath and then hold your breath for 10 seconds. Some patients might need a
second puff of MDI for some patient might need a second puff of MDI symptomatic relief. Which
of these instructions for reusing the MDI is not correct?
(a) Wait for iminute between two puffs
(b) Shake the inhaler again before use
(c) Carefully rinse your mouth and throat
(d) Clean the inhaler before use

125. Palivizumab is a humanized monoclonal antibody. For which of the following conditions
has it been approved for?
(a) Coxsackie virus
(b) Avain influenza
(c) Parainfluenza

www.EduMedWeb.com www.EduMedWeb.com www.EduMedWeb.com


www.EduMedWeb.com www.EduMedWeb.com Page 18

(d) Respiratery syncytial virus

126. A-2-month-old child was brought to the sub center by his mother with complaints of fever
for two days. Weight of the child is 2 kg. On examination, the child is restless and irritable, skin
pinch went back in 2 second, oral mucosa is dry and eyes were sunken. There were ten
pustules on his forehead? What should be done at the sub center?
(a) Refer to higher centre with mother giving frequent sips of ORS
(b) Immediately admit the child, give IV fluids and then refer to higher center
(c) Give first does of antibiotic and refer to higher center
(d) Send child home with few packets of ORS and call after 3 days.

127. Which of the following is the most common symptom of aortoiliac occlusive disease?
(a) Calf claudication
(b) Gluteal claudication
(c) Impotence
(d) Symptomless

128. What is the frequency of laser used in cataract surgery.


(a) 10-9 seconds
(b) 10-15 seconds
(c) 10-12 seconds
(d) 10-6 seconds

129. A sexually active male presented with warts on his penis which of these strains of HIV is
most commonly the cause of these lesions?
(a) HPV 6 and 11
(b) HPV 16 and 18
(c) HPV 1, 2 and 3
(d) HPV 3, 5

130. An abandoned dead male fetus was found and bought by police for autopsy. The following
was the report “Eyes are open, papillary membrane is absent, crown Rump length is 23 cm,
tests are at the superficial inguinal ring and weight of the fetus is 1000 g” What is the estimated
age of the fetus?
(a) 5 months
(b) 6 months
(c) 7 months
(d) 8 months

131. A-38-years-old female presented to the emergency with extensive burns. The patient had
grade 3 burns on the face back, upper arms and forearms along with singeing of hair. Which of
these is not a proof of inhalation burs?
(a) Yellow colored sputum
(b) Singeing of eyebrow and facial hair
(c) Blackish soot deposit on posterior part of tongue
(d) Hoarseness & stridor of voice

132. A-20-years-old patient presented to the emergency with altered sensorium. NCCT revealed
hydrocephalus, requiring urgent neurosurgical intervention. The patient had nobody
accompanying or any identity proof. Under which of these sections of Indian penal code, the

www.EduMedWeb.com www.EduMedWeb.com www.EduMedWeb.com


www.EduMedWeb.com www.EduMedWeb.com Page 19

neurosurgeon can perform the procedure without obtaining an informed consent from a
guardian?
(a) Section 92
(b) Section 90
(c) Section 89
(d) Section 87

133. A doctor used the same needle used in a patient with HIV inject in another patient. The
latter patient on testing found to be infected with HIV. The doctor s punishable for this
negligence according to which section of India penal code
(a) Section 269
(b) Section 203
(c) Section 202
(d) Section 166B

134. The following patient was brought for an autopsy. What is the cause of the discoloration of
the region?
(a) Hematoma formation
(b) Loss of blood
(c) Methemoglobin
(d) Sulphmethemoglobin

135. A lady is brought from village, unconscious, about 12 hours after ingesting some kind of
unknown poison her heart rate is 103/min, blood pressure in 90/50 mm Hg and respiratory rate
is 19/min. Her breath smells like kerosene. Which of the following should not be done in
emergency room?
(a) Vasopressors should be administered intravenously
(b) Atropine should be administered till singers of recovery
(c) Gastric lavage should be done
(d) Immediate airway management

136. St. Anthony’s fire refers to poisoning by:


(a) Aflatoxin
(b) Ergot alkaloids
(c) Crotalaria juncea
(d) Spainsh Fly

137. All of these are suggestive of domestic violence in a child except:


(a) Bucket Handle fractures of metaphyses
(b) Corner fractures
(c) Micro fractures in the metaphyscal region
(d) Wormian bones

138. A study was done in 3 states to see the mean blood pressure in each community. Health
workers were assigned and they visited each house in the three communities. Mean blood
pressure in each community was found and compared. What type of study design is
represented were?
(a) Cohort study
(b) Cross – sectional study
(c) Case control study
(d) Field trial

www.EduMedWeb.com www.EduMedWeb.com www.EduMedWeb.com


www.EduMedWeb.com www.EduMedWeb.com Page 20

139. A-42-year-old woman from a dry state who ingested rye for long time presented with
complaints of weakness in both the lower limbs, nausea and fatigue. Overdue couse of time,
she is completely unable to walk. What is the likely cause –
(a) Lathyrussativus
(b) Argemonemexicana
(c) Amanita
(d) Ergot alkaloids

140. CA-125 is a marker for screeing of ovarian cancer. To characterize this test,
histopathological confirmation of overian cancer was done in a cohort of patients. 60/100
woman who tested positive for this test actually had ovarian cancer. What is the negative
predictive value of this test
(a) 20 / 100
(b) 40 / 100
(c) 60 / 100
(d) 80 / 100

141. Which of these is the best determining the threshold for diagnosis of a positive test?
(a) Pre – test probability
(b) Receptor – operator curve
(c) Pearson coefficient
(d) Analysis of variance

142. In a cohort study to study association between facture and disease, the risk ratio was
calculated to be equal to 1. What does this signifies?
(a) There is no association present between the facture and the disease
(b) There is positive association between the facture and the disease
(c) There is negative association between the facture and the disease
(d) Data insufficient to comment

143. A study was conducted to find average intra – ocular pressure. IOP was measured in 400
people and the mean was found to be 25mm Hg with a standard deviation of 10mm Hg. What is
the range in which 10p of 95% of the population would be lying?
(a) 22 – 28 mm Hg
(b) 20 – 30 mm Hg
(c) 24 – 26 mm Hg
(d) 23 – 27 mm Hg

144. Hypereosinophilic syndrome is a disease characterized by elevated eosinophil count in the


blood for arrest 6 months, without any recogrizable cause, with involvement of either heart,
nervous system or bone marrow. What should be the eosinophil count for diagnosis?
(a) > 500 / mm3
(b) > 1000 / mm3
(c) > 1500 / mm3
(d) > 2000 /mm3

www.EduMedWeb.com www.EduMedWeb.com www.EduMedWeb.com


www.EduMedWeb.com www.EduMedWeb.com Page 21

ANSWER KEY

1 d 26 c 51 c 76 b 101 b 126 c
2 a 27 b 52 a 77 c 102 a 127 b
3 b 28 a 53 a 78 a 103 b 128 b
4 b 29 b 54 a 79 d 104 d 129 a
5 c 30 c 55 a 80 b 105 b 130 c
6 d 31 b 56 c 81 c 106 b 131 a
7 d 32 d 57 b 82 b 107 b 132 a
8 b 33 b 58 d 83 d 108 b 133 a
9 d 34 b 59 c 84 b 109 b 134 d
10 d 35 c 60 a 85 a 110 c 135 c
11 b 36 c 61 a 86 b 111 b 136 b
12 b 37 d 62 b 87 a 112 a 137 d
13 a 38 b 63 b 88 a 113 b 138 b
14 b 39 b 64 d 89 a 114 a 139 a
15 a 40 b 65 a 90 d 115 d 140 c
16 b 41 c 66 c 91 d 116 b 141 a
17 b 42 a 67 a 92 c 117 d 142 a
18 d 43 b 68 d 93 c 118 b 143 c
19 d 44 b 69 b 94 c 119 b 144 c
20 a 45 d 70 b 95 d 120 c 145
21 c 46 b 71 a 96 d 121 a 146
22 a 47 c 72 c 97 a 122 c 147
23 a 48 b 73 a 98 a 123 c 148
24 a 49 a 74 b 99 a 124 d 149
25 d 50 c 75 b 100 b 125 d 150

www.EduMedWeb.com www.EduMedWeb.com www.EduMedWeb.com


www.EduMedWeb.com www.EduMedWeb.com Page 1

AIIMS PG MAY 2017


(Based on Memory)
1. Which of the following an example of Anaplerotic reaction
a. Pyruvate to OAA b. Pyruvate to Acetyl COA
c. Pyruvate to lactic acid d. Pyruvate to alcohol

2. Thiamine deficiency cause decrease in energy production because decrease activity all
except.
a. PDH COMPLEX
b. Alphaketogularate dehydrogenase
c. Branch chain ketoacid dehydrogenas
d.Trans ketolase

3. In which of the following insulin / glucagon ratio is law –


a. Fat synthesis b. Cholesterol synthesis
c. Glycogen storage d. Ketogenesis

4. ATP are produced in anaerobic glycalysis with which substrate:


a. Fructose b. Glucose
c. Galactose d. Glycogen

5. Maximum thermic Effect of food is seen with


a. Carbohydrate b. Fat
c. Protein d. Thermic energy not related to macronutrients

6. The reason for high VLDL and triglyceride is diabetes mellitus?


a. High activity of lipoprotein lipase and high Activity of hormone sensitive lipase
b. High activity of lipoprotein lipase and low Activity of hormone sensitive lipase
c. Low activity of lipoprotein lipase and high Activity of hormone sensitive lipase
d. Low activity of lipoprotein lipase and low Activity of hormone sensitive lipase

7. Mode of action of phenylbutarate defective urea cycle?


a. Scavenge excess nitrogen using glutamine
b. Scavenge excess nitrogen using glycine
c. Induces enzymes of urea cycle
d. Increase ATP production

8. Which of following true about mitochondrial DNA:


a. Has 23 chromosome derived from both parents
b. Mutations less frequent than nuclear DNA
c. Less than 20y. ETC enzymes subunits are encoded
d. Has more than 30 109 base pairs

9. Decreased Insulin-Glucagon ratio is seen in all Except:


a. Glycogen storage b. Glycogen breakdown
c. Gluconeogenesis d. Ketogenesis

10. Which of the following is a editing techniques?


a. CRISPR / cas9

www.EduMedWeb.com www.EduMedWeb.com www.EduMedWeb.com


www.EduMedWeb.com www.EduMedWeb.com Page 2

b. Genome vide association studies (GWAS)


c. Scanning Force Microscopy (SFM)
d. Genexpert

11. Which of the following is responsible for Bezold jarisch reflex?


a. Serotonin b. Histamine
c. Angiotensin d. Prostaglandin

12. The oxygen carrying capacity of a 18-year old boy hemoglobin of 14g/dl is?
a. 14 b. 16
c.18 d. 22

13. What will be the oxygen carrying capacity of an 18 year old male with blood hemoglobin
level of 14gm/dl?
a. 18ml /dl b. 14ml /dl
c. 12ml /dl d. 10ml /dl

14. Role of acetone free methyl alcohal in leishman stain is:


a. Fixes the cells to side
b. Stains the cells
c. Disrupts the membrane
d. Preserves the cell in their normal metabolic state

15. A patient of anemia with hemoglobin of 5 was evaluated.WBC and platelet was normal and
reticulocyte count was 9% calculate corrected reticulocyte count:
a. 4 b. 4.5
c. 3 d. 3.5

16. If the RB gene phosphorylation is defective which of the following will happen
a. Arrest at G1 b. Arrest at G2
c. No effect on cell cycle as RB Gene phosphorylation is not needed
d. Cell cycle progresses and cell divides

17. Which of the following can change the gene expression by methylation and acetylation
without affecting the content of gene?
a. Amplification b. RNA editing
c. Epigenetics d. Delection

18. ALK gene mutation is seen in which of the following cancers?


a. Synovial sarcoma b. Ewings sarcoma
c. Fibromyo sarcoma d. Inflammatory myofibro blastic tumor

19. Which one of the following is a negative acute phase reactants.


a. Ferritin b. Albumin
c. Hepcidin d. Crp

20. RDW denotes:


a. Anisocytosis b. Poikilocytosis
c. Polychromasia d. Hypochromasia

21. Which of the following is the new screening marker to diagnose hepatic fibrosis.

www.EduMedWeb.com www.EduMedWeb.com www.EduMedWeb.com


www.EduMedWeb.com www.EduMedWeb.com Page 3

a. Transaminases b. Laminin and Hyalunoridase


c. Bilirubin and ALP d. GGT

22. Maximum risk of pancreatic cancer is seen with?


a. peutz jeghers syndrome b. Hereditary Pancreatitis
c. Fap d. Fammm syndrome

23. Mechanism of action of proteases inhibitors is?


a. Inhibition of protein translation and synthesis
b. Inhibition of assembly of virus particle
c. Inhibition of formation of viral DNA from RNA
D. Inhibition of formation of viral RNA copies

24. Components of lente insulin are?


a. 30% amorphous + 70% crystalline
b. 30% crystalline + 70% amorphous
c. Same as NPH insulin
d. Only 70% amorphous

25. Which of the following is a bacteriocidal drug against mycobacterium leprae?


a. Amoxicillin b. Ciprofloxacin
c. Erythromycin d. Ofloxacin

26. Which of the following antirheuatic drug acts by increasing extracellular adenosine?
a. Hydroxychloroquine b. Leflunomide
c. Methotrexate d. Azathioprine

27. Which of the following is true about vasomotor reversal of date in guinea pig?
a. Stimulation of 1 following by 2 Stimulation
b. 2 Stimulation by low does epinephrine followed by 1Stimulation
c. Stimulation of 1 followed block of 2 receptors
d. 2 Stimulation by norepinephrine followed by 1Stimulation!

28. Mechanism of action of oseltamivir (Tamiflu) medicine is?


a. It blocks hemoglution on the iral surface
b. It is a neuraminidase inhibitor
c. It is an inhibitor of viral transcription
d. It Inhibits viral particle uncoating

29. Mood stabilized with anti – suicidal properties?


a. Lithium b. Valprate
c. Carbamezapine d. Lamotrigin

30. Empirical treatment of meningococcal meningitis


a. Ceftriaxone b. Cefepime
c. Tigecycline d. Carbapenems

31. Antidote fibrinolytic therapy?


a. Vitamin K b. Protamine sulphatic
c. Epsilon amino caproic acid d. Alteplase
32. Which of the following is a p glycoprotein induces?

www.EduMedWeb.com www.EduMedWeb.com www.EduMedWeb.com


www.EduMedWeb.com www.EduMedWeb.com Page 4

a. Erythromycin b. Ketoconazole
c. Voriconazole d. Rifampicin

33. DOC for pneumocystis jirovecil prophylaxis in an immunocompromised patient?


a. Amoxycilin b. Cotrimoxazole
c. Rifampin d. Fluconazole

34. Storage of drugs in tissues is suggested by?


a. Large apparent Volume of distribution
b. Less excretion in urine
c. Small apparent volume of distribution
d. Excretion in saliva

35. Drug of choice for strawberry vagina is:


a. Doxycyeline b. Metronidazole
c. Fluconazole d. Amoxicillin

36. 1-3 glucan assay is not useful for which of the following?
a. Pneuocystis Jeroveci b. Aspergillis fumigates
c. Coyptococus necformans d. Conddoalbicans

37. After digestion of anti body by papain ,which of the following fragment will be produced?
a. Two fab fragment and one F (ab)2 fragment
b. One F (ab)2 fragment and digestion products
c. Two fab fragment and one FC fragment
d. One F(ab)2 fragment and one FC fragment

38. All are caused due to staphylococcal toxin except?


a. Toxin shock syndrome b. SSSS
c. Food poisoning d. Septicemia

39. All are caused by zika virus except?


a. Petechial rash b. Conjunctivitis
c. Fever with arthralgia d. GBS

40. A 25 years old patient is suspected to have rabies and a swab is taken from cornea-what is
the investigation of choice for diagnosis?
a. Seller stain for Negri bodies b. Direct immunofluorescence
c. RT-PCR d. RFFIT

41. MHC-2 is not present in which of the following cells?


a. Erythrocytes b. Macrophages
c. Dendritic cells d. cells p

42. Which of the following receptors is not present on T cells for development?
a. TDT b. PAX-S
c. CD34 d. CD3

43. What is corpora-basal index?


a. Ratio of breath of first sacral vertebrae to breadth of base of sacrum X100
b. Ratio of breath of Base of sacrum to first sacral vertebrae breadth X100

www.EduMedWeb.com www.EduMedWeb.com www.EduMedWeb.com


www.EduMedWeb.com www.EduMedWeb.com Page 5

c. Ratio of breath of Base of sacrum to breadth of lumbar vertebrae X100


d. Ratio of breath of lumbar vertebrae to breadth of Base of sacrum X100

44. What is kraissl,s lines?


a. Line of fracture in communities practice of bone
b. Line of preasure in a bene due to bullet
c. Line of redness around wound after veiling
d. Relaxed tension lines in skin

45. Jack knife position causes death due to:


a. Burking b. Traumatic asphyxia
c. Positional asphyxia d. Wedging

46. Which of the following artery is not ligated in case of an epistaxis?


a. Anterior Ethmoidal artery b. Maxillary artery
c. Internal casoted artery d. External caroid artery

47. Visual field defect seen in Optic chiasm damage is?


a. Bitemporal hemianopia b. Binasal hemianopia
c. Pie in the sky d. Keyhole defect

48. Which of the following nucleus is involved in the medial rectus palsy?
a. Superior colliculus b. Inferior colliculus
c. Olivary nucleus d. Edinger westphal nucleus

49. A wane in jugular venous pulse is:


a. Atrial contraction b. Passive filling
c. Ventricular contraction d. Ventricular ejection

50. Complication of stroke which need not be managed urgently?


a. Fever b. Spasticity
c. Neglect d. Dysphagia

51. Window period of thrombolysis in case of a stoke patient according to new guidelines is:
a. 1.5 hours b. 2.5 hours
3.5 hours d. 4.5 hours

52. CURB scoring does not include?


a. BP systolic less than 100 and diastolic less than 40
b. RR > 30 / MIN
C. Confusion and BUN > 7mmol
d. Age 65 years

53. Which symptom below is not a feature of myasthenia gravis?


a. Normal papillary reflex b. Flaccidity of muscles
c. Absent reflex d. Ptosis

54. A 28 year old female presents with complaints of fever, headache, vomiting 2nd day
postpartum, on examination no neurological deficits observed. What is the most probable
diagnosis?
a. Meningitis b. Subarachnoid haemorrhage

www.EduMedWeb.com www.EduMedWeb.com www.EduMedWeb.com


www.EduMedWeb.com www.EduMedWeb.com Page 6

c. Cortical vein thrombosis d. Migraine

55. A 30 years old mail presents to causality with life threatening hemoptysis, which of the
following procedure should not be done?
a. Bronchoscopy with cautery
b. Surgical resection of lung segment
c. Bronchial artery embolization
d. Pulmonary artery embolization

56. Which of the following is not a criteria of pyrexia of unknown origin?


a. Fever of more than 38.3 degree at 2 occasions
b. Duration greater than or equal to 3 weeks
c. Absence of immunosuppressed state
d. No definite diagnosis even after 1 week of inpatient investigations

57. Raised TSH with normal / subnormal T4, diagnosis?


a. Primary hypothyroidism
b. Subclinical hypothyroidism
c. Secondary hypothyroidism
d. Primary hypothyroidism

58. Fluid management of burns patient who sustained burns 8 hours ago came to the hospital?
a. 4 mg / kg % burns surface area -1st half in 8 hours & half remaining 16 hours
b. 5 ml / kg % burns surface area
c. 4 ml / kg % burns surface area in next 16 hrs
d. 4 ml / kg % burns surface area in next 24 hrs

59. A boy scalded with hot water, on presentation wound pink, painful. Treatment is:
a. Parafffin gauze and leave open
b. Silver sulfadiazine on alternate days
c. Collagen dressing
d. Debridement

60. What is the sensitivity of USG for detecting LN metastasis in a patient of Breast carcinoma?
a. 10 – 20 % b. 20 – 30 %
c. 30 – 40 % d. 50 – 60 %

61. A 20 weeks pregnant female presented with a thyroid ass. On FNAC it was suggestive of
papillary carcinoma. All o the following can be used as treatment modality except?
a. Lobectomy b. Total thyroidectomy
c. Hemithyroidectomy d. Radioiodine ablation

62. Which of the following is wrong regarding the management of a patient during primary
surgery in the trauma bay?
a. CT is not done in an unstable patient
b. X-ray chest and peliculas an adjaranet to primary survey
c. Cervical x-ray in primary modality
d. Chest x-ray In PA view only

63. A patient of breast carcinoma was operated and started on tamoxifen , which of the
following is the correct modality of follow up?

www.EduMedWeb.com www.EduMedWeb.com www.EduMedWeb.com


www.EduMedWeb.com www.EduMedWeb.com Page 7

a. Yearly bone scans


b. LFT at 3 – 6 month internal
c. 3 monthly clinical examination with yearly mammography
d. 3 -6 monthly routine checkups

64. Best flap for ant chest wall repair?


a. TRAM b. Pectorals muscle
c. Serratus anterior d. VRAM

65. A 40 – years – old female with a 2 cm nodule in the right breast and HPF proved metastatic
nodule in the axilla. Best management is?
a. Quandrantectomy
b. Mastectomy with local radiotherapy
c. Patey,s with adjuvant chemotherapy
d. Halsted,s operation with tamoxifen

66. A 32 years old female can for routine PAP smear testing. The PAP report came back as
HSIL what is the next step in this case?
a. Conization b. Hysterctomy
c. Colposcopy and biopsy d. HPV DNA testing

67. Which of the following is a who guideline in second stage of labor?


a. Perform labor in lithotomy position
b. Give warn compress to the perineum
c. Manual support to perineum with deflexion of head
d. Do a routine episiotomy

68. A 25 – years – old female under workup for primary infertility. On hysterosalpingography
tere was bilateral corneal block. Which is the next step in management?
a. Hydrotubation b. IVF
C. Hysteroscopy and laparoscopy guided surgery
d. Tuboplasty

69. Ureteric injury is most commonly associated with which of the following?
a. Anterior
b. Vaginal hysterectomy
c. Wertheim hysterectomy
d. Abdominal hysterectomy

70. Which of the following fits into the criteria of severe variable deceleration: variations less
than?
a. 100 beats per minute lasting for 60 seconds
b. 90 beats per minute lasting for 60 seconds
c. 80 beats per minute lasting for 60 seconds
d. 70 beats per minute lasting for 60 seconds

71. Which of the following is the best parameter assess in case of male inferlity?
a. Motility of sperms b. The number of sperm
c. Quantity of sperm d. The morphology of sperm
72. At what cervical dilation is plotting of partogram started?
a. 3 cm b. 4 cm

www.EduMedWeb.com www.EduMedWeb.com www.EduMedWeb.com


www.EduMedWeb.com www.EduMedWeb.com Page 8

c. 5 cm d. 6 cm

73. Endometriotic lesion is due to which hormone effect?


a. High estrogen b. High progesterous
c. High cholesterol d. High levels of pralactin

74. What is the biochemical marker for the condition with spade like fingers, large jaw,
acromegaly?
a. 1 GF 1 b. Prolactin
c. TSH d. Somatostain

75. March fracture following stress is seen in which part of the metatarsal?
a. Head b. Neck
c. Shaft d. Base

76. Haglund deformity is seen in which of the following joints?


a. Knee b. Wrist
c. Elbow d. Ankle

77. In polio paralysis band – knee gait is due to weakness of?


a. Gluteus maximus b. Gluteus medius
c. Quadricape d. Gastronemius

78. which of the following is the most common sequelae of traumatic dislocation of shoulder?
a. Normal shoulder healing & normal movements
b. Recurrent dislocation of shoulder
c. Rotator cuff injury
d. Subcapsular tendonitis

79. A 30 years old lady comes with a 2 month history of non scarring patchy alopecia on the left
side. There was no erythema. The diagnosis is?
a. Alopecia ereata b. Trichotillomonia
c. Telogen effluvium d. Androgenetic alopecia

80. A patient comes with skin and oral lesions. All are true except.
a. DSG 3 antibodies b. DSG 1 antibodies
c. Antibody against hemidesmosomes
d. 1Gg type is the most common

81. A child had hyperpigmented patch with hair on check sine birth- The diagnosis is?
a. Congenital melanocytic nevus
b. Verrucous epidermal nevus
c. Lentigines d. Melanoacanthoma

82. A 12 years old child comes with exudative lesions with itching inculeital fossa, popliteal
fossa and neck. The diagnousis is?
a. Dermatitis herpetiformis
b. Pemphigus
c. Psoriasis
d. Atopic dermatitis

www.EduMedWeb.com www.EduMedWeb.com www.EduMedWeb.com


www.EduMedWeb.com www.EduMedWeb.com Page 9

83. A child presented with a circulars 3 cm x 3 cm scaly patchy hair loss with itching in the
lesions.
Which of the following should be done for making the diagnousis?
a. Slit skin smear b. KOH
c. Gram stain d. Tzanck smear

84. Mutism and akinesis in a patient who appears awake and even alert is beat described as:
a. Stupor b. Oncroid
c. Twilight d. Delirium

85. A child presented with diarrhea without blood and mucus. Skin pinch was positive but goes
off within 2 seconds. The child is irritable and drinks eagerly. What is the management?
a. Ors with Zinc b. Home remedies
c. Antibiotics and ors d. IV fluids

86. A 12 years old girl present to the OPD with abdominal colic, nausea, diarrhea and litharge
and headache for last 1 year Her, irritability and anger increased since last few month and she
presented to emergency with weakness of lower limbs, what is the diagnosis?
a. Conversion disorder
b. Acute intermittent prophyria
c. Lower limb paralysis
d. AIDP with migraine

87. Which of the following statements is tree regarding Galeazzi fracture dislocation?
a. Radial collateral ligament tear with interosseous membrane tear with radical shaft fracture
b. interosseous membrane tear with triangular fibro- cartilage complex tear and ulnar shaft
fracture
c. interosseous membrane tear with ulnar shaft fracture
d. interosseous membrane tear with triangular fibro- cartilage complex tear and radical shaft
fracture

88. Which of the following is true regarding congenital CMV infection?


a. Diagnosed only by persistent presence of 1 gm antibody after 6 month
b. It is the most common of non – syndromic sensory neural heaving loss
c. All babies born are symptomatic
d. Mothers of developing countries who transmit the infection are usually symptomatic.

89. Which of the following is not seen fetal alcohol syndrome?


a. Holoprosencephaly b. Microcephaly
c. Maerocephaly d. Thinning of corpus collosum

90. A delivery assistant named sandhya was posted at PHC and sister in charge asked hear to
do P/N, after which she wrote that the station is + 1. What is meant by that?
a. Fetal head A+ level of ischial spine
b. Just below ischial spine
c. Above ischial spine in false pelvis
d. In perineum

91. A RH negative, primi pregnant lady at 38 weeks, who was ICT negative was given first does
of anti D at 28 weeks, which of the following is true regarding the management after delivery?
a. Give second dose within 72 hours depending on blood group of baby

www.EduMedWeb.com www.EduMedWeb.com www.EduMedWeb.com


www.EduMedWeb.com www.EduMedWeb.com Page 10

b. No need of second does


c. Give anti D to baby only
d. Give anti D to mom within 72 hours irrespective of blood group of baby

92. A 28 year old woman came to DPD with a twin pregnancy. She already has had 2 first
trimester abortions and has a 3- years old female child who was born at the end of ninth month
of gestation. Which of the following is her accurate representation.
a. G4P1 + 2 + 0 + 1 b. G4P1 + 0+2 + 1
c. G5p1 + 0 + 2 + 1 d. G5p0 + 1 + 1 + 2

93. A 23 years old female came with complanints of 4 month amenorrhea. The FSH and LH
were elevated above the normal value. Thyroid function test were normal. Which is the next
step in management?
a. Do a urine pregnancy test
b. Give progesterone and stop after 10 days for withdrawal bleeding
c. Estimate serum estradiol values
d. Check USG for polycystic ovaries

94. A 15 years old male presented to OPD with tall stature, on examination, he had spade like
fingers, frontal bossing, his nose and tongue was also enlarger than normal. What is the next
best step diagnosis?
a. Serum prolactin b. Serum insulin like growth factor
(1GF)
c. TSH d. Serum testosterone

95. A 30 year old female present to OPD with non specific polyarthragia, on indirect
immunofluoresence, nucleolar ANA were found to be positive, what can be future presentation
all axcept?
a. 1LD and chronic renal disease
b. Raynanuds phenomenon and selerodactyly
c. Recurrent oral and genital ulcers
d. Decreased salivation and lacrimation

96. A patient presented with ascending flaccid paralysis of both lower limbs. What serum
electrolyte abnormality should be checked for immediately?
a. Serum magnesium b. Serum potassium
c. Serum calcium d. Serum sodium

97. A 2 weeks old infant had conjunctivitis, which later developed into respiratory distress and
pneumonia Chest x-ray showed bilateral lung infiltrates. WBC count 14,300/dl. Which of the
following is the most likely organism?
a. Chlamydia trachomatis b. H. influenza
c. N. gonorrhoeae d.Streptococcus pneumoniae

98. The following are the disease frequencies in a population – Based on the number of cases
which of the following is the correct match?
Disease 1: Number of cases last week 42 this week 43
Disease 2: Number of cases last week 2 and present week 42
Disease 3: Number of cases last week 6 and present week
a. Endemic, epidemic, sporadic
b. Endemic, sporadic, epidemic

www.EduMedWeb.com www.EduMedWeb.com www.EduMedWeb.com


www.EduMedWeb.com www.EduMedWeb.com Page 11

c. Sporadic, endemic, epidemic


d. Pandemic, endemic, sporadic

99. A patient was sputum negative for TB, but chest x-ray was suggestive for TB. Which is the
next step according to RNTCP?
a. Liquid culture b. CB – NAAT
c. Line probe assay d. None of the above

100. A researcher sad he has discovers a new drug which is effective in chronic hypertensives
with a p value of < 0.10. which of the following is true regarding the same?
a. The test is 90 % reproducible
b. 90 % of test results could have occurred by chance
c. Not more than 10 % of the people benefited by the drug could be due to chance
d. 90 % f patients will be benefited by giving the drug.

101. A radiotherapist prescribes a new drug combination of chemotherapy and immunotherapy


for metastatic melanoma. It prolongs the survival. Which of the following is true in this situation?
a. Incidence redices and prevalence increase
b. Incidence remains the same and prevalence increase
c. Incidence redices and prevalence increase the same
d. Incidence increase and prevalence redices

102. Rhinosporidium seeberi belong to?


a. Fungus b. Bacteria
c. Aquatic protistam protozoa d. Virus

103. A- 20 years old male with history of fever and lymphnodes and sore throat was suspected
to have infectious mononucleosis and was ordered a paul- Bunneli test. Which of the following
is the principle of the test?
a. Hetarophic antibody test
b. Neutralisation reaction
c. Complement mediated cell damage
d. Agglutination reaction

104. A girl with Acute promyelocytic leukemia was under going treatment.she developed
techypnea, fever and pulmonary infiltrates what is the treatment of choice?
a. Cytarabine b. Dexmethasone
c. Methotrexate d. Doxorubicin

105. A 40 years old man present with Nyha class 3 dyspnea, creatine of 2.5 mg %, potassium
level of 4.5 meg/l Which of the following durry is contraindicated?
a. Carvedilol b. Spironolactone
c. Enalapril d. Digoxin

106. A patient of CAD with history of M1 2 month back; and clrabets mellitus with LDL 126, HDL
32 and triglycerides 36. Which of the following drug should be given?
a. Rosuvastain 10 mg b. A torvastain 80 mg
c. Fenofibrate and Rosuvastain d. Fenofibrate
107. A drug x was given continuous IV at a rate of 1.6 mg/min, clearance of x was 640 ml/min.
with t1/2 of 1-8h. what would be the concentration of drug after aoweving study state in mg /ml?
a. 0.002 mg b. 0.008 mg

www.EduMedWeb.com www.EduMedWeb.com www.EduMedWeb.com


www.EduMedWeb.com www.EduMedWeb.com Page 12

c. 2.88 mg d. 3.25 mg

108. all are idiosyncratic reactions of carbamazepin except?


a. Rash b. Agranulocytosis
c. Blurred vision d. Steven Johnson syndrome

109. Patient was inistered 200 mg of a drug. 75 mg of the drug is climinated in 90 minutes. If the
drug follows first order kinetics how much drug will remain 6 hours?
a. 6.25 mg b. 12.5 mg
c. 25 mg d. 50 mg

110. 70 year old patient has diabetes mellines and hyper tension. He present with CKD stage 5
and does not want to take insulin. Which antidiabetic drug will you prefer in this patient that does
not require does modification in renal diesese?
a. Linaglipin b. Vildagliptin
c. Glimepride d. Rapaglinide

111. A 40- year-old patient came with complaints of spikes of fever and difficulty in breathing.
Trancesophageal echo found out vegetations in the heart. The culture showed positive for
Burkholderia cepacia. Which of the following is the first line management? Drug of choice for
Burkholderia cepacia pneumonia is?
a. Amimoglycosides and colistin
b. Carbepenams and 3rd generation cephalasporins
c. Tigecycline and cefepime
d. Cotrimoxazole with 3rd generation ceplalsporin

112. Which of the following is accurate regarding the intracellular iron metabolism in iron
deficiency anemia?
a. Transferrin receptor I-iron responsive elements (IRE) increases Transferrin receptor M RNA
concentration and synthesis
b. Transferrin receptor I-iron responsive elements (IRE) decrease Transferrin receptor M RNA
concentration and increases synthesis
c. Apoferritin M RNA I-iron responsive elements (IRE) decrease and ferritin synthesis
d. Apoferritin M RNA I-iron responsive elements decrease and ferritin

113. Cyclin D1 resistant mantle cell lymphoma mrker:


a. Annexin V b. Fox
c. Sox 11 d. Notch

114. Normally in thymus the T cells are exposed to all the antigens of the body so they can be
identified as self and prevent Autoimmunity. Which of the following genes in involved in this
process.
a. NOTCH b .AIRE
c. NOTCH d. CTLA

115. A 29 years old female with history of polyarthralgia and back ache was investigated. She
had nuclear pattern of DNA in immunofloresence. Which of the following is the best fit in her
case?
a. Sclerodactyly, Raymand,s, phenomenon, lung fibrosis
b. Raymand,s , phenomenon, parotid swelling, Dry mouth
c. Glomerulonephritis, malar rash, oral ulcer

www.EduMedWeb.com www.EduMedWeb.com www.EduMedWeb.com


www.EduMedWeb.com www.EduMedWeb.com Page 13

d. Genital ulcer, skin nodules, lung fibrosis

116. An about was brought to the causally with complaints of the rapid heart rate. On
examination everything was normal except for episodic tachycardia and occasional extrasystole
and amblyopia. What might be the most probable case?
a. Cannabis b. Atropine
c. Chronic Nicotine poisoning d. Cocaine

117. All are caused due to staphyloccal toxin except?


a. Toxic shock syndrome b. SSSS
c. Food poisoning d. Septicemia

118. A 4 years old boy developed progressive and gradual hoarseness of voice over a period fo
the part weeks. What is the most probable diagnosis?
a. Acute epiglottitis b. Croup
c. Vocal nodule d. Respiratory papillomatosis

119. A 28 years old female was admitted to the causally with history of head trauma in a RTA.
Her eyes open to pain, she moans ad localizes towards a painful stimulus on her left hand and
away from the stimulus on her right hand, both her legs are in extended posture. what is the
GCS of the patient?
a. 5 b. 7
c.9 d.11

120. A female patient underwent laparoscopic cholecystectomy fdlowe by which she was
discharged the next day. After 3 days the patient presented with complaints of pain abdomen
and fever. On USG abdomen there was a subhepatic collection of fluid measuring 5* 1cm
dimension. What is the next step in the management of the patient?
a. Pigtail insertion and drainage b. ERCP
c. Re – explore and T – tube insertion d. Antibiotic

121. Haglund deformity is seen in which of the following joints?


a. Knee b. Wrist
c. Elbow d. Ankle

122. Which of the following statements is true regarding Galeazzi fracture distocatin?
a. Radial collateral ligament tear with interosseous membrane tear with radial shaft fracture.
b. Interosseous membrane tear with triangular fibro – cartilage complex tear and ulnar shaft
fracture.
c. Intersseus membrane tear with ulnar shaft fracture.
d. Intersseus membrane tear with triangular fibro – cartilage complex tear and radial shaft
fracture.

www.EduMedWeb.com www.EduMedWeb.com www.EduMedWeb.com


www.EduMedWeb.com www.EduMedWeb.com Page 14

ANSWER KEY

1 a 26 c 51 c 76 d 101 b
2 d 27 a 52 a 77 c 102 c
3 d 28 b 53 c 78 b 103 a
4 d 29 a 54 c 79 b 104 b
5 c 30 a 55 d 80 c 105 a
6 c 31 c 56 d 81 a 106 b
7 a 32 d 57 b 82 d 107 a
8 c 33 b 58 c 83 b 108 c
9 a 34 a 59 c 84 a 109 d
10 a 35 b 60 d 85 a 110 a
11 a 36 c 61 d 86 b 111 d
12 c 37 c 62 d 87 d 112 a
13 a 38 d 63 c 88 b 113 c
14 d 39 a 64 b 89 c 114 b
15 c 40 b 65 b 90 b 115 a
16 a 41 a 66 c 91 a 116 c
17 c 42 b 67 a 92 b 117 d
18 d 43 a 68 c 93 c 118 d
19 b 44 d 69 c 94 b 119 c
20 a 45 c 70 d 95 c 120 a
21 b 46 c 71 d 96 b 121 d
22 a 47 a 72 b 97 a 122 d
23 b 48 a 73 a 98 a
24 a 49 a 74 a 99 b
25 d 50 b 75 b 100 c

www.EduMedWeb.com www.EduMedWeb.com www.EduMedWeb.com

You might also like